Deutschland – Kultur

Wikipedia:Auskunft

Abkürzung: WP:AU, WP:AUS

Du konntest eine Information in Wikipedia trotz Benutzung der Suchfunktion der Wikipedia, einer Suchmaschine und des Archivs dieser Seite (Suchfeld unten) nicht finden? Dann beantworten Wikipedianer auf dieser Seite allgemeine Wissensfragen.

Bedenke dabei bitte:

  • So manche Antwort auf eine Frage ist im Internet per Suchmaschine schneller gefunden, als die Frage hier gestellt und beantwortet werden kann.
  • Die Auskunft ist kein Diskussionsforum. Daher ist auch nicht die Ausbreitung von Meinungen oder eigenen Theorien das Ziel, sondern die Verbreitung von belegbarem Wissen.

Für viele Anliegen gibt es spezielle Seiten:

Wie stelle ich meine Frage am besten?
  • Die Kurzanleitung erklärt, wie du eine Frage stellst.
  • Die Fragen werden ausschließlich auf dieser Seite beantwortet, nicht per E-Mail usw. Daher bitte keine persönlichen Kontaktdaten (Anschrift, Telefon, Mail) hinterlassen.
  • Wenn deine Frage ausreichend beantwortet wurde oder du eine Lösung gefunden hast, lass es uns wissen.
  • Für alle entsprechenden Fragen gelten die Hinweise zu Gesundheitsthemen, Rechtsthemen und Rechtsauskunft sowie zum Pilzesammeln.
Fragen beantworten
  • Du weißt die Antwort oder kennst wenigstens Hinweise darauf? Dann antworte so kurz wie möglich, so lang wie nötig, mit Links auf Wikipedia-Artikel oder andere Quellen, die zum Verständnis beitragen.
  • Wenn die Antwort noch nicht in der Wikipedia steht und relevant ist, vervollständige bitte die Artikel zum Thema und verlinke hier auf die entsprechenden Passagen. Sollte eine Ergänzung nicht ohne Weiteres möglich sein (z. B. weil entsprechende Belege fehlen oder es sich bei dem Geäußerten hauptsächlich um persönliche Ansichten der Autoren handelt), setze bitte einen entsprechenden Hinweis auf die Diskussionsseite der betreffenden Artikel. Die Auskunft soll nämlich auch helfen, die Artikel der Wikipedia zu verbessern.
  • Bitte rücke deine Antwort mit Doppelpunkt(en) am Zeilenanfang passend ein!

Abschnitte, die älter als 3 Tage oder seit einem Tag mit dem Baustein {{Erledigt|1=~~~~}} gekennzeichnet sind, werden automatisch archiviert. Möglicherweise findest du auch im Archiv die Antwort auf deine Frage. (Gesamtarchivdiese Wocheletzte Woche). Eine Sammlung von häufig gestellten Fragen findest du auf der FAQ-Unterseite.

10. Januar

Bitte um Übersetzungshilfe/Nachfrage bezüglich Wortbedeutungen

Ich arbeite an einer deutschen Übersetzung von en:RNLB Mona (ON 775). Bei einigen Texteilen bin ich mir nicht sicher, ob ich eher freier herangehen sollte, oder den englischen Text beachten sollte. Bei der Bedeutung bin ich mir nicht sicher, auch nicht, inwiefern den Vorschlägen von Übersetzungsprogrammen zu trauen ist. Unter anderem weiß ich im Satz "After a helicopter search, the Mona was found capsized on Buddon Sands." nicht genau, was Buddon Sands ist. Im Folgenden bin ich mir der gelungensten Formulierung überhaupt nicht sicher. Wie ist allgemein die bisherige Übersetzung gelungen? Tendenziell schlecht und verbesserungsbedürftig, oder doch recht brauchbar? Ich hoffe, jemand mit sehr guten Englisch-Kenntnissen und/oder etwas umfassenderer Fachkenntnis bezüglich der Seefahrt möge mir dabei helfen können.

Gruß, --Universal-InteressierterDisk.Arbeit 16:17, 10. Jan. 2022 (CET)Beantworten

Könnte es sich bei Buddon Sands um einen Tippfehler handeln? Es gibt Sandbänke vor der Mündung des River Duddon.--Hinnerk11 (Diskussion) 16:42, 10. Jan. 2022 (CET)Beantworten
Da mein erster Tipp auf der falschen Inselseite liegt, hätte ich noch einen. Neben dem Stationierungsort des Schiffs liegt die Marinebasis Barry Buddon Training Area. Es könnten die Strände vor dieser Basis gemeint sein, allerdings benennt Openstreetmap sie mit Barry Sands.--Hinnerk11 (Diskussion) 16:53, 10. Jan. 2022 (CET)Beantworten
Sandbank "Buddon Sands" vor der Küste bei Carnoustie. --Geaster (Diskussion) 16:57, 10. Jan. 2022 (CET)Beantworten
Die südöstliche Landspitze heißt Buddon Ness (s. Karten in commons:Category:Old maps of Dundee). Ness bedeutet wohl Landspitze (s. en:Tarbat Ness). Die Buddon Sands werden die Sandbänke vor der Landspitze sein. 62.157.2.126 17:52, 10. Jan. 2022 (CET)Beantworten
Hier ein Bericht mit Karten.--Mhunk (Diskussion) 17:59, 10. Jan. 2022 (CET)Beantworten
Es gibt den Offshore-Windpark West of Duddon Sands an der Mündung des River Duddon in der Irischen See, falls das helfen sollte. --2A02:908:2D12:8BC0:DD2E:EA53:8A19:81C6 19:14, 10. Jan. 2022 (CET)Beantworten
Upps, hatte wohl die Ansicht nicht aktualisiert bevor ich antwortete ... --2A02:908:2D12:8BC0:DD2E:EA53:8A19:81C6 19:15, 10. Jan. 2022 (CET)Beantworten
Danke erstmal für die Antworten --Universal-InteressierterDisk.Arbeit 01:20, 13. Jan. 2022 (CET)Beantworten
Sollte ich den Begriff verlinken? Übersetzen oder so lassen? --Universal-InteressierterDisk.Arbeit 02:54, 16. Jan. 2022 (CET)Beantworten
Wie ist eigentlich dieser Absatz zu übersetzen, besonders der letzte Halbsatz löste bei einem Bekannten große Probleme aus.
 The names of the men who died are commemorated on a plaque on the side of the present day boat house.[4]
The 50th anniversary of the disaster, in 2009, saw a number of memorial events organised to mark the occasion. These included a memorial concert on the actual anniversary date.[5] and talks entitled The Mona Remembered at the local church on 23 and 25 November.[6][7
--2A0A:A541:6082:0:7061:1808:1E08:2C7 23:44, 18. Jan. 2022 (CET)Beantworten
Übersetzt:
Die Namen derer, die starben, sind der Nachwelt erhalten auf einer Tafel an der Seite des heutigen Bootshauses.
Zum 50. Jahrestag des Unglücks fanden 2009 mehrere (organisierte) Gedenkveranstaltungen in Bezug auf das Ereignis statt. Darunter waren ein Erinnerungskonzert am tatsächlichen Jahrestag sowie Gespräche unter dem Titel „Erinnerungen an The Mona“ in der örtlichen Kirche am 23. und 25. November.
‚Commemorated‘, ‚to mark‘ und ‚organised‘ sind hier nur sehr ungenau zu übersetzen, nur nach dem Sinn.--Blue 🔯 15:13, 20. Jan. 2022 (CET)Beantworten
Mein Vorschlag wäre: „An die Namen der Männer, die starben, (oder einfach: der Opfer) erinnert eine Tafel an der Seitenwand des heutigen Bootshauses. Zum 50. Jahrestag des Unglücks wurde 2009 eine Reihe von Gedenkveranstaltungen organisiert, darunter ein Gedächtniskonzert am Jahrestag selbst und Vorträge unter dem Titel „Erinnerungen an die Mona“ in der örtlichen Kirche am 23. und 25. November.“ To mark the occasion lässt sich im Deutschen kaum wiedergeben, zudem ist es redundant (wozu sollen Gedenkveranstaltungen zum Jahrestag denn sonst dienen?) und talk kann sowohl ein Vortrag (evtl. mit anschließender Aussprache) als auch eine Gesprächsrunde sein; um entscheiden zu können, was hier zutrifft, müsste man die Veranstaltung selbst kennen. --Jossi (Diskussion) 00:44, 23. Jan. 2022 (CET)Beantworten
Auf manchen Seiten ist auch zu lesen, "to mark the occasion" sei eine feststehende Wendung, also ähnlich wie "by heart", das nicht (wie ich durch das Theater um Baerbocks Englisch gelernt habe) "im Herzen", "vom Herzen", "durch das Herz", "aufgrund des Herzens" oder ähnliches bedeutet, sondern "auswendig" (!). Vielleicht kann das auch eine Rolle spielen. --Universal-InteressierterDisk.Arbeit 20:25, 25. Jan. 2022 (CET)Beantworten
Der große Langenscheidt gibt für “to mark the occasion” die Bedeutungen „aus diesem Anlass / zur Feier des Tages“ an. Im vorliegenden Fall hielte ich aber Übersetzungen wie „Aus Anlass des 50. Jahrestags des Unglücks“ oder „Zur Feier des 50. Jahrestags des Unglücks“ nicht nur für überflüssig, weil sie sachlich nichts hinzufügen, sondern wegen des doppelten Genitivs auch für ausgesprochen hässlich. --Jossi (Diskussion) 00:02, 27. Jan. 2022 (CET)Beantworten
Doppelten Genitiv finde ich eigentlich recht normal, wenn derartige Sachverhalte zu erklären sind. Aber ganz gründsätzlich könnte man es auch weglassen? Gut. --Universal-InteressierterDisk.Arbeit 03:23, 30. Jan. 2022 (CET)Beantworten

19. Januar

Große Ziele mit der Blockflöte erreichen

Wenn man allein nur mit der Blockflöte recht gut ist, kann man dadurch auch große Ziele erreichen und dadurch zu viel Geld kommen? Welche Optionen hat man denn als guter Blockflötist? 185.155.125.26 11:10, 19. Jan. 2022 (CET)Beantworten

Filmstar werden. --Magnus (Diskussion) 11:18, 19. Jan. 2022 (CET)Beantworten
Ich bitte um ernsthafte Antworten. Dieses Anliegen ist mir sehr wichtig und ernst. Welche Ziele kann man nur mit der Blockflöte erreichen um auf mehr Geld ranzukommen? 185.155.125.26 11:23, 19. Jan. 2022 (CET)Beantworten
Und ich bin wahrhaftig kein Troll. Auch wenn diese Frage von mir etwas komisch klingt, muss ich die Antworten darauf genau, bist ins Detail wissen. Ich muss nämlich schauen wie und wonach ich mich mit meiner Blockflöte dann richten kann um richtig große Ziele nachzugehen. Ich brauch bitte, bitte eure Hilfe und euren Rat und nehmt dieses Anliegen von mir bitte, bitte sehr ernst. 185.155.125.26 11:27, 19. Jan. 2022 (CET)Beantworten
Richtig große Ziele muss man schon selbst finden, die sagt einem keiner (denn wenn sie jemand anders wüsste, würde sie auch jemand anders bereits verfolgen, und die Wiese wäre schon abgegrast, bis du kommst). Natürlich kannst du mit Blockflötenspiel viel Geld verdienen. Du musst dir nur überlegen, wo der Markt dafür ist – also wo du ausreichend viele Leute findest, die für dein Blockflötenspiel kumuliert viel Geld zahlen. Dazu wird es aber nicht reichen, „recht gut“ zu sein. „Recht gut“ Blockflöte spielen können Zigtausende von Leuten weltweit, dafür ist der Markt zu klein. Du musst schon einen Aspekt mitbringen, der dich von allen anderen abhebt. Kurz: Du musst Weltklasse sein. Kann man denn dein „recht gutes“ Blockflötenspiel irnkwo mal in Ohrenschein nehmen? Stell doch mal ein Video auf YouTube. Du willst doch entdeckt werden, dann gehört Klappern zum Handwerk. Kommst du gegen dieses Mädel an? Und wenn du meinst, das sei „recht gut“, dann vergleich es mal mit dem Spiel dieser jungen Dame. Und damit hörst du nicht auf, wenn du Blockflöte studierst, damit fängst du an. --Kreuzschnabel 13:30, 19. Jan. 2022 (CET)Beantworten
Hier ein Text zum Thema: Sie zählen eher zu den leisen Stars der Virtuosenszene. Die Blockflötistinnen und Blockflötisten unserer Tage sind daher überwiegend auch gleich auf mehreren Gebieten tätig: als Pädagogen, Komponisten, Herausgeber und/oder Buchautoren.
Ich denke „leise Stars“ ist als „verdienen nichts als Interpreten“ zu verstehen. Sie brauchen Zweitjobs als Musiklehrer oder ähnlichem, um finanziell über die Runden zu kommen. Da du nicht nur von einem gesicherten Einkommen sprichst (das könntest du als Orchestermusiker wohl bekommen, wobei es wohl nur wenige Stellen für Blockflötisten in Orchestern gibt), sondern von „viel Geld“ sprichst, ist die Antwort: Nein, man kann nicht zu viel Geld kommen. --::Slomox:: >< 11:31, 19. Jan. 2022 (CET)Beantworten
Du kannst auch unsere Liste von Blockflötisten durchgehen und schauen, wie und zu was die noch lebenden Vertreter es gebracht haben und noch in weiteren Quellen forschen. Es gibt natürlich auch die "Networking-Variante", sich irgendwie in die Florian-Silbereisen-o.ä.-Szene einzuschmeicheln und eher zum Herz-Schmerz-Blockflötenstar zu werden als zum technischen Virtuosen. Das ist aber (auch) eine Strategie mit sehr moderaten Erfolgsaussichten. -- 79.91.113.116 11:35, 19. Jan. 2022 (CET)Beantworten
Geh doch bitte im WP:Café weitertrollen. --Joschi71 (Diskussion) 11:41, 19. Jan. 2022 (CET)Beantworten
Halt die Klappe, Joschi! Es hat dich Keiner nach deiner Meinung gefragt! 185.155.125.26 11:24, 20. Jan. 2022 (CET)Beantworten
Wenn in acht Jahren noch kein brauchbarer Gitarrelehrer bzw. -virtuose aus Dir geworden ist, sehe ich bei einem Umsatteln auf die Blockflöte - jetzt, im Alter von 33 Jahren - auch schwarz. Mit Blockflöte alleine kannst Du vielleicht ein paar Volksschulkinder (österreichbezogen) unterrichten, falls Du eine Schule findest, die so etwas anbieten will. Die paar Wenigen, die ich im Blickfeld habe, beschäftigen sich intensiv auch mit Randbereichen, um einigermaßen ausgelastet zu sein: historische Aufführungspraxis, ander (historische Holzblasinstrumente), baroke Musikliteratur etc. --2001:871:F:5A2C:2CFE:352A:A002:3DCF 13:14, 19. Jan. 2022 (CET) PS: Wenn es Dein Hauptziel ist, zu viel Geld (zu) kommen, dann solltest Du Dir einen reichen Schwiegervater suchen. --2001:871:F:5A2C:2CFE:352A:A002:3DCF 13:18, 19. Jan. 2022 (CET)Beantworten

Andererseits: falls du irgendwann mal Ärger mit Polizei und Gericht haben solltest; so ein "ich spiele in meiner Freizeiz in einem Blpckflötenensemble" ich sicher ein extrem guter Satz, um die Richter von Deiner Integration in die Gesellschaft und die guten Zukunftsaussichten zu überzeugen. (Und sorry, aber wer Weltklasse in einem Instrument werden will, sollte intensiv in einem Alter anfangen, in dem es noch nicht möglich ist, Auskunftsfragen zu stellen. Dafür ist es zu spät.) -- southpark 15:43, 19. Jan. 2022 (CET)Beantworten

Hehe der war gut Soutpark. Ich wag mal zu behaupten, dass meine Tante eine sehr, sehr gute Blockflöten Spielerin ist. Trotzdem ihr Geld zum Leben hat sie immer als Musiklehrerin verdient. --Bobo11 (Diskussion) 16:05, 19. Jan. 2022 (CET)Beantworten

Ich kenne zufällig einen professionellen Flötisten. Sein Berufswunsch war seit der Kindheit Flötist zu werden. Er stammt aus einer Familie von Musikern, die alle mehr oder weniger professionelle Musiker waren oder sind. Keiner von denen dachte, dass man vom Flötespielen leben kann. Aber er sagte: "Doch, wenn man zu den besten fünf Flötenspielern der Welt gehört". Ich kann nicht genau sagen, wo überall er studiert hat, aber er hat sicher viel Geld für seine Ausbildung an den besten Einrichtungen in Deutschland, Schweiz und Österreich ausgegeben und ist heute tatsächlich professioneller Flötenspieler. Er spielt sie alle: die historischen Instrumente, die Bassflöte und sonst jede Größe von Blockflöte und sonst noch was. Er hat sicher ein paar tausender ausgegeben für diverse Instrumente aus allen Stilepochen. Er hat es also geschafft, aber nicht mit einer 20 Euro Blockflöte und ein paar Musikstunden und jeden Tag eine halbe Stunde üben. Auf jeden Fall musste er hart dafür arbeiten, seit der Kindheit jeden Tag mehrere Stunden üben und spielt jetzt in renommierten Orchestern und für Plattenaufnahmen. Als mittelmäßiger Blockflötenspieler kannst du in der Fußgängerzone den Hut hinstellen, in ein Orchester wirst du nicht kommen und auch für Plattenaufnahmen oder einzelne Konzerte wirst du ohne Abschlüsse und einen gewissen Namen nicht gebucht. Blockflötenspieler wird man aus Leidenschaft, nicht um das große Geld zu verdienen. Du mußt erst dein Lehrgeld in Form von Musikstunden und Meisterkursen bezahlen, dann musst du dich beweisen und wenn du bekannt bist, erst dann tröpfelt so langsam das Geld. Ich weiß nicht, ob er inzwischen in dem Bereich angekommen ist, wo auch das Geld für seine Instrumente und die Ausbildung wieder herauskommt, aber ich glaube nicht, dass der Reichtum ausgebrochen ist.--Giftzwerg 88 (Diskussion) 16:20, 19. Jan. 2022 (CET)Beantworten

„Viel Geld“ bestimmt nicht, das funktioniert in der Musikbranche schon in normalen Zeiten nicht und derzeit schon überhaupt nicht. Ein stetiges Einkommen, von dem man tatsächlich leben kann, scheint am ehesten bei Instrumentallehrern möglich zu sein. Vielleicht zu deiner Anregung kann ich folgende Geschichte beitragen: Eine Freundin von mir, Opernsängerin (Konzert- bzw. Bühnenkarriere hat bei ihr leider nicht sollen sein), arbeitet seit dem Studium freiberuflich mit privaten Schülern (Klavier und Gesang), wobei ihre persönliche Situation es fast von Anfang an erfordert hatte, auch Fernunterricht zu geben. Als dann Covid kam, war sie nicht nur eine erfahrene Lehrerin, sondern hatte, weil sie Fernunterricht inzwischen auf sehr hohem Niveau konnte, gegenüber der Konkurrenz einen Riesenvorsprung. Sie verdient nicht „viel Geld“, aber mehr als jemals zuvor. Voraussetzung ist natürlich eine reguläre Ausbildung 1. mit dem Instrument und 2. in Musikpädagogik, sowie 3. eine Expertise in einem Bereich, die von anderen Instrumentallehrern nicht abgedeckt wird, und last but not least 4. ein Händchen für Selbstvermarktung. Viel Erfolg wünscht --Stilfehler (Diskussion) 16:57, 19. Jan. 2022 (CET)Beantworten

Mit der bekannteste Blöckflötist im deutschsprachigen Raum dürfte Maurice Steger sein. Übliche Laufbahn: Musikstudium mit Hauptfach (Block)flöte, Gewinn internationaler Preise. Sein virtuoses Spiel ist hier zu bewundern: https://www.youtube.com/watch?v=hggISFswKcw . So eine Karriere mit einem Nischeninstrument machen sicherlich die Allerwenigsten. Oft enden selbst hervorragende Pianisten in Musikschulen oder als Privatlehrer, wo sie unbegabten Kindern den "fröhlichen Landmann" (https://www.youtube.com/watch?v=JWdFd-a9zhc) beibringen müssen. --Doc Schneyder Disk. 17:51, 19. Jan. 2022 (CET)Beantworten

In der heutigen Zeit gibt es doch Youtube, Tiktok und NFT. Da muss niemand mehr mit dem Blockflötenorchester der Jugendmusikschule durch die Seniorenheime tingeln. @185.155.125.26, einfach Deine Blockflötenvirtuosität als Audio oder Video aufnehmen und dann als NFT verkaufen und/oder das Video bei Youtube bzw Tiktok einstellen. --Rôtkæppchen₆₈ 23:54, 19. Jan. 2022 (CET)Beantworten
Hab ich ja oben schon mal gefragt, wo man den Fragesteller auf YT mal hören kann. Kam aber nix. Wenn er einfach nur drauf wartet, dass ohne Kostprobe, Vorspiel und Auswahlverfahren jetzt die Met anruft und ihn als Flötisten fest anstellt für eine Million jährlich … naja, ausgeschlossen ist es nicht, rein logisch gesehen. --Kreuzschnabel 07:58, 20. Jan. 2022 (CET)Beantworten
Ich danke euch allen. Jetzt weiß ich Bescheid und kenn mich aus. Kann man als Blockflötist eigentlich auch als Straßenmusiker tätig sein und wenigstens als Straßenmusiker so viel Geld kassieren dass man gut über die Runden kommt wenn man mit der Blockflöte täglich auf der Straße musiziert? 185.155.125.26 11:23, 20. Jan. 2022 (CET)Beantworten
Probier’s doch einfach mal eine Stunde aus, wie viel da zusammenkommt... --Jossi (Diskussion) 12:12, 20. Jan. 2022 (CET)Beantworten
Tätig sein können: natürlich. Tätig sein dürfen: du brauchst in der Regel eine Genehmigung vom Ordnungsamt, die vermutlich eine Gebühr kostet und meist an Bedingungen geknüpft ist (etwa: nicht länger als eine Stunde am selben Platz musizieren). Damit Geld kassieren: Dazu musst du so gut sein, dass a) nicht nur Flanierer stehenbleiben, sondern auch Leute, die eigentlich was anderes vor hatten, und b) ausreichend viele Zuhörer so begeistert sind, dass sie sich unaufgefordert von einem Teil ihres Kleingelds trennen. Solche Begeisterungen für Straßenmusiker habe ich persönlich eher selten, ich erinnere mich an zwei, ein mitreißender Bluesgitarrist in Stuttgart und ein Bajanist in Karlsruhe, der auf seiner Quetsche Bachsche Orgelfugen in Vollendung spielte. Generell ist Straßenmusik ein hartes Brot, dazu muss man geboren sein. --Kreuzschnabel 12:12, 20. Jan. 2022 (CET)Beantworten
Da es oft und gerne unterschätzt wird: mit dem Hut in der Fußgängerzone kann man verdammt viel Geld zusammenkriegen. Eine Bekannte von mir hat sich so in wenigen Jahren ihr EFH zusammengesungen. Als Opernsängerin wäre sie eher mittelmäßig gewesen (obwohl sie das studiert hatte) und hätte bei weitem nicht so gut verdient. Natürlich muß man bei der Straßenmusik auch betriebswirtschaftlich klug vorgehen und seine Nischen nutzen: viele osteuropäische Musiker sind richtig gut. Beim Gesang können Sie aber i. a. nicht punkten, weil sie auf Deutsch ihren Akzent nicht wegkriegen und sie eben nicht "deutsch" aussehen, und deswegen werden sie z. B. nicht für die Mucke im Altenheim mit Schlagern und Volksliedern engagiert - da muß man nämlich auch deutsche Volks-, Kinder- und Kirchenlieder drauf haben. --77.1.121.114 11:37, 21. Jan. 2022 (CET)Beantworten
Ich hatte Anfang der 1980er Jahre mich mit meinem Cello auf die "Kettwiger" in Essen (die Fußgängerzone) auf eine Bank gesetzt und habe schlicht meine Übungen, Etuden und "Hausaufgaben" vom Musikschulunterricht gemacht. Der Stundenlohn von weit über 40 DM war echt sensationell. Der Durchschnittslohn lag in der Stunde sonst bei rund 15 DM. Genehmigung war ein Fremdwort. Aber damals gab es noch keine anderen Musiker aus Südamerika oder Osteuropa. Ging natürlich nur im Sommerhalbjahr und nur bei schönem Wetter... --2003:D5:FF2A:AB00:910:9604:F37F:7FAC 11:57, 21. Jan. 2022 (CET)Beantworten
Ach, Du warst das... Hab's auf dem Weg von und zur Schule eigentlich auch ganz gern gehört (und mir wie viele "sind ja nur Übungen" gedacht), aber mehr als ein paar Groschen hast Du von mir bestimmt nicht gekriegt - mein Taschengeld habe ich damals doch lieber in Eiskugeln investiert. (Eine Kugel 20 Pf., kann das hinkommen?) --77.8.184.122 12:26, 22. Jan. 2022 (CET)Beantworten
Meistens zwischen Baedecker (gibts schon lange nicht mehr) und dem Kloster/Dom. Da wars am Besten. Am Handelshof wars aber auch nicht schlecht... Gut waren auch die Pommes in der Unterführung zur Ubahn U18... --2003:D5:FF00:AB00:255B:D866:2B75:6170 15:32, 25. Jan. 2022 (CET)Beantworten
Service: [1] --Kreuzschnabel 12:17, 20. Jan. 2022 (CET)Beantworten
Wie oben erwähnt, kann man Blockflöte mittlerweile als Hauptinstrument studieren und wird sich anschließend wahrscheinlich auf Barokmusik spezialisieren. Oder man etabliert dieses Instrument weiter in den anderen Gattungen. Als vollwertiges Instrument wird es mittlerweile angesehen. --Elrond (Diskussion) 15:13, 23. Jan. 2022 (CET)Beantworten
Noch mehr Service: Georg Philipp Telemann hat eine ganze Reihe von Solokonzerten und Triosonaten für Blockflöte geschrieben. Wenn du die Solopartien darin gut spielst, dann stell dich mal bei einem Produduktionsteam vor. Dann können wir weiter sehen. --Altkatholik62 (Diskussion) 01:35, 26. Jan. 2022 (CET)Beantworten

Die Antwort "wer Weltklasse in einem Instrument werden will, sollte intensiv in einem Alter anfangen, in dem es noch nicht möglich ist, Auskunftsfragen zu stellen" war wohl die treffendste. Wer hier in der Auskunft danach fragt, sorry, aber für den ist der Zug abgefahren.
Nichtsdestotrotz kann das Blockflötelernen sinnvoll sein, aus vielerlei Gründen. Was wäre das familiäre Weihnachtsfest ohne blockflötefiepende Kinder. Eine Blockflöte kann Gold wert sein, wenn es darum geht, eine Ehe zu retten, wie wir seit "Pappa ante portas" wissen. Als Straßenmusiker kriegt man sicher ein paar Euro in den Hut, und sei's nur, damit man endlich aufhört.
Im ganzen halte ich es mit der alten Weisheit "Was ist schlimmer als eine Blockflöte?" "Zwei Blockflöten." --217.239.4.223 23:55, 27. Jan. 2022 (CET)Beantworten

Als Zweitinstrumente wären da noch Triangel und Trillerpfeife.--Giftzwerg 88 (Diskussion) 14:27, 30. Jan. 2022 (CET)Beantworten

20. Januar

Quelle, Gründe für „Omicron“

Hallo. Die WHO hat eine SARS-CoV-2-Variante englisch als „Omicron“ bezeichnet (dtsch. Omikron), also mit dem 15. Buchstaben im griechischen Alphabet. Zwei Buchstaben wurden bei der Benennung als neu registrierte Variante übersprungen: „Ny“ (13. Buchstabe) und „Xi“ (14. Buchstabe). Als hautsächliche Quelle für „Omicron“ dürfte diese Bekanntgabe gelten:

Dortselbst steht aber nicht viel über die Intentionen bei der Benennung. Zuständig sei die TAG-VE, die „Technical Advisory Group on SARS-CoV-2 Virus Evolution“, was übersetzt ungefähr „Technische Beratungsgruppe hinsichtlich der Evolution des Virus SARS-CoV-2“ bedeutet.

Mir ist zwar bekannt, dass die Verwechslungsgefahr der Grund für die Auslassung im Alphabet sein soll; aber ich habe keine Quelle seitens der WHO selbst gefunden, wo die Idee herkommt bzw. notiert wurde, Buchstaben zu überspringen. Beim ZDF-Nachrichten-Panorama wurde am 28.11.2021 z. B. Folgendes verlautbart (>>Warum die Corona-Variante "Omikron" heißt<<):

  1. >>Vor Omikron wären nun aber noch die Buchstaben Ny und Xi an der Reihe gewesen. Ny, das auf Englisch Nu heißt, klinge zu sehr nach "new" (deutsch: "neu") und wäre daher missverständlich gewesen, hieß es dazu von der WHO.<<
  2. >>"Xi wurde nicht verwendet, weil es ein verbreiteter Nachname ist", erklärte die WHO.<<
  3. >>Es gibt aber einen sehr gewichtigen Namensträger: Den chinesischen Staatschef Xi Jinping.<<

Es wird dort „Quelle: dpa“ angegeben. So (oder so ähnlich) schreiben es viele Medien, ohne ein Dokument der WHO als direkte Quelle für einen oder mehrere von den drei oben erwähnten Punkte zu benennen (unsortierte Beispiele: [2], [3], [4], [5], [6], [7]). Die New York Times wird ebenfalls zitiert (>>How Omicron, the New Covid-19 Variant, Got Its Name<<), lässt sich aber nicht kostenfrei lesen.

Frage: Gibt es direkte Quellen bei der WHO bzw. bei assoziierten Gremien (z. B. für: >>"Xi wurde nicht verwendet, weil es ein verbreiteter Nachname ist“) und welche Quellen sind das? --2001:16B8:576E:A00:179:2465:8980:E140 14:47, 20. Jan. 2022 (CET)Beantworten

<offtopic>Ist „Omikrönchen“ eher die Verkleinerungsform von Omikron oder aber doch eine Zusammensetzung der Koseform Omi und der Verkleinerungsform Krönchen?</offtopic> --2003:D2:4F16:1E1A:E8E1:1895:911A:FEBE 23:47, 20. Jan. 2022 (CET)Beantworten

Einen häufigen chinesischen Nachnamen zu nehmen wäre wohl wirklich problematsich gewesen, auch wenns nur aus Zufall gewesen wäre. Das mit Nu und New ist auch einleuchtend. --MrBurns (Diskussion) 16:41, 21. Jan. 2022 (CET)Beantworten

Denen muss nur jemand erzählen, wie man ν [nyː] korrekt ausspricht. --Rôtkæppchen₆₈ 16:53, 21. Jan. 2022 (CET)Beantworten
Definiere „denen“. Die WHO weiß es vermutlich schon. Andererseits scheint mir die kurzfristige Umstellung der Aussprachegewohnheiten der vollständigen globalen englischen Sprachgemeinschaft einen Aufwand zu erfordern, zu dem das Wörtchen „nur“ nicht mehr so recht passen will. --Kreuzschnabel 17:36, 21. Jan. 2022 (CET)Beantworten
Wie kommst du darauf, dass die deutsche Aussprache griechischer Buchstaben die korrekte ist? --Digamma (Diskussion) 21:01, 21. Jan. 2022 (CET)Beantworten
Sorry, ich vergaß meinen Lateinunterricht. Die dort gelehrte Aussprache hat mit der damaligen Realität nichts zu tun, sondern viel eher mit hochdeutscher Phonologie. Vielleicht ist es ja mit den Namen der griechischen Buchstaben genauso. Ich musste die sowieso nur für den Matheunterricht lernen. --Rôtkæppchen₆₈ 23:34, 21. Jan. 2022 (CET)Beantworten
Wenn der Artikel Ny korrekt ist und ich die Lautschrift richtug verstehe war die altgriechische Aussprache tatsächlich Nü. --MrBurns (Diskussion) 23:39, 21. Jan. 2022 (CET)Beantworten
International und trotzdem lokal    Hallo, internationale Ausdrücke sind wohl unter dem Strich doch nicht so international, wie man es gern hätte. Auffällig ist das in erster Linie bei der Aussprache. So wird „Putin“ im Englischen ungefähr wie putn und „Makron“ in China wohl ungefähr wie makalon gesprochen.    (‑;  Das soll hier keine solide Umschrift der Aussprache sein; „ungefähr“ eben.  ;‑)    Sogar innerhalb derselben Sprache kann es Variationen geben, z. B. bei „SARS-CoV-2“: „sars-koff-zwei“ oder „sars-koff-zwo“.
Seit die ganze Welt neue Wörter für die Virus-Varianten braucht, merkt man es deutlicher als zuvor, dass griechische Buchstaben in verschiedenen Sprachen auch unterschiedlich geschrieben werden. Beispiele sind Bêta im Französischem und Omikron im Deutschen (statt Beta bzw. Omicron im Englischen).
Auffällig ist, dass von den zwölf gegenwärtig bei cov-lineages.org angezeigten Virusvarianten, denen griechische Buchstaben zugeordnet sind, nur eine Variante bzw. ein Buchstabe einsilbig ist, nämlich µ (en:Mu, de:My). Alle anderen elf Varianten werden mit mehreren Silben geschrieben und gesprochen. Siehe "Lineage List" unter cov-lineages.org; es werden dort 12 WHO-Namen angegeben (22. Jan. 2022 ), die ich hier mit ihren jeweils grundlegenden Pango-Linien (in Klammern) wiedergegeben habe:
  • Alpha (B.1.1.7), Beta (B.1.351), Delta (B.1.617.2), Epsilon (B.1.427), Eta (B.1.525), Gamma (P.1), Iota (B.1.526), Lambda (C.37), Mu (B.1.621), Omicron (B.1.1.529), Theta (P.3) und Zeta (P.2).
Da ν (en:Nu, de:Ny) von der WHO bei der Namensvergabe übersprungen wurde, braucht man es beim Sprechen und Hören auch nicht von µ (en:Mu, de:My) unterscheiden. Es könnte ein Vorteil mehrsilbiger Wörter sein, dass man sie besser auseinanderhalten kann als einsilbige. „en:Omicron“ oder „de:Omikron“ ließe sich z. B. auch dann gut von „Omega“ unterscheiden, wenn jemand bspw. das „R“ im Wort ihren oder seinen Sprechgewohnheiten anpassen würde und jemand anderes entsprechend ihren oder seinen Hörgewohnheit ein „L“ heraushören würde.
MfG --Dirk123456 (Diskussion) 13:28, 22. Jan. 2022 (CET)<Nachträgl. Links korrigiert. --Dirk123456 (Diskussion) 13:44, 22. Jan. 2022 (CET)>Beantworten
Nun, Theta und Zeta in der Aussprache zu unterscheiden, dürfte nicht überall einfach sein. Insbesondere wenn das th in Theta wie englisches th ausgesprochen wird und das z in Zeta spanisch. --Digamma (Diskussion) 23:19, 22. Jan. 2022 (CET)Beantworten
Der Artikel gibt die deutsche (und die neugriechische) Aussprache wieder. --Digamma (Diskussion) 23:16, 22. Jan. 2022 (CET)Beantworten
Nü-ni-nu    (bezieht sich auf: 23:39, 21. Jan. => 23:16, 22. Jan.)    Hallo @MrBurns, ich hätte es auch fast so verstanden, dass „Ny“ (gesprochen „Nü“) wohl eher die „altgriechische Variante“ sein müsste, da nur „griechisch“, „neugriechisch“ und „heutige Aussprache“ im Wikipedia-Artikel auftauchen, nicht aber „deutsch“. Erster Satz des Artikels Ny:
Da ich aber mal die Sprache gewechselt hatte (z. B. en:Nu), sah ich, dass es wohl eher um „deutsches Griechisch“, „englisches Griechisch“ usw. geht, falls man das so nennen kann.    (‑;  Ich hege die Hoffnung, dass niemand die Angabe von Virus-Varianten-Namen mithilfe altgriechischer Phonologie verlangen möge.  ;‑)    Wie dem auch sei, die kurzen kompakten Codes in Klammern am Anfang von Wikipedia-Artikeln helfen oft mehr denjenigen Lesenden, welche mit dem Thema bereits gut vertraut sind.
@Digamma, danke für den Hinweis!
MfG --Dirk123456 (Diskussion) 17:58, 29. Jan. 2022 (CET)Beantworten
Soviel ich mitbekommen hab (vor Allem in meinem Physikstudium) wird es auf deutsch üblicherweise "Nü" ausgesprochen, also wohl wie auf altgriechisch, auf Englisch allerdings genauso wie "new". --MrBurns (Diskussion) 19:15, 29. Jan. 2022 (CET)Beantworten
[ni] geht gar nicht. Habe mal auf der Disku angeklopft deswegen. --94.219.4.152 02:57, 30. Jan. 2022 (CET)Beantworten
Hallo, ich zweifle ja nicht an der Sinnhaftigkeit der drei Argumente, die ich beim ZDF-Nachrichten-Panorama (28.11.2021) und in ähnlicher Art auch in anderen Medien gefunden und bei meiner ursprünglichen Frage (14:47, 20. Jan.) wiedergegebenen habe. Es geht mir um die direkten Quellen dazu.
Alle von mir gefundenen Nachrichten bieten keinen Link auf die Aussagen selbst, die von der WHO getroffen worden sein sollen. Da in den Medien nicht nur mit indirekter Rede gearbeitet wurde ("hieß es dazu von der WHO"), sondern auch mit direkter (>>"Xi wurde nicht verwendet, weil es ein verbreiteter Nachname ist", erklärte die WHO.<<), dachte ich, dass es dazu Belege geben müsse, die von der WHO selbst stammen. Hat sich dazu vielleicht bloß jemand irgendwo anonym geäußert und die Medien schreiben das einfach voneinander ab? Andererseits sprechen die Formulierungen: "hieß es dazu von der WHO" und "erklärte die WHO" nicht dafür, dass es um inoffizielle Aussagen geht. Gibt es vielleicht bloß Twitter-Tweeds, Interviews und Talkshows, in denen sich Personen mit Rollen bei der WHO geäußert haben? --2001:16B8:5779:E00:43D:50A6:E437:9AC3 09:11, 23. Jan. 2022 (CET)Beantworten
Es könnte sich hierbei mglw. um Inhalte einer direkten Kommunikation zwischen den Medien und der WHO handeln. Da drucken Medien dann i.d.R. nicht den Antwortbrief ab oder stellen Emails online, sondern verarbeiten die Inhalte bzw. Teile davon in ihren Artikeln. --94.219.125.69 04:18, 26. Jan. 2022 (CET)Beantworten
Es klingt aber trotzdem mehr wie ein weitergetragenes Gerücht, wenn nicht genau mitgeteilt wird, wer wem wo und wann etwas gesagt haben soll. --2001:16B8:57B0:1300:6853:DC2D:B636:980B 16:00, 29. Jan. 2022 (CET)Beantworten
Benennung: Ergebnis und Vorgang    Hallo, ich denke, dass der Vorgang der Auswahl von „Rufnamen“ der Virus-Varianten – zumindest vonseiten der WHO selbst – deshalb nicht allzu ausgiebig kommuniziert wird, weil das Ergebnis in den Hintergrund rücken würde. Es gibt sicherlich sehr viele Für- und Wider-Argumente, die von der WHO, bzw. vom TAG-VE, bei „Omicron“ zur Diskussion gekommen sind. Das Ziel, nämlich die Bekanntgabe des WHO-Namens selbst, erschien da wohl wichtiger, als es die Bekanntgabe sämtlicher Hintergründe bei dieser Benennung wäre.
Nur mal angenommen, es wäre so gewesen, dass bei einer internen Sitzung unter vielen anderen Argumenten von jemanden kurzzeitig erwogen worden wäre, einen Buchstaben im griechischen Alphabet deshalb nicht zu verwenden, weil es der 13. ist: Es wäre vermutlich nicht hilfreich, das besonders hervorzuheben. Das ausschlaggebende Argument wäre es vermutlich nicht und eine Nebeninformation würde die wichtigere Kernbotschaft zum WHO-Namen nur verwässern.
Da natürlich trotzdem nachgefragt wird, wieso zwei griechische Buchstaben übersprungen worden sind, hat man für die beiden ausgelassenen Buchstaben (en:Nu und en:Xi bzw. Ny und Xi) seitens der WHO vermutlich jeweils das am markantesten und am nachvollziehbarsten erscheinende Argument herausgearbeitet, um es an die Presse weiterzureichen. Unter den Nachrichtenartikeln zum Thema stehen meist Presseagenturen als Quelle, häufig „dpa“.
Das beruht dann wohl, wie zuvor auch schon geschrieben, möglicherweise auf direkter Kommunikation zwischen einigen Presseagenturen und der WHO.
MfG --Dirk123456 (Diskussion) 22:39, 29. Jan. 2022 (CET)Beantworten
So viel Text und doch kein einziger neuer Wissensaspekt. --94.219.4.152 03:00, 30. Jan. 2022 (CET) --94.219.4.152 03:00, 30. Jan. 2022 (CET)Beantworten

26. Januar

Impfpflicht vs. Impfquote

Die Politik diskutiert aktuell darüber, angesichts der etwas mäßigen Impfquote hierzulande eine Impfpflicht einzuführen. Man sollte also annehmen dürfen, daß diese dazu dienen sollte, die Impfquote zu erhöhen. Nun finde ich allerdings nur Studienbelege dazu, daß die Einführung einer Impfpflicht eher nicht dazu führt, daß die Betroffenen sich vermehrt impfen lassen. Bestenfalls ist es gelungen, die Impfquote für eine bestimmte Impfung leicht zu erhöhen, was dann allerdings zu vermehrter Reaktanz geführt hat, so daß u.a. weniger gegen andere Erkrankungen geimpft wurde. Meine Frage ist daher:

Gibt es Belege, die die Annahme stützen, daß die Einführung einer Impfpflicht mit nicht geringer Wahrscheinlichkeit dazu führen würde, die Impfquote effektiv zu erhöhen?

In Österreich glaubt man anscheinend derzeit eher nicht daran, daß deren kommende Impfpflicht irgendetwas nützen wird. Hierzulande hat sich u.a. Cornelia Betsch intensiv mit solchen Fragen beschäftigt, die das auch eher kritisch sieht. Die Ergebnisse der COSMO Studie zum Thema Impfpflicht lassen m.E. ebenfalls wenig Raum für Hoffnung, daß eine solche Maßnahme zu einer wesentlichen Erhöhung der Impfquote führen würde, und die Zustimmungsrate zur Impfung nimmt anscheinend bereits wieder ab. Ich finde es darum eher wichtig, zunächst mal eine faktenbasierte Abschätzung dahingehend zu treffen, wie viele Menschen sich bei Einführung einer Impfpflicht zusätzlich impfen lassen würden und wie viele sich dann erst recht nicht impfen lassen und/oder weitere Schutzmaßnahmen umgehen würden. Ich sehe keinen Sinn darin, über die Notwendigkeit einer Impfpflicht zu diskutieren, solange völlig unklar ist, ob das überhaupt dabei helfen würde, die Impfquote zu erhöhen. Etwas beunruhigend erscheinen mir in diesem Zusammenhang wiederholt gehörte Argumentationen dahingehend, daß es unfair sei gegenüber den Geimpften, wenn man keine Impfpflicht einführt, weil das doch eher wahltaktisch anmutet als wissenschaftlich begründet.

Nebenfrage: Zustimmung zur Einführung einer Impfpflicht kommt aktuell fast ausschließlich von Geimpften. Es kommt mir falsch vor, wenn man ausgehend davon sagt, es gäbe eine breite Zustimmung, obwohl fast nur diejenigen dafür sind, die davon nicht selber betroffen sind. Gibt es für diese Art von Schwäche/Bias/Fehler einen Fachbegriff? --94.219.125.69 06:48, 26. Jan. 2022 (CET)Beantworten

Bitte drösel Deinen Text in Wissensfrage und Plauderei auf. Die Plauderei erledigst Du dann im Wikipedia:Café, denn dazu ist hier nicht der richtige Ort. --2003:F7:DF08:9C00:8135:2E42:BFBA:8CC0 07:35, 26. Jan. 2022 (CET)Beantworten
Der Text ist bereits untergliedert in Fragen und Erläuterungen der Fragen. Es wäre hilfreich, wenn Du etwas zur Beantwortung der Fragen beitragen würdest. --94.219.127.230 02:53, 27. Jan. 2022 (CET)Beantworten
Nebenfrage: Zustimmung zur Einführung einer Impfpflicht kommt aktuell fast ausschließlich von Geimpften. Es kommt mir falsch vor, wenn man ausgehend davon sagt, es gäbe eine breite Zustimmung, obwohl fast nur diejenigen dafür sind, die davon nicht selber betroffen sind. Gibt es für diese Art von Schwäche/Bias/Fehler einen Fachbegriff?. Ist das eine Schwäche oder ein Fehler? Zustimmung zur Inhaftierung von Mördern kommt fast ausschließlich von Personen, die selbst keine Mörder sind (das soll notabene kein Vergleich sein, sondern nur eine Analogie). Zustimmung zu Bußgeldern für Geschwindigkeitsüberschreitungen kommt fast ausschließlich von Personen, die selbst nicht andauern zu schnell unterwegs sind usw. Der passende Begriff für mich lautet demokratische Mehrheitsentscheidung oder vielleicht auch Entscheidung auf wissenschaftlich-epidemiologischer Basis. -- 79.91.113.116 10:37, 26. Jan. 2022 (CET)Beantworten
In welcher Rechtsnorm sind die Begriffe "demokratische Mehrheitsentscheidung" und "Entscheidung auf wissenschaftlich-epidemiologischer Basis" definiert? (Ist wirklich "epidemiologisch" gemeint?)--Wikiseidank (Diskussion) 14:33, 26. Jan. 2022 (CET)Beantworten
Demokratie, Epidemiologie. (Der Fragesteller fragte nach Fachbegriffen, nicht nach "Rechtsnorm", das ergibt hier auch wenig Sinn. -- 79.91.113.116 17:12, 26. Jan. 2022 (CET)Beantworten
Ist das eine Schwäche oder ein Fehler? Da bin ich unsicher, darum die Variation in der Frage. Wenn man sich mal den Verlauf anschaut, kann man sehen, daß die Zustimmung zunächst deutlich geringer war, als die Impfquote auch noch gering war. Dann steigen Impfquote und Zustimmung zu einer Impfpflicht parallel an, wobei Geimpftte sich zunehmend für eine Impfpflicht aussprechen. Es sieht daher danach aus, daß Personen, nachdem sie geimpft wurden, eher einer Impfpflicht zustimmen. Das Mörderbeispiel paßt hier insofern gar nicht und auch nicht das Raserbeispiel, als man da für eine schädigende Handlung sanktioniert wird, während man in Sachen Impfpflicht für Nichtstun bestraft würde, wozu mir am ehesten noch die Anschnallpflicht einfallen würde als Beispiel. Eine "demokratische Mehrheitsentscheidung" kann es m.E. nicht sein, weil Demokratie den Schutz von Minderheiten erfordert, weil man andernfalls eine Diktatur der Mehrheit hätte. Eine "Entscheidung auf wissenschaftlich-epidemiologischer Basis" kann es m.E. nicht sein, weil die Studienlage das AFAICS nicht hergibt, wie ich in der Erläuterung zu meiner zentralen Frage dargelegt habe. Darum würde ich eher einer Analogie ziehen zu bspw. einer mehrheitlich religiösen Gesellschaft, die zu einer Maßnahme befragt wird, die nur Atheisten betreffen würde, was ganz offensichtlich problematisch wäre und darum eigentlich begrifflich gefaßt sein müßte. Der passende Fachbegriff zu diesem Beispiel könnte mglw. auch zu meiner Nebenfrage passen. --94.219.127.230 02:53, 27. Jan. 2022 (CET)Beantworten
Impfpflicht, die sich auf den Arbeitsplatz bezieht, ist sehr wirksam. Impfquoten von 99% nach Einführung sind nicht ungewöhnlich [8] -- 79.91.113.116 09:30, 27. Jan. 2022 (CET)Beantworten
Okay, es erscheint also als möglich, in einzelnen Unternehmen eine sehr hohe Impfquote zu erreichen. Bleibt die Frage, ob sich dadurch auch die landesweite Impfquote erhöhen läßt oder ob in diesem Artikel bloß über einige wenige, nicht repräsentative Vorzeigefälle berichte wurde. --94.219.127.230 01:27, 28. Jan. 2022 (CET)Beantworten
Mich erstaunt, wie sehr die Impfpflicht in den Fokus gerückt ist. An sich ist es aus vielen Bereichen (Verhandlungstechniken, Nudging, Verhaltensexperimente usw.) klar, dass Strafen eher dazu führen, dass man sich ein Gruppe von Gegnern heranzüchtet. Der richtige Weg wäre eigentlich Belohnung, etwa einen Steuerbonus von 100 Euro bei Impfung oder soetwas. Man muss die sicher dazu anlaufenden Untersuchungen aus Österreich abwarten, aber meine Erwartung ist, dass man mit einer Impfpflicht keine Verbesserung erzeugt. Auch wenn die meisten Argumente gegen die Impfung ziemlich schwach sind, Impfgegner glauben ja selbst an diese Argumente. Die momentan diskutierte Idee einer Beratungspflicht halte ich hingegen für eine gute Idee. Denn dadurch könnte man diese Leute tatsächlich erreichen. 80.71.142.166 14:42, 26. Jan. 2022 (CET)Beantworten
Unwahrscheinlich. Eine Beratungspflicht würde den Impfvorgang nur für die, die ohnehin wollen, zusätzlich verlängern. Ich war gerade bei der dritten Impfung richtig froh, dass es ohne zusätzliche Brühe zügig ging. Oder ist die Beratungspflicht so gemeint, dass die Imfgegner eine Beratung über sich ergehen lassen müssen, um dann trotzdem »nein« zu sagen? Wenn ich die Impfverweigerer unter meinen Kollegen sehe, die wird das nicht überzeugen. Bakterien, Viren und Imfungen waren mal Schulstoff. Könnte es sein, dass das inzwischen nicht mehr so ist? Viel mehr als dumpfe Ängste vor einer Erkrankung durch die Impfung, die sie eigentlich verhindern sollte, sind mir noch nicht untergekommen. Allerdings hießen die erbinformationspeichernden Stoffe seinerzeit noch Desoxyribonukleinsäure DNS und Ribonukleinsäure RNS ohne das angelsächsiche A, das offenbar doch viele nicht deuten können. –Falk2 (Diskussion) 01:58, 27. Jan. 2022 (CET)Beantworten
Das ist die Frage, was unter „Beratung“ verstanden wird. Eine vom Impfling(m/w/d) zu unterschreibende ärztliche Aufklärung ist bereits Pflicht. Das kann das trilinguale Filmchen (de/en/tr) im Impfzentrum, ein illustrierter Vierseiter oder sonst ein geeignetes Medium sein. Dadurch wird aber auch keine Fehlinformation verhindert oder revidiert, wie prominente Beispiele zeigen. Menschen, die mutwillig gesellschaftszersetzender Propaganda aus Drittstaaten ohne Rechtshilfeabkommen rein aus Fundamentalopposition wider jeder Vernunft folgen, werden so oder so nicht erreicht. --Rôtkæppchen₆₈ 02:43, 27. Jan. 2022 (CET)Beantworten
Schaut Euch mal die von mir verlinkten Studienergebnisse an: Die meisten derzeit (noch) Ungeimpften haben Angst vor der Impfung. Leider spricht die Studienlage (s.o.) eher dagegen, daß man die mit zusätzlichen Fachinformationen überzeugen kann, weil die wohl eher von Gefühlen geleitet werden. Vonseiten der Ärzteschaft gibt es schon vorgreifende Stellungnahmen, daß man sich an Zwangsberatungen nicht beteiligen würde, weil man Patienten nicht vergrätzen will. Vertrauensbildende Maßnahmen erscheinen daher noch am ehesten als erfolgversprechender Ansatz, was vorwiegend im Nahfeld gelingen dürfte - sofern nicht gleichzeitig von außen Ängste geschürt werden. Der Staat wird aktuell zunehmend als Bedrohung wahrgenommen, auch und gerade - aber nicht nur - von den Ungeimpften. Darum frage ich mich ernsthaft, ob es irgendwelche Studien gibt, die nahelegen, daß eine Impfpflicht tatsächlich effektiv zur Bewältigung der Pandemie beitragen könnte, ohne gleichzeitig soziale Verwerfungen zu katalysieren. Wir befinden uns als Gesellschaft aktuell in einer Transformationsphase (Klima/Umwelt, Globalisierung, Digitalisierung usw.), da sind Vertrauen und Zusammenhalt wichtiger denn je. Das Virus wird so oder so bald endemisch werden, vielleicht sogar schon jetzt mit Omikron, darum sollte der Preis, den wir für die zeitnahe Erlangung des endemischen Zustands zu zahlen bereit sind, nicht allzu hoch sein. Autoritarismus, der letztlich nicht dazu führt, daß wir die Pandmie deutlich schneller überwinden, könnte erheblichen Schaden an unserer Demokratie zur Folge haben, weshalb es wirklich gut wäre zu wissen, wie wahrscheinlich es ist, daß eine Impfpflicht den gewünschten Effekt auf das pandemische Geschehen hätte und nicht bloß einen zusätzlichen Streßtest unserer Demokratie darstellen würde. Fest daran zu glauben hilft hierbei nicht weiter, Studienbelege schon eher. --94.219.127.230 02:53, 27. Jan. 2022 (CET)Beantworten
Eine freiwillige Aktion, die dann, wenn man sie nicht freiwillig macht, zur Pflicht wird - den Dreh muss man auch erstmal verstehen. --2003:C0:770F:7E00:84DF:839A:97E0:3C1B 11:44, 27. Jan. 2022 (CET)Beantworten
Ist es nicht immer so, dass es eine Pflicht für ein sinnvolles/notwendiges/etc. Verhalten nur dann gibt, wenn dieses Verhalten nicht auch ohne Pflicht praktiziert wird? Würden alle Autofahrer vor Schulen von sich aus maximal 30 km/h fahren, bräuchte es dort die Anordnung von 30 km/h nicht. Und würde nie jemand etwas stehlen, bräuchte es keine Gesetze, die das verbieten bzw. mit Strafe belegen.--141.30.182.48 15:07, 27. Jan. 2022 (CET)Beantworten
Naja, es ist schon etwas anderes, wenn man sagt: "Die Impfung ist freiwillig und wird auch immer freiwillig bleiben." und dann umschwenkt auf: "Weil sich zu wenige freiwillig haben impfen lassen, gibt es jetzt doch eine Impfpflicht." Sowas kostet Vertrauen. Das soll hier aber - obwohl sehr interessant - nicht Thema sein, weil nicht Teil der an dieser Stelle zu behandelnden Wissensfrage(n). Falls es zu diesem Aspekt weitergehenden Diskussionsbedarf gibt, seid bitte so lieb, diesen im Café auszuleben, damit der Thread hier nicht verwässert. --94.219.127.230 01:27, 28. Jan. 2022 (CET)Beantworten
Hat vielleicht noch irgendjemand weiterführende Hinweise zur Beantwortung der eingangs gestellten Fragen? Es müßte doch zumindest irgendwelche tragfähigen Belege geben, daß eine allgemeine Impfpflicht die Impfquote wesentlich erhöht, wenn sowas zu genau diesem Zweck hier per Gesetz eingeführt werden soll - Stichwort: Geeignetheit. --94.219.4.152 02:08, 31. Jan. 2022 (CET)Beantworten

Groß-/Kleinschreibung

Wörter wie "Genueser Flotte" werden groß geschrieben, oder? Also, dieses "Adjektiv...

Sollte ich recht haben, nur interessehalber: Warum ist das eigentlich so?

--Universal-InteressierterDisk.Arbeit 23:43, 26. Jan. 2022 (CET)Beantworten

Die Adjektive bilden zusammen mit den Substantiven Namen oder Begriffe, die eine eigene, feststehende Bedeutung haben. Die Regeln (D87, 88, 90) dazu sind allerdings nicht ganz eindeutig. --Optimum (Diskussion) 00:08, 27. Jan. 2022 (CET)Beantworten
Doch, sind sie. § 61 der Amtlichen Rechtschreibregeln lautet: Ableitungen von geographischen Eigennamen auf -er schreibt man groß. Eindeutiger geht’s nicht. Von den Duden-Regeln ist hier nur D90 einschlägig. Falls es dem Fragesteller aber um eine inhaltliche, nicht nur um eine formale Begründung ging: Die Formen auf -er leiten sich vom Genitiv der Einwohnerbezeichnung her. Genitive wurden im Mittel- und Frühneuhochdeutschen (und später noch in dichterisch gehobener Sprache) gern vorangestellt: Statt „die Flotte der Genueser“ sagte man „der Genueser Flotte“. Als die Voranstellung des Genitivs in der Alltagssprache außer Gebrauch kam, hielten sich diese erstarrten Formen und wurden schließlich zu Quasi-Adjektiven umgedeutet: „die Genueser Flotte“ analog zu „die genuesische Flotte“. Die echten geographischen Adjektive sind die Formen auf -isch, die ganz normal kleingeschrieben werden. --Jossi (Diskussion) 00:20, 27. Jan. 2022 (CET)Beantworten
Die Frage hatte ich so verstanden, dass es dem TO prinzipell um die Großschreibung von Adjektiven ging und nicht nur um die Ableitungen von geographischen Eigennamen. --Optimum (Diskussion) 01:19, 27. Jan. 2022 (CET)Beantworten
Dann hat er das Beispiel allerdings unglücklich gewählt. Geographische Eigennamen auf -er immer gross (Hamburger, Schweizer, Genueser), wie Jossi schreibt, aber "hamburgische, schweizerische, genuesische" klein. Dann gibt es noch die gerade in der Schweiz gebräuchliche Zusammenschreibung: "Genfersee" etc. Gestumblindi 02:13, 27. Jan. 2022 (CET)Beantworten
Das Wort 'Eigenname' in der Beschreibung ist maßgeblich interessant. Wenn ich sage, dass heute Abend die Schweizer Freunde meines Vaters kommen, ist das zwar kein zusammenhängender Eigenname, aber das Adjektiv endet auch auf -er. Ist es denn nach der amtlichen Rechtschreibung richtig, das Wort "Schweizer" in diesem Fall großsszuschreiben?--Blue 🔯 06:51, 27. Jan. 2022 (CET)Beantworten
Ja, spezielle Regel geht vor allgemeine Regel. Die spezielle Regel lautet: Ableitungen auf -er von geographischen Namen immer groß (also auch „Schweizer“). Deshalb greifen die allgemeinen Regeln zu Eigennamen hier nicht und können außer Betracht bleiben. --Jossi (Diskussion) 11:32, 27. Jan. 2022 (CET)Beantworten
Und wenn nun das ohmsche Gesetz nicht nach Ohm, sondern nach Schweizer benannt wäre, dann wäre es das ... Gesetz? --95.116.49.34 04:36, 28. Jan. 2022 (CET)Beantworten
Das schweizersche Gesetz. Grüße Dumbox (Diskussion) 06:58, 28. Jan. 2022 (CET)Beantworten
Wieso das? Wenn man mal etwas rumblättert, fällt auf, daß bei "XYsches Gesetz" der Name des Namensgeber häufig groß geschrieben wird; auch wp-intern. Wenn es da eine allgemein anerkannte Regel gibt, sollte das mal vereinheitlicht werden. Ich habe sowas bislang immer als ein Fall für die Regeln zu Eigennamen gewertet und durchgehen groß geschrieben. --88.68.87.67 03:54, 2. Feb. 2022 (CET)Beantworten
Vielleicht verdeutlicht es der Unterschied zwischen „schwarzes Loch“ und „Schwarzes Loch“. Ersteres ist irgend ein Loch, das schwarz ist. Letzteres ist ein ganz bestimmtes Gravitationsphänomen. Nun konnte man sagen, dass „genueser Flotte“ eine Flotte ist, die einen Bezug zu Genua hat, und Genueser Flotte eben die eine Flotte, die gewissermaßen ein Begriff für sich ist. Allerdings schreibt man, wie oben schon geschrieben wurde, Ableitungen auf "-er" immer groß. Warum das so ist? Tja, warum ist der Mond rund. Das muss man so lernen. 80.71.142.166 07:42, 27. Jan. 2022 (CET)Beantworten

Danke, Jossi, für die spannende Erklärung. Voranstellung des Genitivs, sollte man vielleicht wieder einführen. Das Schiff kam an in der Hamburger Hafen. Sehr schön. --Skopien (Diskussion) 09:59, 27. Jan. 2022 (CET)Beantworten


Also, wenn irgendwo von einer "genueser Siedlung" geschrieben wird, ist das ein (zu korrigierender?) Rechtschreibfehler. Habe ich das richtig verstanden? --Universal-InteressierterDisk.Arbeit 00:25, 29. Jan. 2022 (CET)Beantworten
siehe »Von Ortsnamen abgeleitete Adjektive«.--Blue 🔯 08:26, 29. Jan. 2022 (CET)Beantworten
Also ist es in Ordnung, wenn ich im Artikel Eroberung von Konstantinopel (1453) alle klein geschriebenen auf Großschreibung ändere? --Universal-InteressierterDisk.Arbeit 11:43, 29. Jan. 2022 (CET)Beantworten
Ach so, noch was zum vorangestellten Genitive (Genitiv): Bei Einzelwörtern, besonders bei Personennamen, ist er noch sehr verbreitet und auch in anderen Fällen kann er durchaus vorkommen (dann freilich eher in gehobener Sprache). Ihr würdet doch auch eher "Lisas Auto" sagen, und nicht "das Auto Lisas" oder "das Auto der Lisa", oder? (nicht signierter Beitrag von Universal-Interessierter (Diskussion | Beiträge) 00:25, 29. Jan. 2022 (CET))Beantworten
Dir fällt aber schon auf, dass der Artikel Eroberung von Konstantinopel (1453) heißt, und nicht Eroberung Kontantinopels? ;-) --Skopien (Diskussion) 13:21, 30. Jan. 2022 (CET)Beantworten
Jedoch ist „das Auto von Lisa“ durchaus verbreitet, und besser als „Lisa ihr Auto“. Man sieht daran übrigens, dass der korrekte Genitiv eigentlich viel kürzer und bequemer ist als die umgangssprachlichen und unbeholfen klingenden Varianten mit Präposition. --2003:D0:2F18:5711:D904:6192:3C9B:4B82 16:12, 29. Jan. 2022 (CET)Beantworten
"Besser" halte ich für die falsche Kategorie. Die 3 Varianten sind Ausdruck verschiedener regionaler Varianten der deutschen Sprache. Im Bairischen zum Beispiel ist der Genitiv weitgehend ausgestorben, da scheint 'die Kinder von die Nachbarn' oder eben 'das Auto von Lisa' die völlig korrekte Form. Und in anderen Gegenden halt 'der Lisa ihr Auto'. Logisch wäre das im Hochdeutschen falsch, aber im Dialekt nicht. --Skopien (Diskussion) 13:13, 30. Jan. 2022 (CET)Beantworten
Wo Genueser, wo genuesisch?    Hallo @Universal-Interessierter, deine Frage ist gar nicht so leicht, wie man ich das anfangs annahm.    Zum Beitragsende↓
Es gibt da all diese speziellen Regeln und Ausnahmen bei der Groß- und Kleinschreibung, bspw., wenn Bindestriche dazu kommen (im Duden online:  D22 – „eine Cloud-basierte Lösung“, aber „eine cloudbasierte Lösung“;  D68 – „Moskau-freundlich“) oder wenn einzelne Ausdrücke eigenen Bestimmungen folgen („iPhone“, „mRNA“).
Je spezieller die Ausdrücke sind, desto spezieller werden die Regeln. Vielen Dank an @Jossi2 für die beiden Beiträge (00:20 und 11:32, 27. Jan.) zu dem Thema! Bisher war ich davon ausgegangen, dass die beobachteten Unterschiede zwischen den Endungen „-(i)sch“ und „-er“ bei geographischen Bezügen darauf zurückgehen, dass die Endung „-er“ sehr häufig bei Eigennamen angewendet wird, während das bei „-(i)sch“ seltener der Fall ist.
Im Duden steht zwar auch etwas erklärender Text (online: »Von Ortsnamen abgeleitete Adjektive auf „-(i)sch“ und „-er“«), aber kaum etwas darüber, in welchen Fällen man „-(i)sch“ oder „-er“ einsetzen sollte, wenn beide Formen zur Verfügung stehen.
Einige Ausdrücke auf der Duden-Seite [9] wirken auf mich konstruiert:
  • „rostockerische Spezialität“, „ein berlinischer Ausdruck“ und „ein typisch berlinerisches Wort“.
Bei anderen Beispielen fällt auf, dass es Eigennamen sind:
  • „eine Berliner Weiße“, „die Berliner Mauer“
und andere wirken wie Eigennamen, z. B. dieses:
  • „die Berliner Flughäfen“.
Es erscheint auf den ersten Blick auch nicht besonders schlüssig, dass der gleiche Bedeutungsinhalt mal groß- und mal kleingeschrieben wird:
  • „die Berliner Mundart“ bzw. „die berlinische Mundart“.
Na ja, ist eben so; das eine ist üblich, das andere wissenschaftlich und „berlinerisch“ gibt es auch noch.
Könnte es sein, dass es nicht nur bei der Groß- und Kleinschreibung von „-(i)sch“ oder „-er“ Regeln gibt, sondern auch einige weniger verbindliche Konventionen hinsichtlich der Verwendung in Eigennamen? Das hieße dann z. B., dass man beim Kreieren eines Namens mit geographischem Bezug unter bestimmten Bedingungen bevorzugt die Endung „-er“ anhängen würde, ansonsten aber eher die Endung „-(i)sch“ zum Einsatz käme, wenn das Adjektiv kein Namensbestand werden soll. Eine solche Konvention träfe nicht überall zu, denn viele Artnamen enden bspw. auf „-(i)sch“ o. ä. (z. B. der Sibirische Tiger).
Bei deiner Frage mit der Genueser Flotte und den anderen Wortgruppen mit einem Bezug zu Genua scheint es so zu sein,
  • dass zum einen Eigennamen auftreten und
  • dass zum anderen Adjektive verwendet werden, ohne dass sie zusammen mit dem jeweiligen Substantiv einen Namen bilden würden.
Du hast nachgefragt, ob man das kleine g in „genueser Siedlung“ in einem Text, in dem „irgendwo von einer "genueser Siedlung" geschrieben“ wurde, ganz generell korrigieren sollte, indem ein großes G zum Einsatz käme.
Der Regel entsprechend hast du dann alle Zeichenketten, die im Artikel „genueser“ lauteten, in „Genueser“ umgewandelt (diff=219663201&oldid=219560001). An keiner der korrigierten Stellen betrifft das zuvor kleingeschriebene „genueser“ einen Eigennamen.
Deshalb habe mich gefragt, ob man an einigen der korrigierten Stellen stattdessen vielleicht auch „genuesisch“ schreiben könnte bzw. sollte. Das weiß ich aber nicht. Es sei angemerkt, dass woanders beides, „Genueser“ und „genuesisch“ vorkommen (z. B. unter Genueser Kolonien#Ägäis: „... der Genueser Patrizierfamilie ...“ und weiter: „... fast sämtliche genuesischen Kolonien ...“).
Ich hoffe nicht, dass es auch noch /[G|g]enueserisch/ o. ä. gibt.    (‑;  Das wäre ja noch genuesischer!  ;‑)    Zum Beitragsanfang↑
MfG --Dirk123456 (Diskussion) 00:15, 2. Feb. 2022 (CET)Beantworten

27. Januar

Wo ist die Buchfunktion geblieben

Ich habe früher häufiger die Buchfunktion benutzt. gibt es sie noch und wenn ja, wo?

--2003:C4:BF25:2166:AD98:D6F:6AEB:CCFD 09:17, 27. Jan. 2022 (CET)Beantworten

https://de.wikipedia.org/wiki/Hilfe:Buchfunktion --87.147.191.130 10:33, 27. Jan. 2022 (CET)Beantworten

Die Hilfe ist nicht hilfreich, denn die Funktion ist verschwunden. --31.213.219.157 11:55, 27. Jan. 2022 (CET)Beantworten
Genau das steht doch dort (begründet) geschrieben? --87.147.191.130 12:16, 27. Jan. 2022 (CET)Beantworten
Psst, Geheimtipp, nicht weitersagen! Schnappt euch ein Buch und übernehmt es. Aber dafür muss man, glaube ich, angemeldet sein. --MannMaus (Diskussion) 16:21, 30. Jan. 2022 (CET)Beantworten

Präsidenten des Thüringer Landesamt für Bau und Verkehr

Guten Tag, ich brauch die ehemaligen Präsidenten des Thüringer Landesamt für Bau und Verkehr.


--195.191.14.89 14:50, 27. Jan. 2022 (CET)Beantworten

Vor dem aktuellen gab es offenbar nur Markus Brämer (ab 2008 bis Ende 2018). --Magnus (Diskussion) 15:03, 27. Jan. 2022 (CET)Beantworten
Das ist korrekt. Markus Brämer war der "Gründungspräsident" des Thüringer Landesamtes für Bau und Verkehr, das 2008 aus der Zusammenlegung des Landesamtes für Straßenbau und dem Staatlichen Hochbauamt hervorging. --LinusS (Diskussion) 17:27, 30. Jan. 2022 (CET)Beantworten

Sozialer Wohnungsbau

Im Artikel Sozialer Wohnungsbau steht: Bedingt durch die Laufzeit der Verträge fallen die Bestände des sozialen Wohnungsbaus nach einigen Jahrzehnten dem allgemeinen, durch die Mietengesetzgebung regulierten Markt zu. Dadurch verringert sich der Bestand, seitdem der neu hinzukommende soziale Wohnungsbau deutlich verringert wurde. - Warum wurde das in Deutschland so angelegt? Aus meiner Sicht wäre das österreichische Modell mit dem Gemeindebau, die dann für "immer" ein sozialer Wohnungsbau bleiben sinnvoller. So fördert der Staat mit Steuern Bauwerke, die dann nach Ablauf der Verträge doch wieder teuer vermietet werden können. So kriegt man doch die Wohnungsnot nie in den Griff. --sk (Diskussion) 17:40, 27. Jan. 2022 (CET)Beantworten

In Deutschland gibt es auch kommunalen Wohnungsbau. "Anfang der 1980er Jahre schwand der politische und gesellschaftliche Konsens zum kommunalen Wohnungsbau" (s. Abschnitt Privatisierung kommunaler Wohnungsunternehmen in Deutschland nach 1988). In Sozialer Wohnungsbau in Berlin ist auch was zur Entwicklung geschrieben. 62.157.2.126 18:22, 27. Jan. 2022 (CET)Beantworten
Trotzdem ist die grundsätzliche Frage berechtigt. Ich frage mich auch schon lange, warum man die öffentliche Förderung nicht strikt auf gemeinnützige Bauträger beschränkt, statt in the long run private Investoren zu subventionieren. --Jossi (Diskussion) 18:52, 27. Jan. 2022 (CET)Beantworten
Bauen kann nur, wer Grundstücke besitzt. Die privaten Investoren haben die Grundstücke. Durch Zuschüsse und Steuervorteile sind sie bereit, einen gewissen Anteil an günstigem Wohnraum zu schaffen. Ohne die Förderung würde es noch weniger Sozialwohnungen geben. Stattdessen würden besser ausgestattete Wohnungen gebaut, die höhere Mieten bringen. Außerdem lassen sich soziale Brennpunkte ein wenig vermeiden, weil unterschiedliche Bevölkerungsschichten im gleichen Haus wohnen und die "Schwachen" nicht in "Ghettos" konzentriert werden.--Vertigo Man-iac (Diskussion) 19:05, 27. Jan. 2022 (CET)Beantworten
So mag das in der Theorie aussehen, funktioniert aber nur sehr eingeschränkt in der Praxis. Seltsam auch, daß das genau so läuft, wenn es um Grundstücke geht, die (zunächst) dem Staat gehören. Letztlich macht man denjenigen, die bauen, ein Geschenk. Durch die zeitliche Verzögerung fällt nicht auf, daß man damit letztlich Steuergelder und staatlichen Besitz an die Privatwirtschaft verscherbelt. Ich vermute, das gehört mit in die Kategorie Privatisierungswahn, der uns in den letzten Jahrzehnten massenhaft Infrastruktur gekostet und nach der Wende den Osten versaut hat. Ganz besonders seltsam ist, daß eine Stadt wie bspw. Wien damit anscheinend überhaupt kein Problem hat: Insgesamt gibt es in Wien 890.000 Wohnungen. 60 Prozent der Wiener Bevölkerung lebt in einer geförderten oder Gemeindewohnung. Denn zusätzlich zu den kommunalen Wohnungen zählt Wien weitere rund 200.000 dauerhaft sozial gebundene Wohneinheiten von Gemeinnützigen Wohnbauvereinigungen zum Bestand. Diese wurden, und werden nach wie vor, mit Mitteln aus der Wohnbauförderung errichtet und unterliegen bis zur „Ausfinanzierung“ nach meist 35 Jahren den Bestimmungen des Wohnbauförderungsgesetzes, das eine Kostenmiete vorschreibt, also abgesehen von einer geringen Eigenkapitalverzinsung keine Rendite zulässt. Anschließend fallen diese Wohnungen aber nicht aus der sozialen Bindung, sondern unter das Wohnungsgemeinnützigkeitsgesetz, das erneut leistbare Mieten vorsieht. Sie sind damit dauerhaft sozial gebunden. --94.219.127.230 00:28, 28. Jan. 2022 (CET)Beantworten
Das zeigt aber wunderbar das Problem auf: Langfristig ist es grundsätzlich auch für Kommunen attraktiv, Wohnraum zu vermieten. Durch den Wertzuwachs der Immobilien steigert das auch den Wohlstand der Kommune. Die Entscheidungsträger haben aber selten Lust, sich mit den Mieteinnahmen zu begnügen. Da wird dann privatisiert und Kasse gemacht, weil man das Geld "jetzt" und nicht in den nächsten 20 Jahren braucht. Dazu kommt, dass der Staat meistens ein sehr schlechter Unternehmer ist. Gewinnmaximierung führt auch zu Innovation und Wettbewerb, der Staat verwaltet meist nur. Im Ergebnis ist das selten besser, aber oft kostspieliger.--Vertigo Man-iac (Diskussion) 16:58, 28. Jan. 2022 (CET)Beantworten
Oh ja. Allerschlimmstes mir bekanntes Beispiel hierfür ist Berlin, das vor einigen Jahren mal eben massenhaft Wohnungen verschleudert hat, um die Stadtkasse aufzufüllen, und kurze Zeit später für einen Wahnsinnsbetrag einen kleinen Teil dieser Wohnungen zurück gekauft hat - und zwar dummerweise genau den Teil, der annähernd wertlos, weil komplett verrottet ist. Das kommt davon, wenn man seine Mitarbeiter vorwiegend nach ihrer Systemkompatibilität aussucht, was zumindest im deutschen Staatswesen enscheinend oftmals bedeutet, daß man sich gezielt irgendwelche Idioten aussucht, die möglichst wenig darüber nachdenken, ob das, was sie tun, tatsächlich zielführend ist. --84.58.127.36 03:34, 29. Jan. 2022 (CET)Beantworten
Eine Kommune finanziert sich neben der Gewerbe- und Grundsteuer zu einem beträchtlichen Teil aus der Einkommensteuerumlage (leider gibt es hierzu keinen Wiki-Artikel). Eine Kommune (bzw. deren Entscheidungsträger) hat deshalb Interesse am Zuzug oder Verbleib von möglichst gut verdienenden Personen und wenig Interesse an Sozialwohnungen.--2A01:C22:7749:7100:2861:79B1:AFD7:E9E0 10:10, 28. Jan. 2022 (CET)Beantworten
Öhm, muß eine Kommune nicht auch Sozialleistungen finanzieren und sollte daher an möglichst günstigem Wohnraum für Geringverdiener interessiert sein? Und müßten Unternehmen, die sich dort ansiedeln, nicht daran interessiert sein, ihren Mitarbeitern möglichst wenig Gehalt zu zahlen, was bei hohen Mieten sehr schwierig wird, weshalb man dort dann eher nicht investiert und infolgedessen auch keine Gewerbe- und Grundsteuer zahlt? --84.58.127.36 03:34, 29. Jan. 2022 (CET)Beantworten
Nein, Kommunen wollen keine Geringverdiener (Stichwort „Einkommensteuerumlage“) und auch keine Unternehmen im Niedriglohnsektor. Unternehmen im Hochlohnsektor bringen neben höheren Löhnen und somit höherer Einkommensteuerumlage vermutlich auch mehr Gewerbesteuer. Wenn eine Kommune nicht die Wünsche ihrer Wählerschicht (in einer teuren Gegend mehrheitlich keine Geringverdiener) finanzieren kann, werden bei der nächsten Kommunalwahl halt andere ans Ruder gestellt. So ergibt sich ganz automatisch, dass Kommunen in teuren Wohngegenden – trotz aller Lippenbekenntnisse - wenig bis keine Sozialwohnungen errichten wollen.--2A01:C23:BDEA:3000:462:7D14:1F8A:396D 17:31, 29. Jan. 2022 (CET)Beantworten
Wie soll das gehen, wenn wir gleichzeitig den Niedriglohnsektor fördern? Die Menschen verschwinden ja nicht einfach, wenn die Mieten steigen, sondern fallen dann den sozialen Sicherungssystemen zur Last. Oder kann man Bürger neuerdings outsourcen, wenn deren Performance nicht den Erwartungen entspricht? Wären Arbeitslager eine gangbare Alternative zum Sozialwohnungsbau, damit die Mittel- und Unterschichtler dann nicht alle obdach- und beschäftigungslos auf den Straßen herumlungern? --84.58.127.36 19:54, 29. Jan. 2022 (CET)Beantworten
Die Menschen verschwinden natürlich nicht, wenn in ihrer angestammten Heimat ihr Einkommen nicht mehr für die Lebenshaltungskosten reicht. Sie müssen dann halt dahin ziehen, wo diese Faktoren besser im Gleichgewicht sind, es sei denn, sie sind mit den nicht gerade üppigen Sozialleistungen zufrieden. (Nicht dass ich das richtig finde, aber das ist die Realität.). Natürlich kann man Bürger auch outsourcen. Das ist gängige Praxis, bspw. in Schlachtbetrieben (Tönnies Holding). Viele kommen aus Billiglohnländern und leben in Unterkünften, gegen die eine Sozialwohnung der reine Luxus ist. Vermutlich werden sie später auch wieder in ihre Heimat zurückkehren oder sind nie endgültig von dort weggezogen. Ähnlich ist es bspw. bei Erntehelfern (Spargel) oder den Subunternehmern Im Tief- und Hochbau. --2A01:C23:C5EA:3C00:95B4:22E4:9F97:F066 09:40, 30. Jan. 2022 (CET)Beantworten
Öhm, hier geht es nicht um EU-Ausländer, die bei uns besser verdienen als zu Hause, sondern um Inländer, die sich die Mieten in Deutschland nicht mal in Vollzeitbeschäftigung leisten können unter den aktuellen Bedingungen. Wo sollen die denn bitte hinziehen? Und wer arbeitet dann in den Städten und Gemeinden, wenn dort massenhaft Menschen wegziehen, weil ihr Einkommen nicht mehr für die Lebenshaltungskosten reicht? Und seit wann bitte ist die Tönnies Holding ein Staat mit eigenen Bürgern? Habe ich da was verpaßt? --94.219.4.152 02:19, 31. Jan. 2022 (CET)Beantworten
Naja, man müsste mal schauen, ob der Artikel Sozialer Wohnungsbau auch auf einen Artikel Sozialer Wohnungsbau in Deutschland verweist und wie sich der Bestand der Anzahl der Wohnungen entwickelt, die unter Wohnungsbindung stehen.
Hier ist erst einmal eine bestimmte Dauer ab der Errichtung zu beachten. Kommt es aber zu Renovierungen von Dächern, Heizungsanlage, Fenstern usw. unter neuerlicher Inanspruchnahme von öffentlichen Darlehen oder Zuschüssen, entsteht wiederum eine Bindung für gewisse Dauer.
Nach meiner Erfahrung waren zwischenzeitlich die Mieten im sozialen Wohnungsbau sogar höher als auf dem freien Markt.
Die Fehlbelegungsabgabe führte dazu, dass im sozialen Wohnungsbau erwerbslose Mieter verblieben und die Verdiener verschwanden... soziale Ghettoisierung war die Folge.
Der freie Markt allein führte aber nicht zu einer Abhilfe von der Wohnungsnot im 20. Jahrhundert. Offensichtlich war es attratktiver, alte Kaschemmen überteuert zu vermieten anstatt neuen Wohnraum zu schaffen. Beim Thema Wohnraum hat der Staat mehr Verantwortung als bezüglich des Angebots im Bereich des normalen Konsums. --BanditoX (Diskussion) 11:03, 28. Jan. 2022 (CET)Beantworten
Zu vermieten ist gar nicht mehr notwendig. Man kauft einfach irgendeine Imobilie und wartet dann ab, wie deren fiktiver Wert mit dem stetigen Wachstum der Immobilienblase immer weiter zunimmt. Mieter stören dabei eher noch, weil die ständig irgendetwas wollen, anstatt einfach nur die vom Markt bestimmten Quadratmeterpreise abzudrücken. Das Leben als Immobillienbesitzer ist um vieles einfacher, wenn man sich damit begnügt, die Preisentwicklung abzusitzen. Das geht voll easy, solange steigende Mieten und steigende (fiktive) Verkaufswerte sich gegenseitig in eine Teuerungsspirale treiben, wie man derzeit bspw. besonders schön in London sehen kann. Der Staat guckt dabei schulterzuckend zu, weil er weiß, daß man den Markt nicht stören darf in seinem Wirken, weil die Lobbyisten sagen, daß jeglicher Eingriff in das freie Spiel der Kräfte des Marktes fürchterliches Unglück bringen würde und weil die Unternehmen, für die die Lobbyisten arbeiten, stinkreich sind, müssen die es ja wohl am besten wissen. --84.58.127.36 03:34, 29. Jan. 2022 (CET)Beantworten
In den 1980ern schwenkten so gut wie alle politischen Parteien auf klassisch-marktliberale Positionen. Der Markt wurde als Allheilmittel für jegliches Problem angesehen. Die Kommunen verkauften nach und nach ihre Wohnungsgesellschaften und konnten mit dem Verkaufserlös das ein oder andere Prestigeprojekt realisieren. Damit kam auch eine schleichende Veränderung der Marktstrukturen. Ein 30-40% grosser Block von Sozialwohnungen wurde nun von privaten Unternehmen betrieben, die aber kein Interesse hatten, neue Wohnungen als Sozialwohnungen anzulegen. Damit schwand dieser Sockel an billigem Wohnraum, wodurch der Druck auf den Wohnraum stieg und die Mieten gerade im Billigsegment erhöht werden konnten. Die Mietmisere von heute nahm ihren Anfang.
Es stört den Markt weniger, auf der Anbieterseite ein Billigangebot zu platzieren, als die Umsätze, Preisentwicklung oder ähnliches zu reglementieren. Umgekehrt kann der Staat ja auch bei zu niedrigen Preisen als Käufer auftreten und den Markt damit stützen (so geschehen beim Kauf der Hypo-, der Commerzbank usw.). Und weil die Politiker von damals längst im Sarg liegen oder dement vor sich hin sabbern, haben ihre Nachfolger freie Hand den nächsten kurzsichtigen Unfug auszuhecken. Yotwen (Diskussion) 17:11, 28. Jan. 2022 (CET)Beantworten
Gibt es irgendwo Statistiken darüber, wie viele Wohnungen in den Jahren jeweils gebunden waren?--BanditoX (Diskussion) 19:37, 28. Jan. 2022 (CET)Beantworten
Das Statistische Bundesamt hat eine Abteilung Wohnen in Deutschland. Yotwen (Diskussion) 19:49, 28. Jan. 2022 (CET)Beantworten

ISIN für Aktien welche nicht an Börsen gehandelt werden

Warum haben Aktien die nicht an der Börse gehandelt werden eine ISIN? Ausser wenn sie kurz vor dem Börsengang sehe ich keinen Vorteil.--Sanandros (Diskussion) 23:33, 27. Jan. 2022 (CET)Beantworten

Es gibt diverse regulatorische Meldepflichten für Kreditinstitute und andere Unternehmen, die Aktien halten. Da sind dann unter anderem auch nicht-börsennotierte Aktien an Bundesbank etc. zu melden, teilweise auf Einzeltitel-Ebene. Dafür (und für die Auswertung bei der Bundesbank etc.) ist eine ISIN nicht zwingend erforderlich, aber als eindeutiger Schlüssel vereinfacht es die Sache und ist daher durchaus vorteilhaft. (Ob das ein Vorteil für die Aktiengesellschaft selbst ist, mag da hingestellt sein.) --77.87.228.67 10:12, 28. Jan. 2022 (CET)Beantworten
erstens gibt es nicht "die Börse" und ausserdem gibt es auch Wertpapierhandelshäuser wie z.B. L&S --193.56.252.188 10:27, 28. Jan. 2022 (CET)Beantworten
Und warum hat dann Heckler und Koch eine ISIN? Ihre Aktioen werden ja nicht gehandelt.--Sanandros (Diskussion) 22:30, 29. Jan. 2022 (CET)Beantworten
Siehe unsere Artikel ISIN, Identifikation, Authentifizierung und Identifikator. --Rôtkæppchen₆₈ 12:28, 30. Jan. 2022 (CET)Beantworten

28. Januar

Blitzlichtfotogeräusche

Eine Frage an die, die kürzlich persönlich bei Fototerminen waren:

Was ist das vorherrschende "instrumentelle" Geräusch, wenn ca. 10-20 Fotografen gleichzeitig mit Blitzlicht fotografieren? Früher gab es ein knallendes Geräusch, wenn die Blitzbirne ausbrannte. Sind heutige Blitze "geräuschlos" ? D.h. man würde nur das Klicken der Spiegelreflexe hören. Wahr oder fasch? Papa tanzt Tango (Diskussion) 13:39, 28. Jan. 2022 (CET)Beantworten
Die Geräuscher der Blitzlampen nannte man Blitzlichtgewitter (wobei WP dazu etwas anderes weiss). Auch die heute geräuschlose aber optisch nervige Blitzerei von Paparazzis nennt man umgangssprachlich noch so. Gruss, --Markus (Diskussion) 14:03, 28. Jan. 2022 (CET)Beantworten
Auch Wetterleuchten (allerdings sichtbare Blitze ohne Knall).--Wikiseidank (Diskussion) 15:34, 28. Jan. 2022 (CET)Beantworten
Es gibt bzw. gab drei Arten von Blitzlampen: 1. Glaskolben mit einer Füllung aus Metallwolle - vermutlich Mg - und Sauerstoff; die wurden elektrisch gezündet und verbrannten dann chemisch. An besonders laute Geräusche ("Knall") kann ich mich nicht erinnern, aber ich hatte auch nie die "dicken Ladungen" mit den hohen Leitzahlen in Händen, die von Reportern vermutlich verwendet wurden, um große Räume und weitere Entfernungen auszuleuchten. (Die armen Kerle mußten damals die Beleuchtungsverhältnisse beim Blitzlicht einigermaßen genau schätzen und die Blendenöffnung entsprechend einstellen, sonst war die Aufnahme nicht richtig belichtet.) Der Vorteil der Dinger war, daß sie recht zuverlässig waren und für ihre Lichtstärke wenig Platz und Gewicht in Anspruch nahmen, der Nachteil, daß diese Blitzbirnen ziemlich teuer waren und nach jeder Aufnahme ausgewechselt werden mußten, wobei die abgebrannten Birnchen zunächst ziemlich heiß waren; es gab auch Vorrichtungen mit mehreren Blitzbirnen, die relativ unkompliziert nacheinander gezündet werden konnten und so Serienblitzaufnahmen ermöglichten. 2. "Elektronenblitze" mit Gasentladungslampen. Die verbrauchten ziemlich viel Strom und benötigten deswegen mehr oder weniger gewichtige Batteriepakete, außerdem mußte die Elektronik vor jeder Blitzauslösung einen Ladekondensator auf eine hohe Spannung aufladen. Später gab es die auch als "Computerblitze", die die Ausleuchtung durch mehr oder weniger intensive Ausleuchtung automatisch regulierten, teilweise sogar mittels Lichtmengenmessung durch das Aufnahmeobjektiv. Geräuschlos war deren Blitzabgabe auch nicht, aber akustisch doch sehr dezent. 3. Moderne Blitze erzeugen das Licht wohl mittels LED und sind an fast jedem Smartphone zu finden. Die sind geräuschlos, aber in den "dickeren" Leistungsklassen verstehe ich von denen nichts. - Die wesentliche Geräuschentwicklung beim Blitzlichtgewitter dürfte aber nicht von der Funktion der Blitzlichter, sondern vom motorischen Filmtransport und Verschlußspannen bei den Serienaufnahmen sowie den Auslösegeräuschen hergerührt haben - da macht schon eine einzelne Kamera ordentlich Krach, und erst recht das Konzert eines versammelten Pressecorps. --95.116.49.34 22:28, 28. Jan. 2022 (CET)Beantworten
Ergänzend dazu: In den Blitzbirnen und -würfeln war Magnesiumdraht oder -folie drin. In den Gasentladungslampen ist Xenon drin, das ein grell weißes Licht abgibt. Vor den Blitzbirnen und -würfeln gab es pyrotechnisches Blitzlichtpulver ebenfalls auf Magnesiumbasis. Das qualmte und stank dann. --Rôtkæppchen₆₈ 02:47, 29. Jan. 2022 (CET)Beantworten
LEDs beschränken sich auf Schlaufone und vielleicht einfache Kompaktkameras. Im professionellen Bereich wird auch heute noch mit Gasentladungsröhren geblitzt, einfach weil diese Technik höhere Leuchtdichten ermöglicht und konstruktiv / operativ mittlerweile perfekt beherrscht wird. Zum Beispiel strahlt so eine Röhre allseitig ab, das Licht kann per Reflektor und Kollimatoroptik beliebig gebündelt werden. --Kreuzschnabel 09:30, 29. Jan. 2022 (CET)Beantworten
Das darf man auch ungegugelt so verstehen, daß es Fotoblitzlichtgeräte mit hohen Leitzahlen nicht gibt? (Was mich schon etwas verwundert.) --95.116.29.20 11:30, 31. Jan. 2022 (CET)Beantworten

Wieviel Mannjahre stecken in der Erstellung der deutschsprachigen Wikipedia?

Das würde mich mal interessieren, wieviel Arbeitszeit brutto in diesem Projekt stecken. Man muss dazu sicherlich auch überlegen, wieviel Zeit eigentlich in einem edit stecken, das wiederum könnte man vielleicht noch grob abschätzen anhand der Byte-Menge, die man ja irgendwie tippen, oder kopieren und einfügen muss... Oder anhand von Sessions, wo man erkennen kann, dass hier jemand gerade mal nur mal die Zeit hatte, zum Klo zu gehen zwischen den Bearbeitungen. --BanditoX (Diskussion) 14:38, 28. Jan. 2022 (CET)Beantworten

Das hängt erheblich u.a. davon ab, wie viel Arbeit ein Autor sich macht, d.h. wie viel Recherchezeit mit jedem Edit verbunden sind. Das kann extrem schwanken. --Stilfehler (Diskussion) 14:46, 28. Jan. 2022 (CET)Beantworten
Wie kommst du nur von deinem gerade neu erstellten Artikel Verlorene Lebensjahre auf Personenjahre (Mannjahre) bei der Erstellung der de-Wikipedia? ein SmileysymbolVorlage:Smiley/Wartung/8p . --Pp.paul.4 (Diskussion) 14:48, 28. Jan. 2022 (CET)Beantworten
ein SmileysymbolVorlage:Smiley/Wartung/pfeif  --BanditoX (Diskussion) 23:59, 28. Jan. 2022 (CET)Beantworten
Schwer zu sagen, da Du den Anteil der Fraujahre noch rausrechnen mußt.💅 (nicht signierter Beitrag von 89.204.153.195 (Diskussion) 14:50, 28. Jan. 2022 (CET))Beantworten
Ich schätze, dass im derzeitigen Artikelbestand etwa 800 bis 1200 Arbeitsjahre Schreibaufwand stecken (ohne Recherche- und Diskussionsaufwand). --2003:CD:3F17:6ACE:A50A:92EC:DB3D:CE33 14:51, 28. Jan. 2022 (CET)Beantworten
Darf man fragen, auf welcher Grundlage diese deine Schätzung beruht? --Pp.paul.4 (Diskussion) 02:38, 29. Jan. 2022 (CET)Beantworten
"Mannjahr"? Ist danach gefragt?--Wikiseidank (Diskussion) 15:37, 28. Jan. 2022 (CET)Beantworten
1 Mannjahr = 200 Arbeitstage * 8 h ~1600 h
1200*1600 ~ 2 Mio. Arbeitsstunden nach dieser Schätzung. Erscheint mir bei über 2 Mio. Artikeln arg wenig. Yotwen (Diskussion) 16:52, 28. Jan. 2022 (CET)Beantworten
Laut Vorlage:Status sind es 2,6 Millionen Artikel, aber die Byte-Anzahl und den Mittelwert kenn ich gerade nicht. Wenn man mal einen Durchschnitt von 3 Stunden pro Artikel nehme würde... käme man auf etwa 5000 Mannjahre, also 250 Menschen mal 20 Jahre in Vollzeit. Bei einem Stundensatz von 12,50 Euro wären es 100 Millionen Euro Lohn, die Sozialversicherung, Steuer, die Werkhalle, den Werkarzt und den Betriebspsychologen mal nicht gerechnet. -- BanditoX (Diskussion) 00:06, 29. Jan. 2022 (CET)Beantworten
Du vergisst die Zeit, die auf Meta verballert wird. Je 3 h Artikelarbeit kommt da mindestens noch einmal so viel für LD, VM und sonstige Vergnügungen drauf. Yotwen (Diskussion) 07:48, 29. Jan. 2022 (CET)Beantworten
Von 7.349.094 Seiten sind 2.658.702 Artikel und der Rest nicht. Das ist viel Platz für Gequatsche. Ich kenne keine Analyse, wie viele Benutzerseiten, Benutzerdiskussionsseiten, Artikeldiskussionseiten usw. leer, mit Bot-Hinweisen gefüllt oder mit Kommentaren versehen es gibt, und auf wie vielen echten Meta-Seiten vorne und hinten man sich so tummelt. Wieviele Mannjahre stecken wohl allein in der Seite WP:Auskunft? -BanditoX (Diskussion) 15:17, 29. Jan. 2022 (CET)Beantworten
Lass dir von einem der Wizz-Kids ermitteln, wie viele Änderungen der Seite „Auskunft“ erfolgten. Für ein Edit braucht man im Mittel ca. 5 Minuten. Das Ding kann in 2 Minuten gelesen werden und wird ca. 50 x gelesen. (Oder du passt die Zahlen deinen Vorstellungen an). Dann rechnest du ein wenig und addierst die "Edit-Zeiten" und die "Lese-Zeiten" und du hast eine ungefähre Abschätzung. Es würde mich nicht wundern, wenn du deutlich höher herauskämst, als du es dir ohne Schätzung hättest träumen lassen. Yotwen (Diskussion) 16:50, 29. Jan. 2022 (CET)Beantworten
Wo finde ich die Wizz-Kids? --BanditoX (Diskussion) 22:51, 31. Jan. 2022 (CET)Beantworten
Du musst nicht jeden Tippfehler wiederholen: "Whizz-Kids" von Whizzard = Zauberer - Das sind die Leute, die in der Datenbank wühlen können, wie ich in meiner Sockenschublade.
Wenn ich die suche, dann mache ich das so: Würde bitte jemand in der Datenbank rausfinden, wie oft die Seite "WP:Auskunft" schon editiert wurde? Danke im Voraus. Yotwen (Diskussion) 09:02, 1. Feb. 2022 (CET) Und weil sie "Whizzards" sind, müssen sie das nicht mal lesen. Sie wissen schon vorher, wann ich frage. Cool, nicht?Beantworten
Bitte unbedingt den Unterschied zwischen Wiz und Whiz memorieren. --Rôtkæppchen₆₈ 09:09, 1. Feb. 2022 (CET)Beantworten
Für die Antwort nach den Edits braucht man kein Whizz-Kid zu sein: 306.139, leicht der Seiteninformation zu entnehmen (zu finden in den Werkzeugen links in der Desktop-Ansicht).--Naronnas (Diskussion) 09:12, 1. Feb. 2022 (CET)Beantworten
Gesamtzahl der Bearbeitungen 306.140 [10]. Wurde dieser Seite eventuell auch mal die ältere Versionshistorie entzogen? Falls ja, wohin?
Im übrigen aber rechne ich jetzt mal los für die Auskunft mit Bearbeitungszahl * 2 Minuten / 60 Minuten / 1600 = 6,4 Mannjahre. --BanditoX (Diskussion) 09:31, 1. Feb. 2022 (CET)Beantworten
Ich glaube, du musst die Lesezeiten auch berücksichtigen. Eine Frage muss ggf. mehrfach gelesen werden, bevor du einen "positive hit" erzielst: Jemanden, der die Frage beantworten kann/will. Yotwen (Diskussion) 11:00, 1. Feb. 2022 (CET) PS: Und für mich bist du schon ein Whizz-Kid, wenn du weisst, dass es die Seiteninformation gibt, Kollege Naronnas.Beantworten

29. Januar

Wann änderte cassius clay seinen namen?

--2001:9E8:2514:7500:7537:6AD7:59B6:EC5A 13:48, 29. Jan. 2022 (CET)Beantworten

vor 1975 schätze ich. Siehe Abschnitt "Leben" in unserem Artikel "Muhammad Ali" gruß --2003:C6:1718:FFFD:6CFD:E833:ABD7:1AFF 13:58, 29. Jan. 2022 (CET)Beantworten
Einfach nachlesen: Muhammad Ali, da steht’s. --Kreuzschnabel 13:59, 29. Jan. 2022 (CET)Beantworten
Nein, stimmt nicht. Da steht zwar was zur Namensänderung ohne Datum, die verlinkte Quelle nennt aber den neuen Namen nicht. --Hachinger62 (Diskussion) 17:29, 29. Jan. 2022 (CET)Beantworten
Das Jahr kann man dem Artikel entnehmen, einfach nach "Name" im Text suchen. Funktioniert auch im englischen Artikel. 62.157.2.126 18:44, 29. Jan. 2022 (CET)Beantworten
Doch, stimmt. Da steht Im Februar 1964 bekam Clay einen Weltmeisterschaftskampf gegen Sonny Liston […] Im weiteren Verlauf des Jahres machte Clay seine Mitgliedschaft in der Nation of Islam öffentlich und wählte Muhammad Ali als Namen. --Kreuzschnabel 23:33, 30. Jan. 2022 (CET)Beantworten
Im Mai 1965 (vor dem Liston Rückkampf) nannten ihn manche Zeitungen Clay und andere Ali.--Wikiseidank (Diskussion) 19:27, 29. Jan. 2022 (CET)Beantworten
...On 25 February 1964, he defeated Sonny Liston to become heavyweight champion of the world. On 27 February, he announced that he had joined the Nation of Islam and, on 6 March, he changed his name to Muhammad Ali (Hauser 1996: 11 = Thomas Hauser: Muhammad Ali in Perspective, Collins Publishers (1996)) ... und hier aus dem Mund des Meisters selber. Papa tanzt Tango (Diskussion) 20:30, 29. Jan. 2022 (CET)Beantworten

Corona-Warn-App

Ich habe die Corona-Warn-App des RKI schon von Anfang an, Blutooth und GPS ist dauerhaft an, Handy ist immer in der Hosentasche und gehe nie ohne weg. Dennoch habe ich noch nie eine dieser roten "erhöhtes Risiko"-Warnmeldungen (und auch keine gelben) erhalten. Ich kenne auch in meinem unmittelbaren Freundeskreis niemanden, der ne Meldung bekommen hat. Mache ich etwas falsch oder wird die App nicht häufig genug benutzt (laut App wurden gestern über 40.000 Personen gewarnt (was theoretisch 0,005% der deutschen Bevölkerung entspricht). Also dass ich in der Gaststätte, in der Uni und im Zug nie Menschen nahe genug komme, bezweifle ich. ... Vielen Dank im Vorraus! --2003:EC:BF16:1A34:38F0:4AA7:7443:CB2 14:29, 29. Jan. 2022 (CET)Beantworten

Die App ist offenbar nutzlos und Schrott. Bleibt wieder mal ein Klopper an Ausgaben, die der Steuerzahler tragen muss.--BanditoX (Diskussion) 15:18, 29. Jan. 2022 (CET)Beantworten
Das ist mitnichten „offenbar“. Das Warnrot setzt ja außer der Abstandsunterschreitung zu einem Infizierten noch zweierlei voraus: (1), dass derjenige seine Infektion auch in seine App eingetragen hat, und (2), dass man sich mindestens 10 Minuten lang durchgehend näher als anderthalb Meter neben diesem aufgehalten hat. --Wwwurm Paroles, paroles 15:33, 29. Jan. 2022 (CET)Beantworten
die App wurde aus einer infantilen technikgläubigen Haltung heraus geschaffen. Für mich als Informatiker war von vornherein klar, dass diese Idee kompletter Humbug ist, der in der Realität nie funktioniert. --91.21.0.182 22:48, 29. Jan. 2022 (CET)Beantworten
Der logische Schritt von „gestern wurden über 40.000 Personen gewarnt“ zu „Die App ist offenbar nutzlos und Schrott“ ist mir nicht ganz klargeworden, könntest du den für Begriffstutzige wie mich noch mal etwas näher ausführen? --Kreuzschnabel 21:54, 29. Jan. 2022 (CET)Beantworten
Gelbe Warnungen gibt es nicht. Bei niedrigem Risiko bleibt die App grün und es gibt keine Push-Nachricht. Wenn man nicht jeden Tag in die App guckt, kriegt man das also gar nicht mit. Im Kontakttagebuch kannst du sehen, falls es Begegnungen mit niedrigem Risiko in den letzten zwei Wochen gab. --Entbert (Diskussion) 16:43, 29. Jan. 2022 (CET)Beantworten
PS: Für die App brauchst du kein GPS.--Entbert (Diskussion) 16:44, 29. Jan. 2022 (CET)Beantworten
Doch: [11], "Die App verlangt Standortzugriff. Warum?" --BlackEyedLion (Diskussion) 18:38, 29. Jan. 2022 (CET)Beantworten
Steht doch in Deiner Quelle, einfach lesen: "Bei vielen Android-Geräten muss die grobe Standortermittlung aktiviert sein, damit die Technologie Bluetooth Low Energy (Bluetooth LE) genutzt werden kann. Das hängt nicht mit der Corona-Warn-App zusammen, sondern mit der in den meisten Smartphones verwendeten Technologie." --2001:16B8:5C9E:9D00:81B6:A35:7A56:4143 18:53, 29. Jan. 2022 (CET)Beantworten
"Erst seit Android 11 funktioniert Bluetooth LE auch ohne Standortfreigabe. Allerdings ist die Corona-Warn-App so konzipiert, dass sie auch auf älteren Android-Versionen lauffähig ist. Deshalb kann es sein, dass sie auch auf Android-Versionen ab 11 den Standortzugriff verlangt." Bei älteren Android-Versionen kann die App gar nicht ohne Standortzugriff funktionieren. --BlackEyedLion (Diskussion) 22:18, 29. Jan. 2022 (CET)Beantworten
Insgesamt 1,2 Millionen mal ist die rote Risikowarnung der Corona-Warn-App in der zweiten Januarwoche angesprungen. Bei mir war die Warn App im letzten Jahr auch schon zwei mal Rot. Nur, weil man bisher Glück gehabt hat, sollte man nicht anfangen zu Verallgemeinern. --2001:16B8:5C9E:9D00:81B6:A35:7A56:4143 19:08, 29. Jan. 2022 (CET)Beantworten

Was offenbar nicht geht, - soweit ich weiß -, ist ein "Probealarm", mit dem man ausprobieren könnte, ob auch alles richtig eingestellt ist. Seit 2020 niemals eine Warnung bekommen zu haben würde mich derzeit auch etwas skeptisch machen, ob alles "richtig eingestellt" ist. --Doc Schneyder Disk. 21:37, 29. Jan. 2022 (CET)Beantworten

Ich hab bisher erst zwei oder drei Warnungen bekommen, immer geringes Risiko, bis auf die letzte. Die Warnung kam am Mittwoch und der Kontakt war am Samstag davor. Ein Eigentest mit einem Test aus der PEI-Liste war dann aber negativ. Was ich irritierend finde ist, das die Corona-Warn-App nicht mehr das grellrote Coronavirus als Benachrichtigungsssymbol verwendet, sondern ein weißes Congstar-Symbol. Als Nicht-Congstar-Kunde hab ich mich erst einmal gewundert. --Rôtkæppchen₆₈ 22:00, 29. Jan. 2022 (CET)Beantworten
Völlig unabhängig von Sinn/Unsinn, zur Verlässlichkeit. Ich bekam Donnerstag den roten Warnhinweis wegen einer Begegnung am Dienstag. Auf dezente Nachfrage am Freitag konnte mir sofort gesagt werden, bei welchem Kollegen ich mich hätte infizieren können. Dieser hatte sich Mittwoch beim Arbeitgeber und in der App krank gemeldet und Dienstag hatte ich in einer 60-minütigen Besprechung ca. 2 Meter von ihm entfernt gesessen. Ich konnte auch mit Sicherheit ausschließen, dass es noch zu irgendeiner anderen Begegnung an diesem Tag gekommen ist. --2A02:908:2D12:8BC0:31CE:B947:32E3:A92F 23:15, 29. Jan. 2022 (CET)Beantworten
Ich bekomme regelmäßig Hinweise für Donnerstag. Und da bin ich regelmäßig im ÖPNV länger unterwegs. Allerdings bisher immer recht spät dananch, also so Mittwoch bis Donnerstag. --Bahnmoeller (Diskussion) 15:59, 30. Jan. 2022 (CET)Beantworten

Vielleicht hattest Du einfach keinen Kontakt? Ich selbst hatte bisher erst zwei und beide waren mir persönlich bekannt, so dass ich auch weiss, dass diese Meldungen korrekt waren. Die Meldungen erschienen kurz nachdem die beiden ihren Status in ihrer App mitgeteilt hatten. Die App erfüllt also ihren Zweck. Wenn sie keine Warnungen anzeigt, heisst das vermutlich also schlicht: "Glück gehabt". --GrandmasterA (Diskussion) 13:21, 31. Jan. 2022 (CET)Beantworten

Gespräch auf Facebook im "Originallook" als Beweis sichern

Es ist möglich, sich Kopien von den Dialogen in privaten Nachrichten auf Facebook zu speichern. Es ist natürlich auch möglich, die Texte eines Dialogs in Word zu speichern sowie als Screenshots. Aber erstere Sicherung ist umgekehrt chronologisch sortiert und schlecht lesbar. Letztere Methoden wären anfällig für Manipulation und wenig beweiskräftig. Gibt es einen Weg, solche Dialoge in "Originallook" zu speichern, so dass sie gesichert sind - selbst wenn der Account des Gesprächspartners auf Facebook gelöscht wird? --AF (Diskussion) 19:26, 29. Jan. 2022 (CET)Beantworten

Facebook bietet an, das du dir die Daten deines Kontos herunterladen kannst. Ich weiß nicht, ob a)dort die Nachrichten enthalten sind und b) on das auch für Unterhaltungen mit gelöschten Konten gilt. Zumindest a) kannst du ausprobieren. --2A01:598:A806:5B54:A897:239C:EDDE:63C8 08:46, 30. Jan. 2022 (CET)Beantworten
Bildschirmkopie?--2003:C1:974D:C900:8C2D:F02B:626B:1970 14:18, 30. Jan. 2022 (CET)Beantworten
Bildschirmkopie ist eine Weiterleitung auf den vom Fragesteller erwähnten Screenshot. --MannMaus (Diskussion) 15:56, 30. Jan. 2022 (CET)Beantworten

Sind LEDs dimmbar?

Der Fall: eine Lichterkette ist zu hell und sollte gedimmt werden. Die Parameter auf dem Beipackzettel:

20 LEDs (Strom, keine Batterie)
EU Plug-in Größe 3.8M*20LED
Produktleistung 1.2 W
IP-Rang IP44

Außerdem steht etwas sehr kleingedruckt auf dem Stecker (so wie Input/Output), könnte ich noch schauen. Achtung, made in china :-) - Danke, -jkb- 21:32, 29. Jan. 2022 (CET)Beantworten

Strom - keine Batterie :-)? Ich interpretiere das als mit Netzspannung betrieben. Grundsätzlich sind LEDs dimmbar, entweder über den Stromfluß oder das Tastverhältnis (die An-Aus-Zeit). Bei einer Lichterkette wird der Strom durch die Hintereinanderschaltung bestimmt. Du bräuchtest also ein Vorgeschaltgerät, dass die LEDs z.B. 50% Prozent der Zeit einschaltet und das schneller als die menschliche Flicker-Fusionsfrequenz. Keine Ahnung ob sowas kommerziell erhältlich ist. Gruß, --Burkhard (Diskussion) 21:44, 29. Jan. 2022 (CET)Beantworten
Es kommt auf die Innenschaltung des Treibers an. Wenn Du Pech hast, dann bewirkt ein Dimmer gar nichts, außer dass die Lichterkette bzw deren Treiber schneller bis sehr schnell kaputtgeht. Üblicherweise sit da ein Symbol drauf (gebogenes Dreieck im Kreis), wenn das Leuchtmittel dimmbar ist. Das Nicht-dimmbar-Symbol ist ein gebogenes Dreieck im durchgestrichenen Kreis, der von einem Dreieck umgeben ist. --Rôtkæppchen₆₈ 21:53, 29. Jan. 2022 (CET)Beantworten
Huch, bislang dachte ich, ich bin der geborene Elektroselbstversorger - Steckdosenversetzung, Fußbodenheizung im Bad ... Aber das hier ist krass: Tastverhältnis, Vorschaltgerät, Treiber... Zumal die Kette von meiner Frau betrieben werden möchte. Ich dachte es mir so, dass ich einen Dimmer kaufe, vielleicht mit Fernbedienung, und dort die Kette anschließe. Nun ruft sie, da sie etwas Alternatives ausgekugelt hat: Da liefert jemand eine Kette mit einer Mogelpackung Dimmer, womit man das nicht stufenlos, aber in vier Stufen "dimmen" kann. Ich schau mir das an. Aber wirklich zuerst vielen vielen Dank!!! -jkb- 22:21, 29. Jan. 2022 (CET)Beantworten
LEDs kann man immer heller und dunkler machen, indem man sie rasend schnell ein und ausschaltet. Also so 1000 mal pro Sekunde. Wenn es allerdings eine Lampe ist mit eingebauter Elektronik, dann kann es sein, dass die Elektronik das nicht mitmacht. --91.21.0.182 22:41, 29. Jan. 2022 (CET)Beantworten
@-jkb-: Du könntest mal versuchen, das „Kleingedruckte“ auf dem Typenschild mit der Smartphonekamera oder Lupe sichtbar zu machen. Vielleicht steht da etwas erhellendes drauf. --Rôtkæppchen₆₈ 22:55, 29. Jan. 2022 (CET)Beantworten

Laienfrage: Mit welcher Spannung werden die LEDs der Lichterkette denn versorgt? Ansonsten: Hier ist viel über die Dimmbarkeit von LEDs nachzulesen: https://www.beleuchtungdirekt.de/blog/welche-led-lampen-sind-dimmbar --Doc Schneyder Disk. 01:19, 30. Jan. 2022 (CET)Beantworten

Datei:Stecker Lichtterkette.JPG
Das Kleingedruckte vom Stecker nebenan. Und ja, die üblichen 220 V. Und möglicherweise brauche ich Hilfe: es gab hier eine Lizens so wie "vorübergehend zu Demozwecken" oder ähnlich, ich finde sie nicht. Kann jemand schauen ob es so OK ist? -jkb- 14:43, 30. Jan. 2022 (CET)Beantworten
Danke. Du brauchst also einen Dimmer, der mit 3 bis 4 Volt und 600 Milliampere zurechtkommt. Den schaltest Du dann zwischen Steckernetzteil und Lichterkette. der hier könnte passen, ist aber ein Bastelprojekt und ist teurer als die Lichterkette. --Rôtkæppchen₆₈ 18:06, 30. Jan. 2022 (CET). --Rôtkæppchen₆₈ 18:06, 30. Jan. 2022 (CET)Beantworten
Warum machst du dir bei einem selbsterstellten Foto Sorgen um die Lizenz? --Digamma (Diskussion) 19:52, 30. Jan. 2022 (CET)Beantworten

30. Januar

Warum Sturmtief und kein Sturmhoch?

Sturm entsteht durch Luftdruckunterschied. Wodurch wird die Symmetrie zwischen Hoch- und Tiefdruck gebrochen? Ist es die Beschleunigung der Luftmassen beim Weg zum Tief? --2003:D3:7F14:2F00:9D07:8250:BD3:264B 11:48, 30. Jan. 2022 (CET)Beantworten

Sturm entsteht durch hohe Druckgradienten, d.h. gro0e Druckunterschiede auf kurze Distanz. Die Druckgradienten sind im Umfeld eines Tiefes deutlich größer als im Umfeld eines Hoches (warum könnte ein Meteorologe erklären). Deshalb treten Stürme vor allem in Verbindung mit Tiefs auf.--Mhunk (Diskussion) 12:05, 30. Jan. 2022 (CET)Beantworten
Danke, daß Du meine Frage wiederholt hast ;-) Sie harrt weiterhin einer Antwort. --2003:D3:7F14:2F00:9D07:8250:BD3:264B 14:06, 30. Jan. 2022 (CET)Beantworten
Ich verstehe, was gemeint ist: der Sturm ist an der Grenze zwischen Tief und Hoch, wird aber dem Tief zugerechnet. – Nun, extrem unwissenschaftlich formulierte Vermutung: Das Hoch drückt die Luft von sich weg zum Tief hin, während das Tief sie ansaugt, also ist der Sturm dichter ums Tief rum. Mal bei Ventusky mit ein paar Kartenansichten rumspielen, z.B. auf Luftdruck umschalten (aktuell ist Wind eingestellt) und in der Zeitachse wandern, es sieht aus, als ob das stimmen könnte. Und auf jemand Fundierteren warten ;-) eryakaas • D 14:25, 30. Jan. 2022 (CET)Beantworten
Ich merke grad, ich hab eigentlich auch wiederholt, nur mit mehr Worten. eryakaas • D 14:31, 30. Jan. 2022 (CET)Beantworten
Gradientwind#Kritische Krümmung erklärt es hochwissenschaftlich. Besonders der letzte Satz des Abschnitts ist beachtenswert. --Rôtkæppchen₆₈ 14:33, 30. Jan. 2022 (CET)Beantworten
Danke sehr. Laienhaft ausgedrückt, könnte es also auch ein statistischer Effekt sein: Der Phasenraum um eine Quelle (Hoch) ist größer als der um eine Senke (Tief). --2003:D3:7F14:2F00:9D07:8250:BD3:264B 15:19, 30. Jan. 2022 (CET)Beantworten
Offensichtlich weiß der Fragesteller nichts mit dem Begriff Gradient anzufangen, sonst wäre ihm der Unterschied zwischen Luftdruckunterschied (seine Frage) und Druckgradient (meine Antwort) ins Auge gefallen. Die Symmetrie wird dadurch gebrochen, daß bei einem Hoch kein grßer Druckgradient auftritt, dagegen sehr wohl bei einem Tief. Der Druckunterschied zwischen Hoch und Tief führt somit zu einem Sturm beim Tief und nicht beim Hoch.--Mhunk (Diskussion) 14:52, 30. Jan. 2022 (CET)Beantworten
Offensichtlich hältst Du die Bälle gerne nicht so flach. Vielleicht ist diese meine Spekulation nicht so kühn wie Deine. ;) --2003:D3:7F14:2F00:9D07:8250:BD3:264B 15:15, 30. Jan. 2022 (CET)Beantworten
Wie kommst Du drauf, dass ich spekulieren muss? Sturm#Entstehung_und_Erscheinung sagt das gleiche wie ich, nur in anderen Worten. Und die Corioliskraft verhindert normalerweise, wie ich gerade nachgelesen habe, dass sich bei den Hochs ein starker Druckgradient ausbildet.--Mhunk (Diskussion) 16:16, 30. Jan. 2022 (CET)Beantworten
Offensichtlich bezogen sich die Spekulationen auf die vermuteten Offensichtlichkeiten. Übrigens ist ein Gradient kein Skalar. ;) --2003:D3:7F14:2F00:D0E4:911C:F1A0:B4B3 20:00, 30. Jan. 2022 (CET)Beantworten

Kann man die Wasserhärte mit einem Membranfilter reduzieren? Wie klein (µm) sind die Bestandteile im Wasser?

Hallo,

mich würde interessieren ob man nicht gewollte Bestandteile des Wassers auch mit Membranfiltern aus dem Grundwasser filtern kann. Ich habe gelesen das diese in den letzten Jahren immer besser geworden sind. Konkret geht es bei mit um die Ertüchtigung von Brunnenwasser in Nutzwasser für WC. Leider ist das Grundwasser in weiten Teilen von Deutschland durch ehemals andere Regularien aus der Massentierhaltung und Agrarpolitik sehr belastet. Hinzu kommen teilweise noch natürliche Belastungen, welche das Grundwasser nur noch sehr eingeschränkt nutzbar machen.

Die Wasseraufbereitungsverfahren, welche ich recherchieren konnte, sind sehr aufwändig. Vorfilter, Sedimentfilter 5 und 1 µm, Enthärtung über Harze, Umkehrosmose sowie Nachfilter würden Anlagekosten von über 2.000 € verursachen und ziehen einen hohen Wartungs- und Betriebsstoffeinsatz (Salze, Filter) nach sich.

Daher die Frage, kann man nicht mit einem Membranfilter ungewollte Bestandteile wie °dH, Sulfat, Nitrat ausfiltern? Laut Periodensystem ist Wasser- und Sauerstoff ziemlich leicht? Aber wie groß sind die gelösten Salze und anderen Verbindungen im Wasser konkret?

--95.90.103.87 11:58, 30. Jan. 2022 (CET)Beantworten

Gelöste Salze (Ionen) bekommst du nur über Ionentauscher ("Harze") oder osmotische Prozesse aus dem Wasser heraus. Pte. Salt (Diskussion) 12:07, 30. Jan. 2022 (CET)Beantworten
Die Antwort hast Du bereits selbst gegeben: Bei der Umkehrosmose handelt es sich um Membranfilter. Nur leider brauchen die einen hohen Druck, um zu funktionieren. Hydrogencarbonat bekommst Du mit Entcarbonisierung aus dem Wasser. Damit ist auch ein Teil der °dH weg. --Rôtkæppchen₆₈ 13:03, 30. Jan. 2022 (CET) Zu den Größen (zusamengegoogelt, Radius in Picometern): Ca2+ 100, Mg2+ 72, HCO3 156, SO42− 258, NO3 179, H2O 135. --Rôtkæppchen₆₈ 13:30, 30. Jan. 2022 (CET)Beantworten
danke. Also wenn Ca > H2O geht es wirklich nicht.... --95.90.103.87 18:59, 30. Jan. 2022 (CET)Beantworten
Härte von Brunnenwasser, durch Massentierhaltung, Agrarpolitik und natürliche Belastungen nur noch sehr eingeschränkt nutzbar, mit einem Membranfilter reduzieren, um es in Nutzwasser für WC zu ertüchtigen? --Vsop (Diskussion) 22:32, 30. Jan. 2022 (CET)Beantworten
Ist doch klar: Das Nitrat aus dem Brunnenwasser reagiert mit dem Ammonium aus den Fäkalien zu unerwünschtem Sprengstoff. *scnr* --Rôtkæppchen₆₈ 23:13, 30. Jan. 2022 (CET)Beantworten

Frage zum neuen Tuhl 'Abrufstatistik'

Hallo Kolleginnen und Kollegen Wikipedianer, die neue Möglichkeit der Abrufstatistik begeistert mich. Habe schon beobachtet, kommt der "Graf von Monte Christo" im Fernsehen, schnellt der Artikel Der Graf von Monte Christo gleich mal um 20.000 Einblicke hoch! Jetzt meine Frage: Am 11. Januar habe ich den Artikel The King Steps Out eröffnet. Am 26. Januar gab es nun eine Spitze mit etwa 75 Einblicken mehr. Was war an diesem Tag? Sissi im Fernsehen? Sternberggedenktag? Rolz Reus (Diskussion) 12:40, 30. Jan. 2022 (CET)Beantworten

Da ist die Hauptdarstellerin Grace Moore verunglückt. BTW: Die Abrufstatistik ist nicht neu. -- Jesi (Diskussion) 12:55, 30. Jan. 2022 (CET)Beantworten
das schreibt sich "Tool". --2003:C6:1718:FF5D:F503:4CD:A4C4:A992 13:57, 30. Jan. 2022 (CET)Beantworten
wer sich Rolz Reus nennt, weiß das (-: eryakaas • D 14:28, 30. Jan. 2022 (CET)Beantworten
… und der geneigte Leser fragt sich nur, wieso er seinen neuen Artikel nicht „Se King Schtepps Aut“ genannt hat (dschörmen eckzent intändet, „Tuhl“ ist ja auch keine englische Aussprache). --Kreuzschnabel 19:03, 30. Jan. 2022 (CET)Beantworten
... sondern welche? --Gruenschuh (Diskussion) 07:05, 31. Jan. 2022 (CET)Beantworten
Denglische. „tuhl“ spreche ich mit geschlossenem u wie „Stuhl“ ohne sch-Laut. Der Vokal in „tool“ ist viel offener. --Kreuzschnabel 10:25, 31. Jan. 2022 (CET)Beantworten
Wenn ich mirs bei dict.cc anhöre, klingt das "oo" immer noch wie "uh", nur das engl./amerik. "l" rutscht wie ein erstickend quakender Frosch in die Luftröhre. --Gruenschuh (Diskussion) 10:26, 1. Feb. 2022 (CET)Beantworten

Sören Storks

Bundesliga-Schiedsrichter Sören Storks hatte diese Saison noch keinen einzigen Bundesliga-Einsatz. Was ist da los? Persönlicher Wunsch oder keine ausreichende Leistung? Gruß, -- 91.56.128.29 13:40, 30. Jan. 2022 (CET)Beantworten

Offenbar diese Saison nur als Video-Assistent im Einsatz. Also kann keine ausreichende Leistung wohl nicht der Grund sein. --TheRunnerUp 17:25, 30. Jan. 2022 (CET)Beantworten
Was macht das denn für einen Sinn? Fitness-Test nicht bestanden? Leitet auch keine Spiele in der 2. Bundesliga [12] -- Toni 21:56, 30. Jan. 2022 (CET)Beantworten

Köln, Freiburg, Ulm

Wo flog der Fotograf?

Jetzt kommt ein Sonntagsrätsel: In welcher Stadt steht der Dom auf dem Bild nebenan (für manche sicher zu leicht)?--Blue 🔯 17:51, 30. Jan. 2022 (CET)Beantworten

Datei:Kölner Dom um 1900.jpg, siehe insbesondere die Treppen vor dem Südportal. 62.157.2.126 18:20, 30. Jan. 2022 (CET)Beantworten
Hm, Datei:Kölner Dom um 1900.jpg stellt mich vor ein weiteres Sonntagsrätsel: Sie ist laut „globale Dateiverwendung“ angeblich in Hunderten von Seiten eingebunden, vor allem in Hindi, Bangla und Englisch, z. B. in en:Strasbourg Mosque, sie ist aber dort nicht eingebunden. Wer kann das lösen? --Pp.paul.4 (Diskussion) 22:16, 30. Jan. 2022 (CET)Beantworten
Doch, wenn Du genau hinschaust, ist sie dort über eine Vorlage eingebunden, die auf religiöse Orte oder Gebäude hinweist. Dort ist ein kleines Piktogramm des Doms zu sehen. 80.71.142.166 23:04, 30. Jan. 2022 (CET)Beantworten
Nach Betrachtung des Piktogramms mein Senf: Das Piktogramm sieht aus wie die Zehen eines Fußes, als Piktogramm sollte man besser etwas sofort als Referenz auf das Gemeinte Wahrnehmbares wählen, die Vorlage ist zu kleinteilig, besser wäre zum Beispiel eine klassisch dimensionierte Holzkirche mit Schiff, Turm und einem Kreuz obendrauf. Wo kann ich das anstoßen?--Blue 🔯 23:08, 30. Jan. 2022 (CET)Beantworten
Problem erkannt. Das Mini-Bild (für mich nicht erkennbar) ist nicht nur in en:Template:France-religious-struct-stub und in en:Template:Religious-struct-stub eingebunden, sondern in vielen anderen Sprachen as:সাঁচ:Religious-struct-stub, bh:टेम्पलेट:धार्मिक-स्थल-आधार usw. --Pp.paul.4 (Diskussion) 00:38, 31. Jan. 2022 (CET)Beantworten
Das sind die Abstriche, die Wikipedia als Soziales Experiment mit sich bringt. Ich habe jetzt auch keine „Verve“ für Meingungsbilder etc. wegen dieses Fehlgriffs.--Blue 🔯 09:51, 31. Jan. 2022 (CET)Beantworten
Symbole genug gibt es unter c:Category:Religious building icons, wenn jemand sich daran versuchen möchte. --Pp.paul.4 (Diskussion) 21:11, 31. Jan. 2022 (CET)Beantworten

Lothar Brauner

Andreas Kossert zitiert in "Ostpreußen Geschichte und Mythos" (Siedler, Erste Auflage) auf Seite 83 Lothar Brauner zu Lovis Corinths Gemälde "Kirchhof in Nidden": "gibt den Charakter der Landschaft auf der engen Landzunge wieder, zu dem auch die Kreuze der Verstorbenen gehören". Wer ist (war) Lothar Brauner und aus welcher Quelle zitiert Andreas Kossert?

--2003:D9:8F24:F100:6116:B9B8:6143:5A1C 18:25, 30. Jan. 2022 (CET)Beantworten

Lothar Brauner war Kustos an der Berliner Alten Nationalgalerie (vor und nach der Wende). Er hat den Ausstellungskatalog zu Lovis Corinth mit herausgegeben [13]. Ob das Zitat dort enthalten ist, war nicht rauszufinden.--Mhunk (Diskussion) 19:53, 30. Jan. 2022 (CET)Beantworten
Danke --2003:D9:8F24:F100:FC16:77CA:93EF:8DD8 11:19, 31. Jan. 2022 (CET)Beantworten

Suche einen Roman der älter als 67 Jahre ist ...

... und sich mit dem alten Ägypten beschäftigt. Hintergrund meiner Frage ist, dass ich seit acht Jahren einen kleinen Lesezirkel unterhalte und jedes Jahr ein Buch gelesen wird. Die Bücher müssen älter als 67 Jahre sein und, wobei sich außer mir niemand an die Vorgabe hält, noch nicht vom Vorschlagenden gelesen sein. Daher fällt Mika Waltari mit "Sinuhe der Ägypter" raus, kenne ich schon und fands gut. "Nofretete" von Nicole Vidal ist wohl leider noch nicht alt genug. Gibt es einen weiteren Klassiker in dieser ägyptischen Richtung? Einer der mit "Quo Vadis" und "Ben Hur" mithalten kann? Rolz Reus (Diskussion) 19:17, 30. Jan. 2022 (CET)Beantworten

Rächende Geister von Agatha Christie ist aus dem Jahr 1944.--2A02:8109:BD40:65C4:0:0:0:8E6F 20:27, 30. Jan. 2022 (CET)Beantworten
Thomas Mann: Joseph und seine Brüder (4 Bände). --2003:D0:2F18:57D9:9134:9668:B1F7:301A 22:42, 30. Jan. 2022 (CET)Beantworten
Ob jemand, der Ben Hur interessant fand, sich auch mit T.M. anfreunden kann? --2001:871:F:AC3C:8C21:1C6B:4B17:E3C1 23:14, 30. Jan. 2022 (CET)Beantworten
Das 2. Buch Mose. --2003:F7:DF08:9C00:1D9:D571:838F:D421 23:01, 30. Jan. 2022 (CET)Beantworten
Der Brand der Cheopspyramide --2001:871:F:AC3C:8C21:1C6B:4B17:E3C1 23:14, 30. Jan. 2022 (CET)Beantworten
Ein Klassiker der Ägypten-Romane war Georg Ebers, lebte im 19. Jh. --2003:D0:2F18:57D9:9134:9668:B1F7:301A 23:16, 30. Jan. 2022 (CET)Beantworten
Das Land aus dem Meeresschaum von Iwan Jefremow. --Megalogastor (Diskussion) 01:35, 31. Jan. 2022 (CET)Beantworten
Pharao von Bolesław Prus. Gab es auch mal als Monumentalschinken. --Kiew86 (Diskussion) 12:02, 31. Jan. 2022 (CET)Beantworten
Danke schön! Den Film "Pharao" habe ich in grauenvoller avantgardistischer Erinnerung. Das Danke ist trotzdem ernst gemeint. Ich schlage jetzt Georg Ebers "Die Schwestern" vor. Rolz Reus (Diskussion) 13:11, 31. Jan. 2022 (CET)Beantworten

ARD-Livestream

Ich wollte mich vorhin bei Anne Will per livestream zuschalten. Eigentlich kein Problem. Aber es heißt trotz mehrfachem Versuchs immer: „Diese Sendung steht Ihnen aus rechtlichen Gründen leider nicht zur Verfügung.“ Weiß jemand Genaueres? --2003:D0:2F18:57D9:9134:9668:B1F7:301A 22:41, 30. Jan. 2022 (CET)Beantworten

Einfach hier nachfragen. --Rôtkæppchen₆₈ 22:59, 30. Jan. 2022 (CET)Beantworten
Danke, eben erfolgt. Die folgenden Sendungen laufen übrigens einwandfrei. --2003:D0:2F18:57D9:9134:9668:B1F7:301A 23:13, 30. Jan. 2022 (CET)Beantworten
Auch im aktuellen Programmlistung auf Programm.ARD.de fehlt für Anne Will der Link zur Mediathek, den aber fast alle anderen Sendungen haben. Das scheint wohl ein Rechte- bzw Lizenzproblem zu sein. --Rôtkæppchen₆₈ 23:22, 30. Jan. 2022 (CET)Beantworten
ist jetzt aber unter Link in der Mediathek zu finden. Keine Ahnung, wie lange. Vielleicht hat einfach nur jemand den falschen Knopf gedrückt gehabt.--Nordprinz (Diskussion) 00:31, 31. Jan. 2022 (CET)Beantworten
Jetzt ist auch im Programmlisting ein Link auf die Mediathek. Auf Mediathekview ist die Sendung (noch) nicht, was aber nichts heißen muss. --Rôtkæppchen₆₈ 00:42, 31. Jan. 2022 (CET)Beantworten
Mediathek-view hinkt meist ein paar Stunden hinterher. Hab ich auch schon bei anderen Sendungen festgestellt. Gut, die müssen erst die Mediathek elektronisch auswerten. --Nordprinz (Diskussion) 11:01, 31. Jan. 2022 (CET)Beantworten

31. Januar

Dialoge in Spielfilmen

Müssen die Filmschauspieler heutzutage immer noch ihre Texte auswendig wissen oder gibt es da elektronische Hilfsmittel? Ich hatte erst den Einfall, dass die Texte nachvertont werden, aber das geht nicht wegen den Lippenbewegungen ... --95.114.105.55 05:41, 31. Jan. 2022 (CET)Beantworten

Die Texte werden mittlerweile via Kontaklinsen direkt auf die Netzhaut projeziert. Dank digitaler Nachbearbeitung sieht man die Antennen die den Schauspielern aus der Fontalle ragen im fertigen Film auch kaum... Suchtestt du nach sowas in der Art? -Ani--46.114.156.33 07:55, 31. Jan. 2022 (CET)Beantworten
Cue card
Und für Impfgegner, die such weigern, sich einen entsprechenden Chip injizieren zu lassen, gibt es immer noch die guten alten Cue Cards. --Rôtkæppchen₆₈ 08:18, 31. Jan. 2022 (CET)Beantworten
Ich finde in Manche mögen's heiß die Information nicht, dass Marilyn Monroe ihren Text angeblich abgelesen hat, was man an den Augenbewegungen erkennen soll. Jedenfalls sind Cue Cards nicht beliebt, weil man eben an den Augen erkennen kann, dass der Text abgelesen wird. --BlackEyedLion (Diskussion) 09:11, 31. Jan. 2022 (CET)Beantworten
(Quetsch) Hier im Video bei 03:56 recht gut zu erkennen. --Karotte Zwo (Diskussion) 12:17, 31. Jan. 2022 (CET)Beantworten
Anders als im Theater müssen die Schauspieler ihren Text nur für die jeweiligen Szenen kennen. Und diese werden ja oft an unterschiedlichen Tagen gedreht. Meist sind es nur ein paar Sätze, sodass es nicht schwer ist, den Text zu lernen. Ich bin sicher, dass mithilfe von künstlicher Intelligenz schon bald auch hier leicht nachgeholfen werden kann, wenn der Text mal nicht sitzt oder etwas nicht lippensynchron ist. Dann macht die AI es nachträglich lippensynchron. Das funktioniert ja jetzt schon halbwegs gut, wie man an Videos von Obama und Trump sieht, denen irgendetwas in den Mund gelegt wird. Wenn wir noch drei Jahre warten, ist soetwas perfektioniert. Mal sehen, ob man dann überhaupt noch Synchronsprecher braucht. Google Translate und Deepl funktionieren ja jetzt schon immer besser. Und neulich gab es eine Anwendung, bei der man nur ein paar Sekunden sprechen musste und die App hat gelernt, wie die Stimme klingt und konnte sie danach imitieren. Wenn man es weiterdenkt: Braucht man in 10 Jahren überhaupt noch Schauspieler? 80.71.142.166 09:06, 31. Jan. 2022 (CET)Beantworten
Diese Diskussion lief schon vor gut 100 Jahren: Braucht man nach der Erfindung der Schallplatte überhaupt noch Musiker, oder gibt es schon bald Apparaturen, mit denen die Komponisten die Rillen selbst in die Platten schnitzen können, ohne sich mit Proben und Aufnahmen herumärgern zu müssen? Und als das durch war, war der Tonfilm der neue Feind, der die Kultur zerstört … --Kreuzschnabel 09:44, 31. Jan. 2022 (CET)Beantworten
Da gute Schauspielleistung nicht darin besteht, einen Text vorzulesen oder herunterzuleiern, sondern eine Rolle zu spielen und dabei einen vorgegebenen Text wie eine ganz natürliche spontane Äußerung der dargestellten Rolle klingen zu lassen: Ja, ich persönlich bin davon überzeugt, dass das ohne Auswendiglernen (besser noch Inwendiglernen, die Englischen haben da mit „to learn by heart“ eindeutig den besseren Ausdruck) des Textes nicht möglich ist. Eine Alternative ist nur Improvisieren, wo es erwünscht ist. Aber ein von wo auch immer abgelesener Text klingt prinzipiell anders als ein in die jeweilige Szene integrierter Text. --Kreuzschnabel 09:49, 31. Jan. 2022 (CET)Beantworten
Kann zumindest nicht schaden für den Notfall ein paar Schauspieler im Keller zu lagern. Strom kann ja auch mal ausfallen. Mal ernsthaft: Was sollen die Schauspieler denn dann machen? Von ein paar Alkohol- & Drogenexzessen kann man doch nicht anständig leben. Und noch ernsthafter (diesmal wirklich): daran möchte ich nicht denken. -Ani--46.114.156.33 09:48, 31. Jan. 2022 (CET)Beantworten

Teleprompter lautet die naheliegende Antwort. Wird auch beim Fim eingesetzt, wenn auch wohl seltener als bei TV-Moderationen oder Politikerreden. --FordPrefect42 (Diskussion) 12:32, 31. Jan. 2022 (CET)Beantworten

Anschlussfrage

Aus persönlichem Interesse, wo wir gerade mal dabei sind: Sagen wir mal, ein Film wird im Original englisch produziert und soll dann deutsch synchronisiert werden. Dafür braucht das Synchronstudio eine Internationale Tonspur (IT), die zwar die Geräusche der Szenen enthält, nicht aber die zu übersetzenden Dialoge. Dafür kann ich mir drei Vorgehensweisen vorstellen: 1.: Die IT wird von einem Geräuschemacher nachträglich erzeugt, d.h. sämtliche Szenengeräusche, Atmo, Vogelzwitschern, Meeresbrandung etc. der deutschen Version sind nicht die am Set aufgenommenen Originalgeräusche, der Film wird vollständig stumm übernommen. 2.: Der Film wird am Set als IT aufgezeichnet, also mit stummen Lippenbewegungen, und auch für die Originalversion anschließend von den Originaldarstellern synchronisiert. 3.: Der Film wird Szene für Szene zweimal aufgezeichnet, einmal mit gesprochenen Dialogen, einmal stumm für den IT, d.h. die in der Synchro zu sehenden Szenen sind nicht dieselben Takes wie die der Originalversion. – Ich halte 1. für die wahrscheinlichste, aber ich weiß es erstaunlicherweise wirklich nicht. --Kreuzschnabel 10:08, 31. Jan. 2022 (CET)Beantworten

Das Erste ist richtig. en:Foley (filmmaking)#Uses: „Scenes where dialogue is replaced using dubbing also feature Foley sounds.“ Siehe auch music and effects track (M&E track) oder sound effects track, en:Field recording und en:Dubbing (filmmaking)#ADR/post-sync. --BlackEyedLion (Diskussion) 10:29, 31. Jan. 2022 (CET)Beantworten
Der Artikel Internationale Tonspur ist tatsächlich ausbaufähig was die Entstehung jener Tonspur angeht. Es ist in der Praxis so, dass mit verschiedenen Mikros aufgenommen wird. Am Set interessieren in der Regel hauptsächlich die Dialoge, deswegen die berühmten Mikrofonarme, die über den Köpfen der Schauspieler schweben, aber auch Mikros am Körper, etc. Die sind nicht dafür da, irgendwelche Hintergrundgeräusche aufzunehmen, sondern ausschliesslich die Dialoge. Wenn man die Original-Geräuschkulisse am Set erfassen will, wird das mit anderen, entsprechend platzierten und ausgerichteten Mikrofonen gemacht. Im Tonstudio werden da dann noch weitere Geräusche ergänzt, oder eben gleich alle Hintergrundgeräusche erzeugt - und bei den Dialogaufnahmen eben auch Störgeräusche entfernt. Es kommt auch vor, dass Schauspieler ihre eigenen Aufnahmen nachsynchronisieren müssen, weil die Dialogaufnahme eben doch zu viele Hintergrundgeräusche erfasst hat, aber so weit, dass man eine Produktion gleich von vorneherein auf dein Szenario 2 oder 3 auslegt, geht man nicht. --Karotte Zwo (Diskussion) 10:34, 31. Jan. 2022 (CET)Beantworten
In der Praxis gibt es dafür Geräuschexperten. Pferdehufen werden da durch das Klappern halber Kokosnüsse auf dem Tisch nachgeahmt. Manches kommt vom Band. 80.71.142.166 10:59, 31. Jan. 2022 (CET)Beantworten
Dass es den Beruf gibt, ist mir bekannt, sonst hätte ich ihn nicht verlinkt :) Meine Frage war, ob der IT in Filmproduktionen so oder anders erzeugt wird. --Kreuzschnabel 11:19, 31. Jan. 2022 (CET)Beantworten
Das wäre mindestens 1a oder sogar 4: Es sind dieselben Takes und zu Teilen auch die originalen Geräusche, wobei eine spezielle IT-Tonmischung dafür sorgt, dass die am Set gesprochenen Originaldialoge nur so leise mit draufkommen, dass sie nach der Synchronisation nicht stören. --Kreuzschnabel 11:00, 31. Jan. 2022 (CET)Beantworten
"Ich hatte erst den Einfall, dass die Texte nachvertont werden...". Ich habe vor einiger Zeit gelesen, dass das tatsächlich passiert. Teils, weil sich hinterher herausstellt, dass die Stimme von der Aufnahme am Set nicht gut rüberkommt, teils weil man möchte, dass etwas anders betont wird oder etwa erst jetzt auffällt, dass einer der Schauspielenden sich versprochen hat etc. Leider finde ich die Quelle nicht mehr, der das zu entnehmen ist. --77.10.19.205 15:48, 31. Jan. 2022 (CET)Beantworten
Hatte Karotte Zwo ja bereits erwähnt. Nennt sich übrigens en:Automated dialogue replacement (ADR). --Magnus (Diskussion) 15:51, 31. Jan. 2022 (CET)Beantworten
Auch hier erklärt und ausbaufähig STEM (die Tonspur besteht aus "Teilspuren".--Wikiseidank (Diskussion) 18:16, 31. Jan. 2022 (CET)Beantworten
Beides. Meist wird in dialogfreien Szenen der Originalton verwendet, geht dies nicht, dann wird der Ton selbst kreiert, wie etwa Straßenlärm, Stimmengemurmel im Hintergrund etc. In der Regel vom Band, es gibt Geräusche-CDs, DVDs, die auch Hobbyfilmer verwenden, z. B. eine von George Lucas. --5DKino (Diskussion) 18:46, 31. Jan. 2022 (CET)Beantworten

Ernst Erich Müller (Politiker) (1904–1982), deutscher Politiker (NSDAP), Fremdenlegionär und Spion

Wer ist mit Ernst Erich Müller (Politiker) (1904–1982), deutscher Politiker (NSDAP), Fremdenlegionär und Spion gemeint? --Dr Lol (Diskussion) 07:32, 31. Jan. 2022 (CET)Beantworten

jener.--Mhunk (Diskussion) 09:15, 31. Jan. 2022 (CET)Beantworten
Danke. 62.157.2.126 13:49, 1. Feb. 2022 (CET)Beantworten
Dieser Abschnitt kann archiviert werden. 62.157.2.126 13:49, 1. Feb. 2022 (CET)

Unsynchroner Ton

Gelegentlich fällt mir bei manchen YouTube-Videos auf, daß Bild und Ton wohl etwas unterschiedlich schnell laufen oder sonstwie außer Tritt geraten, so daß nach ca. 1 h Zeitverschiebungen von einigen Sekunden vorliegen, um die der Ton dem Bild nachhinkt. Wie kommt sowas eigentlich zustande, d. h. wer hat denn da was, und vor allem wie, vergurkt? --95.116.29.20 12:41, 31. Jan. 2022 (CET)Beantworten

Haste mal n Link? Generell sind Ton und Bild bei digitalen Videos nicht wie im chemischen Tonfilm fest miteinander verheiratet, sondern zwei separate Streams, um deren Synchronität sich der wiedergebende Player bemühen muss. Wenn er andererseits kaputte Eingangsdaten bekommt, kann er das auch nicht mehr. --Kreuzschnabel 13:45, 31. Jan. 2022 (CET)Beantworten
Link --95.116.29.20 22:08, 31. Jan. 2022 (CET)Beantworten
Grafikkarte, Framegrabber und TV-Tuner aus den 1990ern
Es kommt aber auch auf Übertragungsweg und Dateiformat an. Es gibt Aufnahmeformate, die Bild, Ton und Untertitel als unabhängige Streams speichern. Da kann es dann bei Quarzfrequenzabweichungen von Framegrabber und Audioadapter zu Abweichungen kömmen. Bei manchen Dateiformaten sind Bild und Ton über einen Zeitcode gekoppelt. Kommt es da zu zu großen Abweichungen, machen Bild oder Ton einen kleinen Sprung, um die Synchronizität wiederherzustellen. Bei Überttragungssstörungen (DVB-S,C,T etc) kann es zu Störungen des Zeitcodes kommen. Bei der Weiterverarbeitung eines gestörten Mitschnittes sind dann Verschiebungen von Bild und ton möglich. Vor 25 Jahren hab ich AVI-Mitschnitte vom Analogfernsehen mit einer Matrox Rainbow Runner gemacht. Da hatte ich immer einen Versatz, der mit der Zeit immer größer wurde. Meine erste DVB-C-Software 2005 hat in regelmäßigen Abständen geknackst, weil sie Bild und Ton nicht synchron bekommen hat. Erst wenn der PC 20 Minuten lief, hörte das auf, war also ein Temperaturproblem. --Rôtkæppchen₆₈ 16:18, 31. Jan. 2022 (CET)Beantworten

„Verbind“

Was könnte mit „Verbind“ (taucht bei jedem Eintrag auf) gemeint sein https://i.ibb.co/QpryB8y/Ohne-Titel.jpg ? --Dr Lol (Diskussion) 14:10, 31. Jan. 2022 (CET)Beantworten

Gebinde dürfte gemeint sein ([14]). Laut meinen Infos liegt das Amt Gandersheim (den Ortsnamen lese ich als Dernhausen, aber so einen Ort scheint es nicht zu geben, ich verlese mich also wohl; wie heißt der Ort?) außerhalb des Verbreitungsgebiets des Hallenhauses (Eine Kartierung anhand von Commons-Bildmaterial habe ich vor Zeiten hier mal versucht). Ich weiß nicht, ob in der Region eine Gebind-Bauweise üblich war (oder ob sie im 18. Jahrhundert üblich gewesen ist). --::Slomox:: >< 14:28, 31. Jan. 2022 (CET)Beantworten
Ja, ein Teil der Dachkonstruktion, Grundlage zur Bezahlung der Zimmerleute ([15]). 62.157.2.126 14:31, 31. Jan. 2022 (CET)Beantworten
Dannhausen? 2A00:23C6:150B:8101:C2D1:B99D:8A84:5616 15:10, 31. Jan. 2022 (CET)Beantworten
BK: ::Der Ort ist sehr wahrscheinlich Dannhausen (Bad Gandersheim). --Proofreader (Diskussion) 15:11, 31. Jan. 2022 (CET)Beantworten
Stimmt, danke. Ich war so fixiert auf mein Dernhausen, dass ich nicht geschnallt habe, dass das ein a ist und Danhausen heißt... --::Slomox:: >< 15:33, 31. Jan. 2022 (CET)Beantworten

Bundesarchiv Best. DS/A 225 Müller, Ernst

Wo ist die Signatur Bundesarchiv Best. DS/A 225 Müller, Ernst in Invenio https://invenio.bundesarchiv.de/invenio/main.xhtml zu finden? Die Signatur stammt aus http://www.rppd-rlp.de/pkc0213 --Hamburgum (Diskussion) 15:50, 31. Jan. 2022 (CET)Beantworten

Mit dem Murks invenio lässt sich allzu häufig nichts finden, es dient nur zum Verstecken der Bestände vor möglichen Anfragen oder Benutzern, vgl. Besprechung. --Pp.paul.4 (Diskussion) 18:06, 31. Jan. 2022 (CET)Beantworten

Rechtschreibung

Guten Tag,
wird ein Patient in oder auf die oder der Kranken(haus)station aufgenommen/eingewiesen ? Was ist richtig ?
Zusatzfrage : beschreibt Solschenizyn in Krebsstation eine Abteilung innerhalb eines Krankenhauses oder eine in einem gesonderten Pavillon untergebrachte Krankenstation ? Letzteres suggeriert in verschiedenen Sprachen die Übersetzung des Titels (fr/es/it/pt). Besten Dank im Voraus, --Bohème21 (Diskussion) 16:21, 31. Jan. 2022 (CET)Beantworten

IMO nur nach Gefühl: Auf eine Station aufgenommen und in ein Krankenhaus eingewiesen ...Sicherlich Post 17:10, 31. Jan. 2022 (CET)Beantworten
Was ist gemeint? Der "geographische" Vorgang ("Sie gehen in die Intensivstation und dort das dritte Zimmer rechts.") oder der administrative Vorgang ("Wir mussten Herrn Müller auf die Intensivstation verlegen." "Auf dieser Krankenhausstation fehlt Personal."). Pte. Salt (Diskussion) 17:39, 31. Jan. 2022 (CET)Beantworten
Ohne es in vorliegenden Fall genau zu wissen: In sehr vielen Fällen gibt es nicht die eine Präposition, sondern mehrere Varianten sind denkbar. 80.71.142.166 18:48, 31. Jan. 2022 (CET)Beantworten
Was meinst Du mit "denkbar"? Es soll hier doch wohl um die korrekte Zuordenung gehen und nicht um irgendwelche Gedankenspiele. --88.68.87.67 02:05, 1. Feb. 2022 (CET)Beantworten
Einen irgendwie gearteten "geographischen" Vorgang, bei dem man in die Intensivstation geht, gibt es nicht. Das hat Pte. Salt sich ausgedacht. --88.68.87.67 02:05, 1. Feb. 2022 (CET)Beantworten
<quetsch> Ganz klares Jein! Bei denen wäre ich lieber "drin" als "drauf". scnr -- Nightflight to Venus (Diskussion) 11:02, 1. Feb. 2022 (CET)Beantworten
Auch hier gilt: im Flugzeug, auf Station, im Bett. --88.68.87.67 03:25, 2. Feb. 2022 (CET)Beantworten
Ach, ich will nicht mit dir streiten, im Normalfall hast du ja Recht. Aber diese mobilen resp. fliegenden Intensivstationen bilden die Ausnahme. "Der Patient wurde in einer fliegenden Intensivstation ins Heimatland überführt; die Anschlagsopfer wurden in einer mobilen Intensivstation erstversorgt". Mit der Hoffnung, dass wir es niemals nötig haben, -- Nightflight to Venus (Diskussion) 04:09, 2. Feb. 2022 (CET)Beantworten
Aus dem Web: "Wenn Sie als Besucher in die Intensivstation [hinein] kommen, so werden Sie unter Umständen mit einem ungewohnten Erscheinungsbild oder mit einem ungewohnten Verhalten des Patienten konfrontiert sein."
Nein, ich habe mir vorher Beispiele angesehen. Ich sehe diesen Unterschied ähnlich wie das englische in/into, also ein Zusatz von "hinein" ist möglich. Oder auch "mitten" wie in Weisser Kittel, blaue Maske: Chefarzt Stephan Jakob steht mitten in der Intensivstation des Berner Inselspitals. Ich gebe zu, dass es nicht immer exakt verwendet wird. Pte. Salt (Diskussion) 10:52, 1. Feb. 2022 (CET)Beantworten
Man kann für jeden noch so bekloppten Sprachfehler Anwendungsbeispiele finden. Dadurch wird der Fehler nicht richtiger. Recherche ist ganz offenbar nicht Deine Stärke und eigene Erfahrungen im Medizinbetrieb hast Du ganz offensichtlich auch keine, also halte Dich doch bitte bei der Beantwortung dieser Wissensfrage zurück mit Deinen zusammengegoogelten Ansichten. --88.68.87.67 03:25, 2. Feb. 2022 (CET)Beantworten
Richtig ist: keine Leerzeichen vor mehrteiligen Satzzeichen. --95.116.29.20 21:43, 31. Jan. 2022 (CET)Beantworten
Kannst du das bitte genauer sagen? Geht es um den Doppelpunkt?--Blue 🔯 22:52, 31. Jan. 2022 (CET)Beantworten
Wie gut das Leerzeichen keine Satzzeichen sind... -Ani--46.114.155.76 06:49, 1. Feb. 2022 (CET)Beantworten
"Das Leerzeichen" ist richtig - "Zeichen" hat das neutrale Genus. Ansonsten: Satz kein Satz und völlig wirr. --77.0.249.48 10:29, 1. Feb. 2022 (CET)Beantworten
Der Artikel Leerzeichen steht nicht ohne Grund in der Kategorie:Satzzeichen, denn es muß aktiv mittels eines Spatiums gesetzt werden, wenn ein Leerzeichen im fertigen Text erscheinen soll. --88.68.87.67 03:25, 2. Feb. 2022 (CET)Beantworten
Geht woanders spielen. --88.68.87.67 02:05, 1. Feb. 2022 (CET)Beantworten
Korrekt ist: im Krankenhaus, auf Station, im Zimmer. --88.68.87.67 02:05, 1. Feb. 2022 (CET)Beantworten

Guten Tag, vielen Dank für die Aufmerksamkeit, Beteiligung und richtige Antwort, zu der ich inzwischen auch einen Nachweis gefunden habe. Es sei für das Mitmachen auch jenen gedankt, die ungefragt wenig hilfreiche vom Thema abweichende Diskussionen führen, mit dem freundlichen Hinweis, das dies in der Auskunft wohl fehl am Platze ist und unterlassen werden sollte. Das Leerzeichen setzt ein aus guten Gründen auf Standardsprache Französisch eingestellter PC automatisch und es möge mir gnädigst vergeben werden, dies nicht korrigiert zu haben. Beste Grüße und frohes Schaffen --Bohème21 (Diskussion) 12:01, 2. Feb. 2022 (CET)Beantworten

Darf man Verdächtige als Mörder bezeichnen?

Guten Abend,

eine Frage zum Polizstenmord von RLP.

[16], hier schreibt die BILD sie haben den Mörder, darf die Zeitung dass?

Ich meine es ist doch noch gar nicht klar ob er der Mörder ist, hier wird sogar der Klarname verwendet, und er ist doch nur der Verdächtige? Wie ist die Rechtslage?--77.181.38.187 17:52, 31. Jan. 2022 (CET)Beantworten

Pressekodex, Richtlinie 13.1 sagt: "Die Berichterstattung über Ermittlungs- und Gerichtsverfahren dient der sorgfältigen Unterrichtung der Öffentlichkeit über Straftaten und andere Rechtsverletzungen, deren Verfolgung und richterliche Bewertung. Sie darf dabei nicht vorverurteilen. Die Presse darf eine Person als Täter bezeichnen, wenn sie ein Geständnis abgelegt hat und zudem Beweise gegen sie vorliegen oder wenn sie die Tat unter den Augen der Öffentlichkeit begangen hat. In der Sprache der Berichterstattung ist die Presse nicht an juristische Begrifflichkeiten gebunden, die für den Leser unerheblich sind." Nach meinem Verständnis sind hier die Vorraussetzungen nicht gegeben, d.h. korrekt wäre die Bezeichnung "Tatverdächtiger", wie z.B. im SPIEGEL [17]. --Proofreader (Diskussion) 18:17, 31. Jan. 2022 (CET)Beantworten

Und die nächsten Stufen wären dann "Beschuldigter" und "Angeklagter". Die Rechtslage ergibt sich allerdings nicht aus dem Pressekodex sondern aus dem Gesetz (vgl. unschuldsvermutung). --84.58.54.112 19:11, 31. Jan. 2022 (CET)Beantworten
Weiter unten steht sogar in dem Blatt "Tatverdächtiger". Für die Schlagzeile könnte es sich vielleicht eine Rüge oder sogar Klage einfangen. Und darauf wird es - wie immer - scheißen. --46.79.42.178 18:44, 31. Jan. 2022 (CET)Beantworten

Seit wann schert sich Bild (≠ Bild-Zeitung) um Recht und Gesetz? Oder Anstand? Gruß --Schniggendiller Diskussion 18:25, 31. Jan. 2022 (CET)Beantworten

Die Bild ist glücklicherweise nicht mehr das Maß der Dinge. Auflage der Bild: Entwicklung der verkauften Auflage[1]
Hier fehlt eine Grafik, die leider im Moment aus technischen Gründen nicht angezeigt werden kann. Wir arbeiten daran!
80.71.142.166 18:45, 31. Jan. 2022 (CET)Beantworten
  1. laut IVW, jeweils viertes Quartal (Details auf ivw.de)
  2. Und darunter jetzt bitte die Aufrufzahlen von bild.de und ihren Ablegern setzen. Printausgaben gehen überall zurück, das sagt null über relative Beliebtheit aus. --Kreuzschnabel 19:23, 31. Jan. 2022 (CET)Beantworten
    (BK) Mach's dir mal nicht zu einfach. Die Reichweite der Bildzeitung, ebenfalls nach Statista, lag 2021 bei immer noch 7,82 Millionen. --Aalfons (Diskussion) 19:25, 31. Jan. 2022 (CET)Beantworten
    Darf sie nicht. Macht sie aber, wie immer. Sie hat damit bekanntlich auch schon Unschuldige sozial zugrunde gerichtet, das ist ihr aber wurscht. Du kannst Klage beim Deutschen Presserat einreichen, dann erscheint übermorgen in der BILD in 5-Punkt-Schrift unterm Impressum eine Gegendarstellung. --Kreuzschnabel 19:18, 31. Jan. 2022 (CET)Beantworten
    @Kreuzschnabel: Beim Deutschen Presserat legt man Beschwerde ein. [18] --2003:E7:BF06:A3CC:64EC:6C67:8FCC:25FB 21:07, 1. Feb. 2022 (CET)Beantworten
    Von mir aus legt Gurken ein :) ist klar, was gemeint war, oder? --Kreuzschnabel 21:09, 1. Feb. 2022 (CET)Beantworten
    Ich habe mir nach dem Tod Raimund Harmstorfs, den die Bild-Zeitung in den Tod getrieben hat, geschworen, nie wieder eine Bild-Zeitung anzurühren, und habe es bis heute eingehalten. 80.71.142.166 20:38, 31. Jan. 2022 (CET)Beantworten
    Ja klar: Er nimmt Parkinsonmedikamente ein, die ihn zu einem Suizidversuch veranlassen, und sucht deswegen eine psychiatrische Klinik auf. BILD berichtet darüber. Anschließend nimmt er die Medikamente weiter ein und suizidiert sich. Völlig klar: BILD hat ihn in den Tod getrieben, und das schon vor einem Vierteljahrhundert! (An den Hexenprozessen im Mittelalter war BILD aber zufällig nicht schuld?) --95.116.29.20 21:51, 31. Jan. 2022 (CET)Beantworten
    BILD hatte was damit zu tun: Raimund Harmstorf#Tod.--BanditoX (Diskussion) 22:50, 31. Jan. 2022 (CET)Beantworten
    Da steht nichts von Hexenprozessen. Da wird nur mit substanzlosem Geraune versucht, einen zu inszenieren. --77.0.249.48 03:32, 1. Feb. 2022 (CET)Beantworten
    Er kann nicht Klage beim Presserat einreichen. Klagen reicht man beim Salzamt ein. --95.116.29.20 21:36, 31. Jan. 2022 (CET)Beantworten

    Der Pressekodex ist aber nur eine freiwillige Selbstverpflichtung, kein Gesetz. --MrBurns (Diskussion) 20:52, 31. Jan. 2022 (CET)Beantworten

    Natürlich gilt im Prozess vor einem neutralen Richterkollegium die Unschuldsvermutung, das muss so sein und dürfte jedermann einsichtig sein. Wenn aber die Tat so klar ist wie hier, wo Selbsttötung, Unfall, Notwehr etc. ausgeschlossen ist, ist klar, dass es ein Mord ist, der aber dem Täter noch bei Gericht nachgewiesen werden muss. Der mutmaßliche Mörder könnte also jeden verklagen, der ihn als Mörder bezeichnet, das funktioniert aber auch nur bis zu seiner Verurteilung. Dazu muss er aber irgendwie in Erscheinung treten, versteckend auf der Flucht geht das nicht. Und wenn ein nur „des Mordes Beschuldigter“ solche eine Zeitung verklagt, muss er aufpassen, dass er dabei keine Aussagen macht, mit denen er sich selbst belastet. Außerdem würde man überlegen, ob solche Klage unter dem Aspekt der Verhältnismäßigkeit Erfolg hat. Noch ein Beispiel zu Schluss: Heinrich Himmler werden sicher alle nur als „mutmaßlichen Mörder“ bezeichnen, denn es ist durchaus fraglich, ob gegen ihn eine Beschuldigung wegen eines Tötungsvorwurfs erhoben und diese darüber hinaus noch als Mord bezeichnet worden wäre, wenn er nicht vorzeitig den Tod gefunden hätte. --2003:D0:2F18:57E0:84D:9E9D:F26:7E85 01:19, 1. Feb. 2022 (CET)Beantworten

    Was für ein Unsinn. Selbstverständlich ist Heinrich Himmler ein Mörder gewesen, weil er gemeinschaftlich Massenmorde organisiert hat. Unser Artikel schreibt durchaus richtig: Er ist einer der Hauptverantwortlichen für den Holocaust, den Porajmos, die Ermordung von Millionen von Zivilisten und Kriegsgefangenen im Rahmen seines Generalplans Ost sowie für zahlreiche andere Verbrechen gegen die Menschlichkeit. Alles andere ist rechtsextreme Relativiererei. Es gibt auch so etwas wie ein historisches Urteil, das dazu berechtigt, einen Mörder als einen Mörder zu bezeichnen. Das gilt nicht nur für die Naziverbecher. Und in einem rechtsstaatlichen Kontext gibt es dieses Grundprinzip ohnehin erst seit 1948. Und noch etwas: Selbstverständlich gilt die Unschuldsvermutung auch außerhalb eines Strafprozesses bis zu einem anderslautenden rechtskräftigen Urteil. Eine Mißachtung kann sowohl strafrechtliche als auch zivilrechtliche Folgen haben bis hin zu der Frage der Unvoreingenommenheit von Gerichten. Drittens: Zum Zeitpunkt der Veröffentlichung war nichts so klar, wie du es darstellst. Die Fahndung war noch nicht abgeschlossen, um mögliche weitere Mittäter zu ermitteln. Diese Frage war noch offen. Und der Tathergang war nicht abschließend geklärt. So ziemlich alles an deinem Beitrag steht also auf tönernen Füßen. --84.58.54.112 04:37, 1. Feb. 2022 (CET)Beantworten
    Bitte spar Dir krampfige Worthülsen wie "rechtsextreme Relativiererei", wenn Verharmlosung gemeint ist. Inflationärer Gebrauch von Kampfbegriffen führt für gewöhnlich dazu, daß keiner sowas mehr ernst nimmt. --88.68.87.67 04:04, 2. Feb. 2022 (CET)Beantworten
    Du wirfst hier zwei unterschiedliche Sachen gefährlich durcheinander! Dass die Tat ein Mord ist, wird von niemandem bezweifelt und bestritten, und dass der oder die dafür Verantwortliche(n) Mörder ist/sind, bestreitet auch keiner. Aber solange nicht feststeht, dass die gestern festgenommene reale Person auch tatsächlich diejenige ist, die die Tat begangen hat oder daran beteiligt war, ist es nach allen mir erreichbaren Maßstäben vollkommen daneben, sie schon aufgrund dieser Festnahme als „Mörder“ zu bezeichnen. Ich persönlich erinnere mich mit Grausen an den Kindsmord in Emden 2012, wo ein 17-jähriger unter Tatverdacht festgenommen und nicht zuletzt aufgrund einer reißerischen BILD-Schlagzeile fast gelyncht wurde, bevor sich schließlich herausstellte, dass er keinesfalls der Täter war. --Kreuzschnabel 07:46, 1. Feb. 2022 (CET)Beantworten
    Ob es Mord oder Totschlag oder was auch immer ist, muß erst einmal ermittelt werden. (Mal so als Denkanregung: - Die toten Polizisten wollten den (oder die) Täter erpressen / ausrauben / ermorden, die haben sich gewehrt. - Oder: Sie gingen von einer falschen Polizeikontrolle durch Fake-Polizisten aus. - Völlig absurd? Woher willst Du das wissen? Warst Du dabei? - Deshalb sind sorgfältige objektive Ermittlungen so wichtig.) Die Verteidigung ist übrigens deutlich im Vorteil: die Toten können keine Aussage mehr machen. Und deswegen raten Strafverteidiger Tatverdächtigen auch immer dazu, bei der Polizei keine Aussage zu machen. Ein strafmilderndes Geständnis können sie im Prozeß immer noch ablegen, da brennt nichts an, aber vorlaut geäußertes Täterwissen kriegt man nicht so einfach aus der Welt. --77.0.249.48 10:49, 1. Feb. 2022 (CET)Beantworten
    Was soll das denn sein, der "Aspekt der Verhältnismäßigkeit"? Und wieso sollte es um eine Zivilklage gehen? --77.0.249.48 10:17, 1. Feb. 2022 (CET)Beantworten

    Ja, darf man! Die Unschuldsvermutung gilt nur vor Gericht und dort auch nur im Strafverfahren. Medien wie der BLÖD sind Pressekodizes regelmäßig wurscht. GG und StPO gelten nur für den Staat, nicht für mich! Denn ich brauche mich an die Unschuldsvermutung nicht zu halten! --Heletz (Diskussion) 08:35, 1. Feb. 2022 (CET)Beantworten

    Nein, darf man nicht einfach. Es gibt den Straftatbestand der üblen Nachrede. Wenn etwas noch nicht bewiesen ist und man es trotzdem über jemanden behauptet und das dessen Ruf schadet, ist das strafbar. Der BILD ist das natürlich egal, da die gute Anwälte bezahlen können die das schon auf höchstens eine Geldstrafe hinbiegen.--Naronnas (Diskussion) 08:57, 1. Feb. 2022 (CET)Beantworten
    Und dann stellt sich natürlich noch die Frage, wer denn der mutmaßliche Täter bei der "üblen Nachrede" ist... --77.0.249.48 10:20, 1. Feb. 2022 (CET)Beantworten
    Warum? Inwiefern? --Digamma (Diskussion) 20:54, 1. Feb. 2022 (CET)Beantworten
    Weil man natürlich "die Zeitung" nicht vor Gericht stellen und strafrechtlich belangen kann, denn dafür braucht man Tatverdächtige, denen ihr Verschulden nachgewiesen werden muß. (Ich habe selbst stets sorgfältig darauf geachtet, daß von mir gefälsc "gestaltete" Fotos ohne Autorennennung abgedruckt wurden. Zum Glück sind die Leute meistens aber auch zu dämlich, zu erkennen, daß ein bestimmtes Foto gar nicht das zeigen kann, was die Bildunterschrift aussagt, weil in der vorgeblichen Situation das Foto gar nicht gemacht werden konnte. Bei Bildern geben sich die Leute i. a. damit zufrieden, daß sie stimmig aussehen. Meine Kunden wissen das natürlich, die sind ja nicht doof...) --77.8.217.248 11:11, 2. Feb. 2022 (CET)Beantworten

    Cumbre Vieja, La Palma, Geologie

    Bekannt ist eine geologische Instabilität an der Westküste La Palmas und „dass sich die Flanke langsam Richtung Meer bewegt“. (Egar Zorn am 1. Oktober 2021 im dlf). Bekannt ist jedoch auch, dass die seriöse Geologie sagt, die Flanke könne und werde zwar irgendwann abrutschen, in naher Zukunft sei das wohl nicht erwartet und dass andererseits (auch hinsichtlich der oft kolportierten Katastrophenszenarios als Folge wie ein Tsunami an der Ostküste der USA) ohnehin bisher keine sicheren Prognosen über einen Zeitpunkt möglich sind. Nun gehe ich davon aus, dass man die Bewegungen der Flanke, die ja messbar sind, während des und nach dem Vulkanausbruch beobachtet hat, weil die Frage der Wirkung des Ausbruchs auf das problematische Phänomen ja von großem Interesse ist. Ich finde aber nirgendwo vorläufige Ergebnisse solcher Beobachtungen. Ist es noch zu früh dafür oder steht so etwas nur in der Fachpresse? --84.58.54.112 19:04, 31. Jan. 2022 (CET)Beantworten

    Ich glaube, wenn so etwas aktuell publiziert wäre, hätten es die Autoren von en:Cumbre Vieja tsunami hazard bemerkt; dort findet sich aber nichts vermerkt. --Pp.paul.4 (Diskussion) 21:21, 31. Jan. 2022 (CET)Beantworten

    Wie lang war die Amtszeit von Helmut Kohl?

    Im Artikel Bundeskanzler (Deutschland) steht 5870 wohingegen im Artikel Helmut Kohl 5869 steht. Was ist nun richtig? --2A00:20:B04F:F6E8:98A9:2793:4E9C:9F44 21:27, 31. Jan. 2022 (CET)Beantworten

    Das ist ein Zaunpfahlfehler: Wenn ich mit MS Excel die Datumsdifferenz zwischen 27.10.1998 und 1.10.1982 ausrechne kommt 5869 raus, dazu muss man aber 1 addieren, weil er ja sowohl am ersten als auch am letzten Tag im Dienst war, also war seine Amtszeit 5870 Tage. Man könnte es auch so rechnen dass man eben die Differenz nimmt (weil z.B . wenn man 24h im Amt ist ist man einen Tag lang im Amt aber an 2 Tagen) aber ich glaub das ist unüblich. Oder Kurz: 5870 ist richtig. --MrBurns (Diskussion) 21:35, 31. Jan. 2022 (CET)Beantworten
    Laut unserer Artikel wurde Helmut Kohl am 1. Oktober 1982 zum Bundeskanzler gewählt. Sein Nachfolger Gerhard Fritz Kurt Schröder wurde am 27. Oktober 1998 gewählt. Das ergibt eine Amtszeit von 5870 Tagen für Helmut Kohl. --Rôtkæppchen₆₈ 00:10, 1. Feb. 2022 (CET)Beantworten
    Es steht aber auch dass er schon am 26.10. abgetreten ist. Sonst wärens 5871 Tage, da man ja üblicherweise sowohl den Antrittstag als auch den Abtrittstag mitzählt. --MrBurns (Diskussion) 01:03, 1. Feb. 2022 (CET)Beantworten
    Du widersprichst dir. --Digamma (Diskussion) 20:56, 1. Feb. 2022 (CET)Beantworten
    ich glaub eher du hast dich verlesen. Ich hab sinngemäß geschrieben dass man wenn einer 24h im Amt war könnte man das als 1 Tag zählen, man zählt es aber als 2 Tage weil er datumsmäßig an 2 Tagen im Amt war. --MrBurns (Diskussion) 05:26, 2. Feb. 2022 (CET)Beantworten

    Joseph Goebbels Mutter

    ... hat wohl bis 1953 gelebt. Weiß man, wie sie zum NS und zu der Rolle ihres Sohnes gestanden und ob bzw. wie sie sich dazu jemals öffentlich geäußert hat (etwa nach dem Krieg)?--2A0A:A541:9FFF:0:E50C:B95A:2630:6463 22:16, 31. Jan. 2022 (CET)Beantworten

    Will man's wissen? --77.0.249.48 03:35, 1. Feb. 2022 (CET)Beantworten
    War es nicht Frau Himmler, die Mutter vom Heinrich, die zur Frau Goebbels sagte: „Wissen‘S, mein Heinrich ist ja ein lieber Kerl, aber er ist jetzt mit Leuten zusammen, die sind so bißl unangenehm!“ Oder sagte sie das zur Frau Heisenberg? Jetzt weiß ich es nicht mehr genau. Aber ist es nicht egal, was so eine Mutter dachte? Sippenhaft gibt es nicht bei uns und ab einem gewissen Alter ist jeder selbst für sich verantwortlich. (Wobei berichtet wird, Nazis wie Göring oder Höß seien zu ihrer eigenen Familie geradezu liebevoll gewesen, die Kinder fielen teils wohl später aus allen Wolken, als sie realisierten, was ihre Väter getan. Auf das Urteil von Familienangehörigen kommt es also eher nicht an. Weil befangen.) --Heletz (Diskussion) 08:31, 1. Feb. 2022 (CET)Beantworten
    Hat nicht Goebbels seine Kinder getötet, bevor er selbst Suizid beging? --Digamma (Diskussion) 21:00, 1. Feb. 2022 (CET)Beantworten
    Ja. Seine Frau und er hielten das für fürsorglich.--Blue 🔯 11:24, 2. Feb. 2022 (CET)Beantworten
    Nach Frau Himmler und der Ermordung der Kinder Goebbels wurde nicht gefragt. Gruß --Bohème21 (Diskussion) 12:57, 2. Feb. 2022 (CET)Beantworten

    Impressumspflicht bei com- und org-Domänen?

    Wenn man sich das Impressum von Sonnenstaatland, Psiram, Wikimannia etc. anschaut... gibt es da keine Impressumspflicht, obwohl zur Zielgruppe sicherlich nur Leser in Deutschland gehören? --BanditoX (Diskussion) 22:55, 31. Jan. 2022 (CET)Beantworten

    Der von dir verlinkte Artikel Impressumspflicht hilft da weiter: § 5 Telemediengesetz: „Diensteanbieter haben für geschäftsmäßige, in der Regel gegen Entgelt angebotene Telemedien folgende Informationen leicht erkennbar, unmittelbar erreichbar und ständig verfügbar zu halten“. Die genannten Seiten fallen nicht unter „geschäftsmäßige, in der Regel gegen Entgelt angebotene Telemedien“. -- Chaddy · D 23:43, 31. Jan. 2022 (CET)Beantworten
    Hier ist §18 Staatsvertrag für Rundfunk und Telemedien[19] einschlägig, nicht das Telemediengesetz. --Rôtkæppchen₆₈ 23:54, 31. Jan. 2022 (CET)Beantworten
    Du meinst den Medienstaatsvertrag, der den Staatsvertrag für Rundfunk und Telemedien ersetzt hat. Hm, ja, je nach Auslegung kann man diese Websites dann womöglich als Medienintermediäre ansehen, dann würde tatsächlich der Medienstaatsvertrag greifen. -- Chaddy · D 01:21, 1. Feb. 2022 (CET)Beantworten
    Das mit dem "gegen Entgelt angeboten" (geschäftsmäßig) gilt aber mittlerweile auch als veraltet bzw. eher unbeachtlich. --Grzeszik (Diskussion) 00:00, 1. Feb. 2022 (CET)Beantworten
    Soweit ich das sehe schalten Sonnenstaatland und Psiram keine Werbung. Insofern sollten sie nicht als geschäftsmäßig anzusehen sein. -- Chaddy · D 01:21, 1. Feb. 2022 (CET)Beantworten
    Unter den Begriff „geschäftsmäßig“ fielen früher auch alle stetigen, nicht-gewerblichen Angebote. Ist das heute nicht mehr so? Zum anderen wird auf diesen oben genannten Seiten laufend über andere Personen berichtet. Die betroffenen können sich an keine ladungsfähige Anschrift wenden, da das Impressum fehlt. Liegt hier also ein Pflichtenverstoss vor? Wie könnten die betroffenen Personen dann weiter vorgehen, um den Betreiber zu ermitteln? --BanditoX (Diskussion) 01:41, 1. Feb. 2022 (CET)Beantworten
    Nein, ja, Anzeige erstatten, auf Auskunft klagen. --77.0.249.48 03:38, 1. Feb. 2022 (CET)Beantworten

    BILD und Atze Schröder

    Was ist im Nachgang aus der Affäre https://www.bild.de/unterhaltung/leute/comedian/hubertus-albers-verliert-vor-gericht-54657886.bild.html geworden? Kann man den Artikel über den Künstler mit Perücke und Sonnenbrille nun überarbeiten? --BanditoX (Diskussion) 23:13, 31. Jan. 2022 (CET)Beantworten

    Nein. Ich denke, dass der Künstler gegenüber Wikimedia bisher in seiner Eigenschaft als Kunstfigur agiert hat und daher nichts zu verändern und sein Bestreben zu respektieren ist, dass er öffentlich möglichst konsequent als Kunstfigur agieren will. Wenn der BILD-Artikel vom 31. Januar 2018 tatsächlich richtig informiert (wovon man nach meinem Eindruck bei diesem Blatt in der Regel noch nie ausgehen konnte), dann bestand ein öffentliches Informationsinteresse an dem bürgerlichen Namen, weil er, wie BILD aus dem noch nicht rechtskräftigen Urteil zitiert „die ,Rangelei‘ mit Niels Ruf nicht als ,Atze Schröder‘, sondern als Hubertus Albers vom Zaune brach.“ Zwei Aspekte sind jedoch bemerkenswert:
    Zum einen ist weder irgendein BILD-Artikel noch ein Urteil, dessen Rechtskraft noch nicht eingetreten ist, eine seriöse Grundlage für was auch immer. Vielleicht juckt es viele, den Klarnamen zu verbreiten und das vielleicht erst recht, weil er sich gegenüber Wikipedia dagegen verwehrt hat. Aber genau das geht über das Niveau der Arbeitsweise bei BILD meines Erachtens nicht hinaus und ist nicht weniger erbärmlich. Es ist kein überzeugendes Argument zu erkennen, welche konkreten Interessen eigentlich befriedigt werden und unter Maßgabe welcher ethisch-moralischen, wissenschaftlichen und journalistischen Ansprüche und Leitlinien diese Interessen auch befriedigt werden müssen, wenn bei einem Künstler, der unter seinem Künstlernamen agiert und seinen bürgerlichen Namen nicht öffentlich genannt haben will dieser Wille unterlaufen wird. Der Wert dieser Information für die Gesellschaft ist nicht nur gegen die Rechte der Person sondern auch ihre Wünsche abzuwägen, aber mir ist klar, dass dieser zarte Hauch von Zivilisation und Menschlichkeit über unserer Kultur dafür vielleicht noch viel zu dünn und verletzlich ist. Das gilt auch für mich selbst.
    Der zweite Aspekt ist der Charakter dieses Vorgangs als Paradebeispiel für den Streisand-Effekt. Denn so kann der Künstler auf die Dauer nur verlieren. Dass er von Niels Ruf angezeigt wurde war vielleicht nicht vermeidbar. Wir wissen nichts über das, was da als Rangelei bezeichnet wird und wir wissen nicht, was aus der Anzeige von Ruf eigentlich geworden ist. Bis zu diesem Punkt ist der Künstler möglicherweise noch Opfer seiner vielleicht etwas albernen und in einer profitorientierten Informationsgesellschaft schwer durchzuhaltenden Strategie. Dass er sich dann aber für eine Klage gegen ein auflagenstarkes Boulevardmedium entscheidet, das schon seit vielen Jahrzehnten im gesellschaftlichen Diskurs nicht nur als verdummend angesehen wird sondern dessen Publikationsmethoden auch mehrfach als gewissen- und verantwortungslos beschrieben werden, kommt einer Kohlhaasiade gleich. Oder der Künstler war einfach sehr schlecht beraten. Es gibt nicht nur in den USA oder in England sondern auch bei uns Ebenen der Politik und der Publizistik, in denen ein ruinierter Ruf nichts mehr bewirkt und ungeniertes Handeln nicht verhindert. In denen die Tatsache der Berichterstattung mehr zählt als der Tenor der Berichterstattung.
    Der verlinkte Artikel ist natürlich die Ohrfeige, die rausgetreckte Zunge, das Nachtreten und die Häme gegen den, der es gewagt hat, sich zu wehren und die Warnung an alle, die mit solchen Gedanken spielen. Faktisch hätte BILD aber auch den Künstler weiterhin an seiner Nase durch den Zirkusring gezogen, wenn er mit seiner Klage gegen das Blatt Erfolg gehabt hätte. Die Namensvermeidungsstrategie des Künstlers erzeugt oder verstärkt ja erst den aufmerksamkeitsökonomischen Marktwert. (Vorsicht! Das soll nicht heißen, das Opfer sei selbst schuld sondern: das Opfer ist so oder so ohnmächtig und das entscheidende Moment liegt nicht in der Verhinderung der Namensverbreitung sondern darin, im Verhältnis zu sich selbst im Reinen zu sein und zu bleiben. Ein klassischer dramatischer Topos.) Wer BILD kauft hat diesem Blatt doch schon ganz andere Klopper nachgesehen. Und Mao sagt, man müsse den Feind besser kennen als sich selbst. Das hat das Handeln des Künstlers offenbar nicht bestimmt, denn das alles müsste der Künstler eigentlich wissen. Das ethisch-moralische Niveau seiner Kunst ist ja von dem Niveau der BILD-Zeitung nicht so weit entfernt und Überschneidungen bei den Konsumentengruppen der beider Protagonisten wären sicher nicht überraschend. Weshalb - als praktisches, das Lernen beflügelnde Beispiel für den unvermeidlichen Streisand-Effekt - sein bürgerlicher Name vielleicht etwas eleganter aber nicht weniger erbärmlich als bei der BILD-Zeitung hier in diesen Beitrag eingestreut ist. Da dürfen jetzt diejenigen ran, die gerne den russischen oder chinesischen Weg gehen. --84.58.54.112 03:55, 1. Feb. 2022 (CET)Beantworten
    Gut argumentiert, wenn auch mit etwas viel Rotwein geschrieben. Im Wechselspiel zwischen dem Idealismus, sich selbst schützen zu können, und einer gnadenlos ausgespielten Medienmacht nehmen Gerichte sicherlich nur eine Zuschauerrolle ein. 80.71.142.166 07:19, 1. Feb. 2022 (CET)Beantworten
    "Sich verwehren" ist unidiomatisch bzw. furniertes Wissen. --77.0.249.48 10:02, 1. Feb. 2022 (CET)Beantworten
    Es wurden hier ja Rechtsmittel von Seiten des Künstlers angekündigt. Was ist denn nun das rechtskräftige Ergebnis?--BanditoX (Diskussion) 14:48, 1. Feb. 2022 (CET)Beantworten
    Was hast du denn dazu bisher recherchiert? --84.58.54.112 03:07, 2. Feb. 2022 (CET)Beantworten

    1. Februar

    Rätselhafte Grabinschrift

    Ich habe gerade die diese Dokumentation über den Entführungsfall Orlandi angeschaut. Bei all den absurden Wendungen und Zusammenhängen möchte ich hier nur eine Frage zu einem unscheinbaren und wohl unwichtigen Detail stellen: An den Stellen 28:37 und 32:58 ist deutlich sichtbar die Grabinschrift ENRICO DE PEDIS zu lesen, aber auf der etwas später gezeigten zerbrochenen Steintafel (33:34) von seinem Grab (?) steht dann plötzlich ENRICO DE PEDRIS. Woher kommt das zusätzliche R? Bildmanipulation? Eine zweite Tafel mit Steinmetzfehler? Wieso hat das niemand bemerkt bzw. wird dazu nichts gesagt? Hat jemand eine Idee oder weiß sogar etwas sicheres darüber? --80.218.144.56 03:33, 1. Feb. 2022 (CET)Beantworten

    Die Version mit dem "R" ist nicht "in Stein gemeißelt", sondern eine dem Bild überlagerte Filmschrift, was man u. a. daran sieht, daß alle Buchstaben völlig intakt sind und über die Ränder der vermeintlichen Schäden hinausragen. Da hat sich einfach der Filmproduzent bzw. ein Mitarbeiter vertan. (Ich sehe da übrigens keine zerbrochene Steintafel, sondern ein rissiges Blatt Papier.) --77.0.249.48 05:07, 1. Feb. 2022 (CET)Beantworten
    Sicher? Beim G siehts tatsächlich ein bisschen so aus, als ob der obere Bogen auf dem Hintergrund schwach sichtbar wäre, aber ganz eindeutig scheint mir das nicht, und ansonsten sieht es für mich wirklich so aus, als ob das eine echte zerbrochene Inschrift sein könnte. Aber vorausgesetzt, du hast recht: Warum fabriziert man so etwas, und wie kann man sich bei drei Wörtern dermaßen vertun? --80.218.144.56 05:32, 1. Feb. 2022 (CET)Beantworten
    Der Film ist von http://www.bbfilm.tv/ , du müsstest wegen Fehlern dort fragen. Der Gangster hiess definitiv De Pedis. —MBq Disk 07:08, 1. Feb. 2022 (CET)Beantworten
    Warum schreibt man "G", wenn im Namen gar keines vorkommt? (Warum macht man Fehler? Das habe ich mal eine Kommilitonin - Psychologiestudentin - gefragt. Sie antwortete, das wüßte sie nicht, sie machte keine. Womit sie übrigens recht hatte.) Wie kommt "Verschissmus" auf eine Kranzschleife? --77.0.249.48 07:45, 1. Feb. 2022 (CET)Beantworten
    Und wieso hat nur deutsch.rt.com ein Foto dieser Schleife? So zieht eine Frage die andere nach sich... –MBq Disk 16:30, 1. Feb. 2022 (CET)Beantworten
    Du hast wahrscheinlich den falschen Begriff gegoogelt oder die falsche Suchmaschine genommen. Bei mir hat [20] zahlreiche Treffer gebracht, vor und nach dem Wegklappen des Fehlertextes. --Rôtkæppchen₆₈ 16:35, 1. Feb. 2022 (CET)Beantworten

    Ukraine vs. Russland

    Was kann uns die Ukraine im Gegensatz zu Russland bieten? Rein wirtschaftlich sollte sich man doch eher mit den Russen gut stellen. Warum stellt man sich nicht deutlicher auf die Seite Russlands und nimmt endlich Nord Stream 2 in Betrieb?

    --2A00:20:B04F:F6E8:98A9:2793:4E9C:9F44 09:29, 1. Feb. 2022 (CET)Beantworten

    Du suchst das Wikipedia:Café. --Rôtkæppchen₆₈ 10:01, 1. Feb. 2022 (CET)Beantworten
    Dieser Abschnitt kann archiviert werden. Hier falsch. --Rôtkæppchen₆₈ 10:01, 1. Feb. 2022 (CET)

    Akkus testen, aber wie?

    Ich habe ein kleines Prüfgerät, das kann Stromstärke, Spannung und Widerstand messen. Nun habe ich mehrere alte Akkus, Größe AA und AAA, vor mir. Kann ich nun mit diesem Gerät
    a) messen, wie voll der Akku ist, also ganz voll, halb voll oder fast leer? und
    b) feststellen, ob der Akku reif zum wegwerfen/riseiklen ist? Wenn ja, wie? Danke! --Gruenschuh (Diskussion) 10:35, 1. Feb. 2022 (CET)Beantworten

    Dazu brauchst Du ein spezielles Prüfgerät, das die Akkus unter Last misst. Mit einem normalen Multimeter misst Du nur die Leerlaufspannung und die ist nicht oder nicht immer aussagekräftig. --Rôtkæppchen₆₈ 10:48, 1. Feb. 2022 (CET)Beantworten
    Braucht er gar nicht. Er braucht einen oder mehrere definierte Lastwiderstände, die den Akku rechnerisch in definierten Zeiten (man kann z. B. 10, 30 oder 60 min wählen), entladen. Dann lädt man ihn voll auf, schließt den Lastwiderstand an und dazu parallel das Voltmeter, liest es in regelmäßigen Abständen - z. B. alle halbe Minute - ab und notiert die Meßwerte, bis die Spannung unter Last unter 0,95 V abgesunken ist, dann bricht man die Messung ab und steckt den Akku wieder ins Ladegerät. Aus den Spannungen erhält man die Lastströme und dadurch die entnommene Ladung - kann man sich auch alles graphisch darstellen. Und wenn sich dabei zeigt, daß die Kapazität arg gering oder der Innenwiderstand übel hoch sind, dann darf man sich von der Zelle auch mal verabschieden. Rechenbeispiel: Die Zelle mag nominell eine Kapazität von 1500 mAh haben. Um die in 30 min zu entladen, muß sie mit rund 3000 mA belastet werden. Bei einer Spannung von 1,2 V unter Last braucht man also eine Bürde von 400 mOhm, die 3,6 W Leistung verträgt - man nimmt also einen Widerstand 0,42 Ohm/5 W und mißt den erst einmal genau. Entsprechend für andere Zeiten. - Andere rechnen so: Eine neue Zelle kostet zwei Euro. Wieviel Theater soll man denn machen, wenn eine gebrauchte zickt? Weg damit (ggf. an Bastler mit zuviel Zeit verschenken), neue her... --77.0.249.48 11:16, 1. Feb. 2022 (CET)Beantworten
    Den Ladezustand (noch entnehmbare Restkapazität) einer Akkuzelle kann man prinzipiell nicht messen. Bei Anwendungen, bei denen es darauf ankommt, "zählt" eine Meßschaltung die seit der letzten Aufladung bereits entnommene Ladungsmenge und berechnet bzw. schätzt daraus die noch verfügbare Restmenge. Die gleiche Schaltung (Digitaltechnik mit Mikroprozessor etc.) kann auch die aktuelle Akkukapazität alternder Akkus schätzen, indem sie mißt, wieviel Ladung zwischen "entladen" und "voll" aufgenommen wurde. Und natürlich gibt es teure Lade- und Prüfgeräte, die solche Funktionalitäten eingebaut haben. Wer's - für eine im Grunde überholte Akkutechnologie - braucht... --77.0.249.48 11:30, 1. Feb. 2022 (CET)Beantworten
    Akkutechnologie überholt? OK, mag sein. Aber, liebe 77er URL, was schlägst du jetzt als Akkuersatz vor, ohne dass ich alles wegwerfen muss, das die Akkus verwendet? --Gruenschuh (Diskussion) 13:36, 1. Feb. 2022 (CET)Beantworten
    Klugschiss am Rande: Das ist eine IP, keine URL. Eine URL ist so was wie fußmarsch://kühlschrank.küche.wohnung/tür/unten/bierflaschen/1.bttl --Kreuzschnabel 14:06, 1. Feb. 2022 (CET)Beantworten
    Sorry, Kreuzschnabel, war ein Freudscher Fehler. Ich kann wohl URL und IP unterscheiden. Hab schneller getippt als gedacht. --Gruenschuh (Diskussion) 15:22, 1. Feb. 2022 (CET)Beantworten
    Dann folg doch mal meiner IP und bring mir eine mit :) --Kreuzschnabel 18:18, 1. Feb. 2022 (CET)Beantworten
    Nicht Akkutechnologie allgemein ist überholt, sondern diese spezielle Akkutechnologie (NiMH-Zellen). Stand der Technik sind LiPo-Akkus, und das mit dem Wegschmeißen kommt von ganz alleine, weil diverse Geräte ohnehin fast schneller entsorgt als neu gekauft werden, was nicht unbedingt bedeutet, daß sie defekt sind - sie sind einfach nicht mehr "modern". (Und im übrigen kann man sich auch einfach mal überlegen, ob man für manche Zwecke nicht einfach lieber Primärelemente verwenden möchte - Kosten pro Stück 20 Cent und haben ähnliche Kapazitäten wie NiMH-Zellen, sind aber weitaus nervenschonender.) --77.0.249.48 14:17, 1. Feb. 2022 (CET)Beantworten
    Nö. Alkaline-Batterien haben in Geräten ohne Entladeschlussüberwachung ungefähr einen doppelt so hohen Energieinhalt als LSD-NiMH-Akkus. Dazu haben NiMH-Akkus eine niedrigere Entladeschlussspannung als Alkaline-Batterien, weshalb für Alkaline-Betterien ausgelegte Geräte NiMH-Akkus gar nicht ganz leer machen können. Da braucht das Gerät dann viel häufiger als vorausberechnet frische Akkus. Nickel-Zink-Akkus sind auch keine Lösung, da Schnellkaputtgeher. --Rôtkæppchen₆₈ 15:38, 1. Feb. 2022 (CET)Beantworten
    Du kannst du Spannung messen, wenn er aufgeladen ist. Erreicht er nicht mehr die Nennspannung, ist er hinüber. Erreicht er die Nennspannung, muss er deswegen nicht gut sein. Jetzt kannst du ihn an einen Widerstand anschliessen und die Spannung messen. Nimmt die Spannung sehr schnell ab, ist der Akku defekt. Sodann kannst du die Zeit messen, bis seine Entladeschlussspannung erreicht wird. Ist die sehr kurz, ist er kaputt. Ist sie sehr lang, ist er ok. D.h. die einzige Möglichkeit zur Prüfung ob er wirklich noch gut ist, ist einmal voll aufladen und dann entladen und die Zeit messen. --217.138.222.28 12:35, 2. Feb. 2022 (CET)Beantworten

    Warum keine Schienenräumer?

    Ich lese gerade eine Pressemitteilung der Bundespolizei, nach der mal wieder irgendwelche verpeilten Jugendlichen Schottersteine auf die Schienen gelegt hatten, was wohl häufiger vorkommt und wegen der fliegenden Steinsplitter beim Überfahren gefährlich ist. Ich würde gerne wissen, warum nicht an der ersten Achse Prallbleche montiert sind, die in ca. 1 cm Höhe über der Schiene vor den Rändern herlaufen und so ein Zeug von den Schienen runterschubsen, damit es nicht überrollt wird. Wenn das im Laufe der Zeit verdellt wird, muß das Anprallstück halt ab und zu mal ausgewechselt werden - kann ja wohl nicht die Welt kosten... --77.0.249.48 12:11, 1. Feb. 2022 (CET)Beantworten

    Gibt es, siehe Schienenräumer. --Rudolph Buch (Diskussion) 12:39, 1. Feb. 2022 (CET)Beantworten
    Schon die reichen aber in der Regel nicht soweit herrunter, dass sie Schottersteine sicher wegkriegen. Nicht wenn sie aus festem Material sind. Das hat praktische Gründe. So die Federung und die Notwendikeit, dass sie beispielsweise auf Entgleisungsvorrichtungen fahren können müssen. Bei dem Mass an Freiraum das dafür notwendig ist, passt einfach ein Schotterstein dazwischen. Die Lössung mit Gummiräumer -wie sie bei Baumaschinen praktiziet wird-, ist nicht praktikabel da die zu schnell verschleisen.--Bobo11 (Diskussion) 12:53, 1. Feb. 2022 (CET)Beantworten
    Habe dazu auf der Diskussionsseite von Schienenräumer etwas geschrieben. Der Fehler ist offenbar, daß die Vorrichtungen am gefederten Wagenkasten befestigt sind. Das wäre aber gar nicht notwendig, die könnten auch am Achslagerträger montiert sein und dann mit sehr viel weniger Abstand von der Schienenoberfläche laufen. --77.0.249.48 13:21, 1. Feb. 2022 (CET)Beantworten
    Wenn es es etwas gibt, dass man bei einem Schienefahrzeug etwas NICHT habe will, dann ist das ungefederte Masse.--Bobo11 (Diskussion) 13:40, 1. Feb. 2022 (CET)Beantworten
    Ja klar, deswegen wurden die Radsätze auch abgeschafft... (So ein angedachtes Anbauteil wiegt vielleicht 20 kg - ja und? Was wiegen Radsatz und Achslager? Eine oder mehrere Tonnen?) --77.0.249.48 14:22, 1. Feb. 2022 (CET)Beantworten
    20kg? Schön wärs, das wird eher um die 200kg wiegen. Denn es solte such ja nicht verbiegen. Ich denke du unterschätzt was so ein Bauteil aushalten müsste. Wenn das auf einer ICE Strecke ist, dann sind das 300+ km/h bei der das Bauteil mit dem Schotterstein kolidieren würde. Da ist definitive nichts mit Leichtbau wenn es formstabil sein soll (Schottersteine die mit derselben Geschwindigkeit auf Köpfe treffen hinterlassen hässliche Löcher). Schlieslich könnte ja noch ein weiter Stein auf den Schienen liegen.--Bobo11 (Diskussion) 14:59, 1. Feb. 2022 (CET)Beantworten
    Schienenräumer am Drehgestellrahmen
    Bei hohen Geschwindigkeiten ergibt so ein Schienenräumer überhaupt keinen Sinn, da ja hier cdie Gesetze der Physik gelten. Egal, ob der Schotterstein beim Aufprall zersplittert oder nicht: Der Stein oder dessen Bruchstücke werden mit hoher Geschwindigkeit fortgeschleudert und können auch so Schaden anrichten. Insofern ist es egal, ob der Schotterstein gegen den Radreifen oder einen Schienenräumer prallt. Drehgestellrahmen mit Schienenräumer hat es aber durchaus schon gegeben, z.B. bein Maschinenfabrik Esslingen GT4. Der kam aber nur auf 65 km/h bergab mit Rückenwind. --Rôtkæppchen₆₈ 19:49, 1. Feb. 2022 (CET)Beantworten
    Nur darf auch der nur soweit herabreichen, dass Gleiseinrichtungen wie beispielsweise eine Gleissperre nicht in ihrer Funktion gestört werden. Genau da liegt bei Vollbahnen ja das Problem. Die "Auffahrbleche" (Ich mag jetzt nicht suchen gehen wie die bei der DB genannt werden) bei Entgleisungsvorrichtungen sind so dick, dass eben bei festen Schienenräumer so viel Freiraum vorhanden ist, dass nicht mehr alle Schottersteine erwischt werden können. In der Schweiz beträgt das Abstand das jedes feste Fahrzeugteil bei einer LÜ (ausser den Rädern zu den Schienen) zu festen Bauteilen (Ja, dazu gehören auch die Schienen) einhalten muss 50mm (AB EBV, Seite 343). Das heisst ein nicht zugelassenes Schienenfahrzeug (Das iste eine Variante einer aussergewöhnlichen Sendung), dass einen Schieneräumer hat, der weniger als 50mm Abstand zur Schienenoberkante hat, kann in der Schweiz nicht überführt werden. Und ind er Regel ist es so, dass Masse die bei nicht zugelassenen Fahrzeugen einzuhalten sind, sind es erst recht einzuhalten, wenn man für dieses Fahrzeugen eine Zulassung will.
    @Rotkaeppchen68 das Drehgestell ist auch bei Vollbahnen der übliche Anbauort für den Schienenräumer. Bitte nicht mit Räumschild verwechseln, der durchaus am Wagenkasten beefestigt sein kann. Denn wenn ein Räumschild bzw. Schneeräumer vorhanden ist, üÜbernehmen nicht selten die Sandrohr die Funktion des Schienenräumers. --Bobo11 (Diskussion) 21:43, 1. Feb. 2022 (CET)Beantworten
    @Bobo11: Du verlinkst mit Sandrohr auf ein Gras? --Windharp (Diskussion) 07:51, 2. Feb. 2022 (CET)Beantworten
    Uppsi, da hat sich hinter dem blauen Link mal wieder ein anderes Wort versteck. Windharp damit ist ein Teil des Sandstreuer gemeint. Die Weiterleitung ist Mist, das Wort ist mehrdeutig. --Bobo11 (Diskussion) 12:48, 2. Feb. 2022 (CET)Beantworten
    Mit den "Gesetzen der Physik" ist das auch nicht so in Stein gemeißelt: Angenommen, die Front des Schienenräumers ist eine ebene Fläche, die in Fahrtrichtung zeigt. Dann werden Steinchen auf den Schienen grundsätzlich in Fahrtrichtung geschossen, ganz oder in Bruchteilen. Da können sie wahrscheinlich weniger Schäden anrichten, als wenn Splitter eines überrollten Steins seitlich wegfliegen. (Vermutlich ist das Überrollen sowohl für Schiene als auch Radlauffläche nicht eben ersprießlich und sollte tunlichst vermieden werden - schon deswegen sollte eine Schienenputzvorrichtung eine gewisse Bedeutsamkeit haben. Am Radreifen prallen die mutmaßlich nicht ab, sondern werden in den Spalt zwischen Rad und Schiene eingeklemmt und überrollt, wobei auf jeden Fall etwas kaputtgeht, Stein bzw. sonstiger Fremdkörper, Reifen oder Schiene oder mehreres davon, und es knallt auf jeden Fall beachtlich.) Die Fluggeschwindigkeit ist auch nicht so absolut festgelegt: Wenn die Prallfläche nicht senkrecht steht, sondern ein wenig nach oben geneigt ist, werden die Fremdkörper nach oben geschossen und können dann Fang- und Dämpfungsvorrichtungen am Wagenkasten treffen, die das Geschoß entschärfen. Und letztlich hängt der Betrag der Abprallgeschwindigkeit von Masse und Härte der Prallplatte ab: Das könnte ein nachgiebiges Compoundmaterial sein, in dem anprallende Steinchen steckenbleiben. Ich würde sowas ungefähr so konstruieren: ein nach vorn zeigender Haltearm von vielleicht 2-3 kg Masse, also Größenordnung und Stabilität "Fahrradrahmenrohr", vorne ein relativ weiches Federelement, und an dem ein Prallklotz von 3-5 kg Masse angebaut. Nicht zu große Fremdkörper hinterlassen ihre Treffermarken auf dem Klotz, wobei die Federung verhindert, daß der Haltearm Schaden nimmt. Und wenn's mal heftiger rappelt, weil so ein Vollidiot Pflastersteine oder Betonplatten auf die Schienen gelegt hat (doch, das kommt vor), dann ist die ganze hübsche Konstruktion leider futsch, weil sie abgerissen wird. Na und? Macht nichts, dann wird sie eben ersetzt, Materialkosten im unteren dreistelligen Eurobereich, also Peanuts, und kommt nur selten vor. (Wenn ein Zug Gehwegplatten überfahren hat, dann ist das sowieso teuer, weil erst einmal alle froh sind, daß er nicht entgleist ist, und der zweitens vor einer Untersuchung nicht weiterfährt, sondern in die Werkstatt geschickt wird.) Abstand wegen Entgleisungsvorrichtungen und dergl.? Ja, sorry, Pech für die Vorschriften, die müssen dann leider geändert werden, sprich: da muß man sich dann halt etwas anderes einfallen lassen. Aber vielleicht gibt es auch "smarte" Alternativen zu "klassischen" mechanischen Schienenräumern: Vorstellen könnte ich mir eine schlaue Sensorik mit angeschlossener KI, die den Schienenraum vor dem ersten Fahrzeug überwacht und bei Erkennung von "geeigneten" Fremdkörpern diese von der Schiene wegschießt oder -haut. Es könnte z. B. in einem solchen Fall ein Fallarm vom Wagenkasten heruntergeklappt werden, dessen Ende dann auf der Schiene schleift und mit dem Fremdkörper beabsichtigt kollidiert, der dann nach Bereinigung des Vorfalls automatisch wieder hochgeklappt wird. Wenn dieser bewegliche Fänger genügend massiv ausgeführt ist, könnte er sogar mit "dicken Brocken" wie Pflastersteinen und Gehwegplatten fertig werden. Deren Masse ist schließlich dadurch begrenzt, daß es noch die rahmenfesten Fangeinrichtungen gibt, die kuhfängermäßig im Prinzip alles abräumen, was weiter hoch steht. --77.8.217.248 10:26, 2. Feb. 2022 (CET)Beantworten

    Was bedeutet Anametame

    Ich bin auf der Suche: was bedeutet Anametame - können sie mir helfen --87.161.62.63 13:23, 1. Feb. 2022 (CET) (nicht signierter Beitrag von 87.161.62.63 (Diskussion) 13:23, 1. Feb. 2022 (CET))Beantworten

    Gibt's nicht. Wo kommt der Begriff her? --Magnus (Diskussion) 13:27, 1. Feb. 2022 (CET)Beantworten
    Anameta ist schonmal eine ausgestorbene Spinnenart.💅...n'est pas? (nicht signierter Beitrag von 89.204.153.169 (Diskussion) 17:25, 1. Feb. 2022 (CET))Beantworten

    Streit um alte Gasrechnungen zwischen Erbe des Wohnungseigentümers und Bewohnerin

    Folgender Ablauf:

    • 2016 kaufte der Erblasser eine Eigentumswohnung.
    • Seine Lebensgefährtin zog (nachweislich) 2017 dort ein. Der Erblasser blieb in der alten Wohnung.
    • 2020 starb der Erblasser
    • Erben sind die Kinder; die LGF erhielt per Vermächtnis ein lebenslanges Wohnungsrecht in der von ihr seit 2017 bewohnten Wohnung, wobei nur sie die Verbrauchskosten zu tragen hat.
    • Ende 2021 meldeten sich die Gaswerke mit einer Rechnung von 2016 bis heute. Das Gas war nie ordentlich angemeldet worden.

    Jetzt kommt die Streitfrage, wer bezahlt welche Gasrechnungen? insbesondere für den Zeitraum:

    1. vom Einzug der LGF bis zum Tod des Erblassers
    2. vom Tod des Erblassers bis zum grundbuchlichen Eintrag des vermachten Wohnungsrechts

    Nach Auffassung der Eigentümer und Erben ist die LGF seit ihrem Einzug 2017 alleinige Nutzerin gewesen und daher auch verpflichtet zu zahlen. Lediglich der Zeitraum vom Hauskauf bis zu ihrem Einzug fällt in die vererbten Verbindlichkeiten.

    Die wohnberechtigte LGF findet dagegen, weder die Rechnungen vor dem Tod des Erblassers gingen sie etwas an, noch die aus der Zeit zwischen dem Tod des Erblassers und der grundbuchlichen Eintragung des Wohungsrechts. Lediglich ab dieser Eintragung sei sie "bereit" die Kosten zu übernehmen.

    Danke, -- Seelefant (disk.) 18:01, 1. Feb. 2022 (CET)Beantworten

    Kommt darauf an, mit wem das Gaswerk (konkludent) einen Gasabnahmevertrag geschlossen hat. Ich denke mit dem Erblasser, da die Lebensgefährtin erst später eingezogen ist. Die entstandenen Verbindlichkeiten sind damit auf die Erben übergegangen, es besteht gegen sie ein Anspruch der Gaswerke. Möglicherweise sind aber einzelne Forderungen bereits verjährt. Die Frage ist dann, ob sich die Erben etwas von der Lebensgefährtin zurückholen können. Für den Zeitraum bis zum Tode des Erblassers kommt es auf die (konkludenten) Absprachen zwischen Erblasser und Lebensgefährtin an. Wenn eine Absprache oder die allein die Praxis (aus der man dann auf eine stillschweigende Absprache schließen könnte) bestand, dass die Lebensgefährtin nichts bezahlen muss (Strom, TV, Müllabfuhr, Reparaturen etc.), dürfte gegen sie insoweit kein Anspruch gem. § 812 Abs. 1 S. 1 BGB bestehen (weil ja in Gestalt der Absprache ein Rechtsgrund vorliegt). Für die Zeit nach dem Tod ist es etwas tricky: Bestand eine (stillschweigende) Absprache zwischen dem Erblasser und Lebensgefährtin, gilt diese zunächst bis zu einer Beendigung der Absprache über den Tod des Erblassers hinaus fort. Erst nach Beendigung (die man in der Annahme des Vermächtnis sehen kann) ergibt sich dann aus dem Vermächtnis, sofern die Lebensgefährtin es angenommen hat, ein Anspruch auf Ausgleich der Gaskosten als Verbrauchskosten. Das sollte man aber möglicherweise auch vertraglich regeln. 92.79.101.164 18:15, 1. Feb. 2022 (CET)Beantworten
    Es dürfte sich hier um einen Fall der Grundversorgung handeln. Wenn der Hauseigentümer den Gasanschluss nutzt bzw. nutzen lässt ohne ihn ordentlich anzumelden, haftet er persönlich gegenüber dem Grundversorger. Mit dem Tod des Hauseigentümers ist diese Verpflichtung auf die Erben übergegangen. Einen Weg die Lebensgefährtin heranzuziehen, jedenfalls für Zeiträume vor Eintragung des Wohnrechts im Grundbuch, sehe ich hier nicht. Allerdings: Die Forderungen des Gasversorgers aus 2016 und 2017 dürften verjährt sein - die Erben können die Einrede der Verjährung gegenüber dem Gasversorger geltend machen. -- 2A02:908:121:9900:0:0:0:EED9 20:00, 1. Feb. 2022 (CET)Beantworten
    Fängt die Verjährungsfrist für Grundversorger nicht erst ab Rechnungsstellung an? -- Seelefant (disk.) 20:02, 1. Feb. 2022 (CET)Beantworten
    Negativ, für Grundversorger gibt es keine gesonderten Regelungen zur Verjährung, also greift die Regelverjährung von drei Jahren, und die wäre bei Rechnungsstellung Ende 2021 für die Jahre 2016 und 2017 bereits abgelaufen. Ab Rechnungsstellung beginnt die Fälligkeit der Forderung, d. h. vor Rechnungsstellung kann man nicht in Verzug geraten. -- 2A02:908:121:9900:0:0:0:EED9 20:16, 1. Feb. 2022 (CET)Beantworten
    Und wann beginnt die Verjährungsfrist, wenn kein Vertrag geschlossen wurde? Mit Abnahme des Gases oder mit Kenntnis des Gasversorgers von der ungerechtfertigten Abnahme des Gases (§ 199 (1) Nr. 2 BGB)? 62.157.2.126 09:12, 2. Feb. 2022 (CET)Beantworten
    Mit Entstehen des Anspruchs, das ist in dem Fall der Ablauf des Abrechnungszeitraums. Ob der Versorger monatlich oder (eher üblich) jährlich abrechnet, geht aus den AGB hervor. Dem Gasversorger muss jedenfalls ein Verschulden vorgeworfen werden, dass er die Gasversorgung nicht gesperrt hat und auch danach über einen Zeitraum von fünf Jahren, in dem regelmäßig die Zählerstände abgelesen wurden, nicht aufgefallen ist, dass da ein Zähler weiterläuft, für den kein Vertrag verzeichnet ist, er wird sich also schwerlich darauf berufen können, dass er nichts wusste. -- 2A02:908:121:9900:0:0:0:EED9 09:32, 2. Feb. 2022 (CET)Beantworten
    Bei der hiesigen Gasanstalt läuft es genau andersherum. Der Ableser hat eine nach Adresse sortierte Liste der Kunden und für jeden Kunden eine Liste der Zählernummern. Zähler ohne aktuellen Kunden tauchen gar nicht in der Liste auf und werden so weder abgelesen noch erfasst. --Rôtkæppchen₆₈ 09:41, 2. Feb. 2022 (CET)Beantworten
    Der Hinweis auf den Zähler und das Verschulden des Gasversorgers ist nachvollziehbar. Wenn ein Gasanschluß gelegt und ein Gaszähler eingebaut ist, könnte es schon unter die in § 199 BGB genannte grobe Fahrlässigkeit fallen, den Zähler nicht ab und zu mal abzulesen. 62.157.2.126 09:53, 2. Feb. 2022 (CET)Beantworten
    Wobei das hier alles Stochern im Nebel ist. Der Fragesteller schreibt ja, der Erblasser habe das Gas "nie ordentlich angemeldet" - was immer das heißt. 92.79.101.164 11:28, 2. Feb. 2022 (CET)Beantworten
    Mir fällt noch ein, dass zwischen Erblasser und Lebensgefährtin ein atypischer Mietvertrag bestand, wonach die Lebensgefährtin zumindest keine Miete zahlen sollte. Wenn man nicht ohnehin von einer (stillschweigenden) Abrede, die Lebensgefährtin habe vollkommen kostenlos dort wohnen dürfen, ausgehen muss, wären die Betriebskosten nach § 556 Abs. 3 BGB "spätestens bis zum Ablauf des zwölften Monats nach Ende des Abrechnungszeitraums mitzuteilen", d.h. es könnten von den Erben ohnehin nur noch Kosten des Jahres 2021 geltend gemacht werden. 92.79.101.164 21:39, 1. Feb. 2022 (CET)Beantworten
    Ich bin mir nicht sicher, ob man hier die mietvertragliche Regelung analog anwenden kann. Und selbst wenn, könnte die Lebensgefährtin dem Anspruch einen eigenen Schadensersatzanspruch entgegenhalten, denn: Wäre das Gas ordentlich angemeldet worden, hätte die Lebensgefährtin die Möglichkeit gehabt, zu einem günstigeren Versorger zu wechseln und nicht in der teuren Grundversorgung zu bleiben. -- 2A02:908:121:9900:0:0:0:EED9 09:40, 2. Feb. 2022 (CET)Beantworten
    Es sind im (ansonsten recht umfangreichen und vollständigen) Nachlass an Papieren keinerlei Hinweise darauf zu finden, dass jemals ein Gasvertrag abgeschlossen wurde. Es waren also sowohl der Eigner=Erblasser, als auch die Bewohnerin=Vermächtnisnehmerin in dieser Hinsicht jahrelang untätig. Wie bei dieser Konstellation der Eigner der Bewohnerin dabei im Weg gestanden haben soll, sich um einen günstigeren Anbieter zu bemühen, erschliesst sich mir nicht. -- Seelefant (disk.) 12:01, 2. Feb. 2022 (CET)Beantworten
    Zum Abschluss eines Gasvertrags braucht man die Zählernummer. Die hat nur der Eigentümer, denn der Gaszähler ist (zumindest in den Gebäuden, die ich kenne) verschlossen und nur mit einem gesonderten Schlüssel zugänglich. -- 2A02:908:121:9900:0:0:0:EED9 12:18, 2. Feb. 2022 (CET)Beantworten

    Versorger haben gelegentlich seltsame Vorstellungen. Zwei Vorkommnisse, die ich erlebt hatte: Ein Umzug fand in einem Schaltjahr statt, der jährliche Grundpreis wurde nach Tagen anteilig aufgeteilt. Nur erhöhte der programmierte Algorithmus dabei mal eben den Grundpreis um 1/365, weil er den anteiligen täglichen Grundpreis mal 366 Tage berechnete... Ich war mal in eine Mietwohnung eingezogen, deren Vormieterin vermutlich ohne Erben verstorben war - irgendjemand hatte sie dann als Stromkundin bei den Stadtwerken abgemeldet. Als ich einzog, las ich den Zähler ab und meldete mich als neuer Kunde an. Worauf die Stromer auf die geniale Idee kamen, mir den Grundpreis für die Monate des Wohnungsleerstands und noch zwei oder drei kWh in Rechnung stellen zu wollen. Konnten die gar nicht verstehen, daß ich das mit dem Götzzitat kommentierte. (Wie das dann genau ablief, weiß ich nicht mehr, aber ich habe dafür jedenfalls nicht gezahlt.) Wobei es dabei letztlich nur um Kleingeld ging. Anders sähe es natürlich aus, wenn eine Wohnung im Winter monatelang leer steht und darin dann eine Heizung läuft, also richtig Energie verbraucht wird. Das kann richtig Ärger geben. --77.8.217.248 10:51, 2. Feb. 2022 (CET)Beantworten

    Dass ein Schadensersatzanspruch besteht, kann man anhand des mitgeteilten Sachverhalts nicht beurteilen, und es ist zunächst auch einmal eine Unterstellung, der betreffende Gaslieferant sei nicht der günstigste. Der Vermieter ist nach § 556 Abs. 3 BGB nicht verpflichtet, immer das günstige Angebot zu wählen, sondern er muss den Wirtschaftlichkeitsgrundsatz beachten. Im Münchener Kommentar heißt es hierzu (Zehelein in MüKoBGB, 8. Aufl. 2020, § 556 Rn. 117 f.):
    "Keiner ordentlichen Geschäftsführung entspricht es, wenn sich der Vermieter auf unangemessene, marktunübliche oder überhöhte Entgeltvereinbarungen mit Dritten einlässt (zB zu hohe Grundkosten im Wärmelieferungsvertrag). Der Preis muss in das Verhältnis zum Leistungsinhalt gesetzt werden. Der Vermieter muss nicht unbedingt den billigsten Anbieter wählen, darf aber auch nicht ohne sachlichen Grund ein günstigeres Angebot ausschlagen. Da sich ein exakt angemessenes Entgelt kaum abstrakt ermitteln lässt, wird es unter Heranziehung der Wesentlichkeitsgrenze des § 5 WiStG zugelassen, dass die aufgewendeten die üblichen Kosten um 20 % übersteigen können. Dies kann jedoch nicht als starre Grenze angesehen werden. Der Vermieter ist aus dem Wirtschaftlichkeitsgebot heraus grundsätzlich zu kostengünstiger Beschaffung, auch unter Ausnutzung besonders günstiger Beschaffungsmöglichkeiten verpflichtet, soweit ihm dies zumutbar ist. Maßgeblicher Ansatzpunkt für die Kontrollüberlegung ist daher keine Wertgrenze, sondern die Frage, ob ein verständiger Vermieter die Kosten auch veranlasst hätte, wenn er sie selbst tragen müsste. Da im Rahmen der Wirtschaftlichkeit auch der Verwaltungsaufwand zu berücksichtigen ist, kann vom Vermieter jedoch nicht verlangt werden, dass er alle denkbaren Anbieter zur Abgabe eines Angebots auffordert. Es muss aber zumindest eine ausreichende Anzahl von Angeboten eingeholt werden, um sich einen Marktüberblick zu verschaffen. Eine Ausschreibung ist nicht erforderlich.
    Der Wirtschaftlichkeitsgrundsatz gilt nur innerhalb der vom Vermieter gewählten Versorgungsart und verpflichtet den Vermieter nicht, schon bei der Auswahl die wirtschaftlich vorteilhafteste Versorgungsvariante zu wählen."
    92.79.101.164 11:22, 2. Feb. 2022 (CET)Beantworten
    Es gibt keinen Vermieter. Wir reden von einem ins Grundbuch eingetragenen, lebenslangen Wohnungsrecht. -- Seelefant (disk.) 11:56, 2. Feb. 2022 (CET)Beantworten
    Da Du die Rechtslage nicht ganz umreißt, wäre ein Besuch beim Anwalt umso dringlicher. Es geht um die Zeit, als der Erblasser noch lebte: Da gab es ja, so wie Du das schilderst, noch kein dingliches Wohnrecht zugunsten der Lebensgefährtin, sondern irgendeine andere Grundlage für die Überlassung der Wohnung: Das kann eine reine Gefälligkeit, eine Leihe oder eine Miete gewesen sein - abschließend kann man das hier nicht beurteilen. Wenn es keine expliziten Vereinbarungen gab, kann man auf solche auch aus den Umständen schließen, also daraus, wie die Überlassung und die Übernahme von (sonstigen) Kosten zwischen Erblasser und Lebensgefährtin praktiziert wurden. 92.79.101.164 12:01, 2. Feb. 2022 (CET)Beantworten
    Es sind im (ansonsten recht umfangreichen) Papiernachlass keine schriftlichen Vereinbarungen über die Wohnung zu finden, die sich auf die Lebenszeit des Erblassers beziehen. Allerdings zog die Lebensgefährtin nachweislich in dem Moment in das Haus, in dem ihr erbvertraglich das Wohnrecht für den Todesfall des Erblassers vermacht worden war. -- Seelefant (disk.) 12:04, 2. Feb. 2022 (CET)Beantworten

    Zerbrochener Glaseinsatz für eine Uralt-Thermoskanne, wo bekommt man so was?

    Früher konnte man die Glaseinsätze extra nachkaufen. Heute bei Neupreisen um 8 Euro für Metallthermosflaschen anscheinend nicht mehr. Werden die noch (von wem?) erzeugt? Glas, verspiegelt mit Kunststoffhülle?

    --Luziwuzi (Diskussion) 18:21, 1. Feb. 2022 (CET)Beantworten

    Bei thermos.eu schon geschaut? --77.119.211.190 19:02, 1. Feb. 2022 (CET)Beantworten
    Ich würde mal bei Alfi (sic!) in Wertheim fragen. --77.0.249.48 19:06, 1. Feb. 2022 (CET)Beantworten

    Kann jemand (alt?)deutsche Schreibschrift lesen?

    Guten Abend. Ich wollte fragen, ob jemand hier deutsche Schreibschrift(?)lesen kann und Lust hätte, mir die zu "übersetzen". Ich schaffs nicht und Transkribus auch nicht wirklich :-(
    Ist auch kein Roman, sondern zwei A5-Seiten mit nicht soo viel Text.
    Hintergrund ist ein spannender Dachbodenfund ... eine Kassette mit allerlei Unterlagen und Feldpost vom Onkel meiner Partnerin. Der Brief ist wohl der Letzte aus dem Lazarett bei Wien(?) und muss von jemand anders geschrieben worden sein (alle anderen sind leserlich) da er laut Brief von seinem Kompaniechef mit Schussbruch rechter Oberarm zum Verbandsplatz gebracht wurde. Ende '44 ist er mit 17 im Lazarett gestorben.
    Scans sind hier: https://c.web.de/@334598271899012280/3qizgZnaRLGtV9GI271rrg Schonmal vielen Dank für die Aufmerksamkeit ;-) --Strange (Diskussion) 22:13, 1. Feb. 2022 (CET)Beantworten

    Ich lese:
    Reenhauen [?] 16. XI. 44.
    Liebe Mutter!
    Zuvor die besten Grüße und Küsse.
    Erschrecke nicht, daß ich nicht selber schreiben kann aber ich bin am rechten Oberarm verwundet so daß ich selbst nicht schreiben kann.
    Mir geht es soweit gut nur das Fieber macht mir zu schaffen aber bis das [sic] ich einen neuen Gips habe wird es schon gehen.
    Eben tragen sie mich zum Gipser.
    Und wie geht es euch?
    Wenn es dir möglich ist liebe Mutter, daß du mir ein bischen [sic] was schickst. Nur wenn es dir möglich ist.
    Was macht Vater? Schreibt er?
    Sonst weiß ich für heute nichts mehr.
    Mit besten Grüßen
    Euer Sohn Alfred.
    Liebe Frau Grußmann!
    Ich alls [sic] die Schwester Ihres Sohnes möchte auch Sie herzlich grüßen.
    Will Ihnen mitteilen, daß es ihm ganz gut geht. Nur das Fieber will nicht weg. Aber wenn er einen neuen Gipsverband bekommt wird auch das Fieber nachlassen.
    Essen will er auch nicht viel aber ich schaue schon, daß er wenigstens was zu sich nimmt. Die Kamraden [sic] sind auch sehr nett zu ihm geben ihm Kuchen Kompott u s w.
    Also in Sorge brauchen Sie nicht sein. Wenn was sein würde möchte [?] ich Ihnen sofort schreiben aber einstweilen ist es ja nicht nötig
    Seien Sie recht herzlich gegrüßt
    von Ihrer Schwester Friedl
    --Jossi (Diskussion) 01:04, 2. Feb. 2022 (CET)Beantworten
    "Nur das Fieber will nicht weg". Scheiß-Krieg! Pardon my French. -- Seelefant (disk.) 10:26, 2. Feb. 2022 (CET)Beantworten
    Kannste laut sagen! Kann mir kaum vorstellen, wie wahnsinnig hart das für die Mutter war. 1. Brief von der Kompanie: Sohn bei Kaba(?) gefallen. 3 Wochen später: Sohn doch nicht tot, nur am Arm verwundet. Noch ne Woche später kam wohl der Brief oben. Dann kam wohl nur noch: am 8.12. für FuV gestorben :-( Und der Vater war seit September 44 vermisst. --Strange (Diskussion) 11:22, 2. Feb. 2022 (CET)Beantworten
    Superklasse! Vielen herzlichen Dank, Jossi! Das vor dem Datum oben soll vermutlich Pressbaum heissen. Da gabs bei Wien ein Reservelazarett. Sieht auch im Wehrpass so aus. Auf der Sterbeurkunde steht Wien. Nochmals Dankeschön! --Strange (Diskussion) 11:03, 2. Feb. 2022 (CET)Beantworten
    Gern geschehen! „Pressbaum“ ist sehr plausibel. Was mich (als medizinischen Laien) gewundert hat, ist der Zusammenhang zwischen Gipsverbandwechsel und Höhe des Fiebers, den beide ganz selbstverständlich annehmen. Dass ein neuer Gipsverband fiebersenkend wirken soll, ist mir noch nie begegnet. Gibt es Mediziner hier, die dazu etwas sagen können ? --Jossi (Diskussion) 12:36, 2. Feb. 2022 (CET)Beantworten
    Hm, in der Kiste sind auch noch Briefe vom Vater aus der Gefangenschaft. Er wurde bei der Befreiung Antwerpens von den Briten einkassiert. In einem schreibt er: "...trotz der Amputation wäre es doch so schön gewesen, wenn Alfred nach Hause gekommen wäre" - Vielleicht wollten sie im Lazarett dem Jungen nicht sagen, dass sie ihm den Arm abschneiden würden. Oder der Mutter nicht :-( --Strange (Diskussion) 13:21, 2. Feb. 2022 (CET)Beantworten
    Es wird nicht so sein, daß ein neuer Gipsverband fiebersenkend wirkte, sondern daß man zwischen dem Wechsel des Gipses die wohl entzündete Wunde ("Schussbruch rechter Oberarm") untersuchen und vielleicht behandeln konnte. 62.157.2.126 13:25, 2. Feb. 2022 (CET)Beantworten

    2. Februar

    ab wann gelten Pflichten aus einem Vermächtnis?

    Der Erblasser bestimmte in einem Erbvertrag das Vermächtnis eines Wohnungsrechts für den Vertragspartner, der selbst zum Vollstrecker des Vermächtnisses bestimmt wurde. Der Vermächtnisnehmer nahm das Vermächtnis erst einige Monate nach dem Tod des Erblassers konkludent an, indem er das Wohnungsrecht im Grundbuch eintragen ließ. Er wohnte allerdings schon vor dem Tod des Erblassers im fraglichen Objekt, und zwar allein. Zu welchem Zeitpunkt gelten mit dem Vermächtnis verbundene Pflichten wie die Zahlung von Nebenkosten? Ab der konkludenten Annahme des Vermächtnisses, oder ab Tod des Erblassers? -- Seelefant (disk.) 10:23, 2. Feb. 2022 (CET)Beantworten

    Vielleicht solltes du dich mit Benutzer:Anonyme Frage zusammen tuen und Ihr euch einen Anwalt für Erbrecht suchen.--Oberkaffeetante (Diskussion) 10:44, 2. Feb. 2022 (CET)Beantworten
    Liebe Oberkaffeetante, ein Anwalt wird natürlich beauftragt. Wie beim Arzt ist 'ne zweite Meinung aber sicher nicht verkehrt. -- Seelefant (disk.) 10:58, 2. Feb. 2022 (CET)Beantworten
    Es ist sicher klug, neben dem Anwalt als Profi auch völlig unqualifizierte Meinungen aus dem Internet zu befragen. --212.88.147.22 11:44, 2. Feb. 2022 (CET)Beantworten
    Für ’ne zweite Meinung würde ich mir eher einen zweiten Anwalt suchen. Was du hier bekommst, hat im 2felsfall vor Gericht keinen Bestand, was die Frage aufwirft, wozu. Lies bitte oben verlinkten Rechtshinweis, insonderheit den Teil nach dem ersten Komma. --Kreuzschnabel 11:47, 2. Feb. 2022 (CET)Beantworten
    Ich unterstütze die Empfehlung, einen Anwalt einzuschalten, sehr. Ab wann die dem Vermächtnisnehmer auferlegten Pflichten gelten, bestimmt sich nach dem Inhalt und der Auslegung des Testaments. Steht drin, dass der Vermächtnisnehmer in den Genuss des Vermächtnisses nur kommt, wenn er rückwirkend irgend etwas bezahlt, ist die Sache klar. Ansonsten würde ich grundsätzlich davon ausgehen, dass Verpflichtungen ("Auflagen", § 1940 BGB) erst ab Annahme des Vermächtnisses "gelten" - spiegelbildlich zu dem Vermögensvorteil (wohnrecht), der dem Vermächtnisnehmer auch erst mit Annahme eingeräumt wird. 92.79.101.164 11:48, 2. Feb. 2022 (CET)Beantworten
    Wenn im Erbvertrag nichts abweichendes geregelt ist, frühestens ab Eintragung des Wohnrechts ins Grundbuch, denn erst ab dann gilt das Vermächtnis als angenommen. Das heißt aber nicht, dass der Vermächtnisnehmer für die Zeit davor von der Zahlung von Nebenkosten befreit wäre, denn bis zur Annahme des Vermächtnisses haben die Eigentümer einen Anspruch auf Nutzungsentschädigung gegen den geduldeten Bewohner, und der beinhaltet neben einer angemessenen Miete auch die Nebenkosten. -- 2A02:908:121:9900:0:0:0:EED9 12:12, 2. Feb. 2022 (CET)Beantworten

    Stempel auf mobiler Briefmarke?

    Werden Briefe, die mit einer „mobilen Briefmarke“ frankiert worden sind (Wenn man es so schreibt, klingt die Bezeichnung komplett bescheuert …), im Zustellungsgang eigentlich gestempelt? Google spuckt mir nur Ergebnisse zur Briefmarke mit Matrixcode („digitale Briefmarke“) aus, gemeint ist aber das #PORTO zum selbst aufschreiben. --91.221.58.22 12:43, 2. Feb. 2022 (CET)Beantworten

    Die Post scheint bei der Mobile Briefmarke einfach einen Aufkleber mit QR-Code neben die handschriftlichen Zahlencodes zu kleben (zumindest solange das noch "Handyporto" hieß, ob das jetzt in 2022 immer noch so ist weiß ich nicht). Siehe etwa das Bild.--Naronnas (Diskussion) 12:59, 2. Feb. 2022 (CET)Beantworten

    Wissen

    Automated Chatbot

    Data Security

    Virtual Reality

    Communication

    Support

    Company

    About Us

    Services

    Features

    Our Pricing

    Latest News

    © 2024 campus1.de